FP515 Retirement Flashcards

1
Q

Which of these is a type of defined contribution profit-sharing plan?

A) Cash balance pension plan

B) Employee stock ownership plan (ESOP)

C) Target benefit pension plan

D) Money purchase pension plan

A

B) Explanation
A cash balance pension plan is a type of defined benefit pension plan. An ESOP is a defined contribution profit-sharing plan.

LO 1.2.1

How well did you know this?
1
Not at all
2
3
4
5
Perfectly
2
Q

The maximum service requirement that a thrift plan may impose as a condition of participation is

A) two years.

B) one-and-a-half years.

C) six months.

D) one year.

A

A) Explanation
By law, the maximum service requirement that a thrift plan may impose is two years of service. An employee must become a participant in a thrift plan on the next plan entry date following the later of attainment of age 21 or the completion of two years of service. Of course, a more liberal participation requirement may be imposed. The maximum for a 401(k) is one year.

LO 1.3.1

How well did you know this?
1
Not at all
2
3
4
5
Perfectly
3
Q

Which of these is false regarding defined contribution plans?

A) The employer contribution limit is 25% of the participating employees’ payroll.

B) The retirement benefit is not certain; investment risk is borne by the participant.

C) Includible compensation is limited to the lesser of 100% of compensation or $230,000 in 2021.

D) The maximum allowable employee deferral amount for workers is $19,500 in 2021, not counting any catch ups.

A

C) Explanation
The maximum amount of includible compensation is $290,000, not $230,000. $230,00 is a maximum defined benefit test number in 2021.

LO 1.3.2

How well did you know this?
1
Not at all
2
3
4
5
Perfectly
4
Q

Question 4 of 20
Question ID: 1340519
Which of these statements regarding a top-heavy retirement plan is false?

A) Top-heavy plans favor key employees by providing more than 60% of the plan benefits to these employees.

B) Close scrutiny of the top-heavy rules is desirable from a planning standpoint because an understanding of these rules coupled with effective personnel decisions and plan design may enable the plan to escape top-heavy status.

C) Small-business owners are prone to shaping the organization’s retirement plan primarily to shelter taxes for themselves and key employees, thereby creating top-heavy plans.

D) Defined benefit pension plans can be top heavy; defined contribution plans cannot.

A

D) Explanation
Both defined benefit pension and defined contribution plans can be top heavy, but both must satisfy rules that apply to top-heavy plans.

LO 1.3.1

How well did you know this?
1
Not at all
2
3
4
5
Perfectly
5
Q

Benjamin Scott, age 42, earns $110,000 a year and wants to establish a profit-sharing plan. He employs four people whose combined salaries are $58,000 and who range in age from 24 to 30. The average employment period for all employees is three-and-a-half years. Which vesting schedule is best suited for Benjamin’s plan?

A) Three-to seven-year graded vesting

B) Five-year cliff vesting

C) Two-to-six-year graded vesting

D) Three-year cliff vesting

A

C) Explanation
The choices for vesting in a defined contribution plan are: (1) 100% full and immediate, (2) three-year cliff, or (3) two-to-six graded. Because of the average length of employment, the most suitable vesting schedule from Benjamin’s point of view (cash flow if termination occurs and reallocated forfeitures to Benjamin) is graded vesting. Five-year cliff vesting and three-to-seven-year graded vesting are not available to Benjamin’s company for a profit-sharing plan.

LO 1.3.1

How well did you know this?
1
Not at all
2
3
4
5
Perfectly
6
Q

Which of the following is CORRECT regarding IRS Form 5500?

A) A simplified version, Form 5500-EZ, is available for certain small employers.

B) The IRS 5500 is known as the employer’s annual return/report to the IRS of an employee benefit plan.

C) All of these statements are correct.

D) Filing an IRS 5500 is an ERISA requirement.

A

C) Explanation
All of these statements are correct regarding IRS Form 5500.

LO 1.4.1

How well did you know this?
1
Not at all
2
3
4
5
Perfectly
7
Q

If a qualified defined contribution retirement plan is found to meet the requirements under the Employee Retirement Income Security Act of 1974 (ERISA) and Internal Revenue Service (IRS) regulations, which of these statements is CORRECT?

A) The PBGC will provide coverage to the plan.

B) The participating employees must petition the IRS for acceptance of the plan.

C) The employer is permitted to require three years of service eligibility as long as employer contributions are immediately 100% vested to participants.

D) Employees are not taxed on plan contributions or earnings attributed to plan contributions as long as a plan distribution does not occur.

A

D) Explanation
Employees are not taxed on plan contributions or earnings attributed to plan contributions as long as a plan distribution does not occur. A two years of service eligibility requirement is permitted if 100% immediate vesting is also used. Employees do not petition the IRS for acceptance of the plan. No defined contribution plans are eligible for PBGC coverage.

LO 1.1.1

How well did you know this?
1
Not at all
2
3
4
5
Perfectly
8
Q

Which of the following describe differences between tax-advantaged retirement plans and qualified plans?

I. IRA-funded employer-sponsored tax-advantaged plans may not incorporate loan provisions.

II. Employer stock distributions from a tax-advantaged plan do not benefit from NUA tax treatment.

A) Both I and II

B) I only

C) II only

D) Neither I nor II

A

A) Explanation
Both statements are correct. IRA-funded employer-sponsored tax-advantaged plans are SEPs, SARSEPs, and SIMPLE IRAs.

LO 1.2.1

How well did you know this?
1
Not at all
2
3
4
5
Perfectly
9
Q

Which of the following are minimum coverage tests for qualified retirement plans?

I. Minimum allowed discrimination
II. Average benefits percentage test
III. Ratio test
IV. Maximum compensation test

A) II and III

B) II, III, and IV

C) I, II, and III

D) I and II

A

A) Explanation
The two minimum coverage tests for qualified retirement plans are the average benefits percentage test and the ratio test. To be qualified, a retirement plan must meet at least one of these tests if the plan does not meet the percentage (safe harbor) test.

LO 1.3.1

How well did you know this?
1
Not at all
2
3
4
5
Perfectly
10
Q

Which of the following are agencies that administer and ensure compliance with the federal laws that apply to qualified retirement plans?

I. Department of Labor (DOL)
II. Employee Retirement Income Security Agency (ERISA)
III. Internal Revenue Service (IRS)
IV. Pension Benefit Guaranty Corporation (PBGC)

A) I, II, and III

B) II and IV

C) I and III

D) I, III, and IV

A

D) Explanation
Under the reporting and disclosure requirements of the Employee Retirement Income Security Act of 1974 (ERISA), annual reports and summary plan descriptions are filed with the IRS (which provides copies to the DOL). An annual premium payment form is filed with the PBGC. If the defined benefit plan is not in compliance with funding requirements, the PBGC can terminate the plan. If the defined benefit plan is not in compliance with funding requirements, the PBGC can terminate the plan.

LO 1.1.1

How well did you know this?
1
Not at all
2
3
4
5
Perfectly
11
Q

Qualified retirement plans should do which of the following?

I. They must meet specific vesting requirements.
II. They have special tax advantages over nonqualified plans.
III. They must provide definitely determinable benefits.
IV. They require an annual profit to allow funding for the plan.

A) II and III

B) II, III, and IV

C) I, II, and III

D) I and II

A

C) Explanation
Qualified plans must meet specific vesting schedules. Qualified plans are preferred to nonqualified plans because of the special tax advantages enjoyed by qualified plans. Qualified retirement plans must offer definitely determinable benefits. An annual profit is not required for a qualified plan to be funded.

LO 1.2.1

How well did you know this?
1
Not at all
2
3
4
5
Perfectly
12
Q

Which of the following statements regarding nonqualified retirement plans is CORRECT?

A) Contributions are deductible by the employer when contributed to the plan.

B) Nonqualified plans do not have to meet the nondiscrimination requirements that apply to qualified plans.

C) Benefits are received by the employee income-tax-free.

D) Nonqualified plans are subject to the same Employee Retirement Income Security Act of 1974 (ERISA) requirements as qualified plans.

A

B) Explanation
Nonqualified plans do not have to meet the nondiscrimination requirements that apply to qualified plans. Nonqualified plans are not subject to all ERISA requirements applicable to qualified plans. Benefits are not deductible by the employer until paid and are includable in the employee’s taxable income at the time of receipt.

LO 1.2.1

How well did you know this?
1
Not at all
2
3
4
5
Perfectly
13
Q

Ryan, who is 50, is employed by Best Mutual Funds (BMF) and participates in its profit sharing Section 401(k) plan for 2021. The plan allocates contributions based on relative compensation and is not integrated with Social Security. Ryan’s current annual compensation is $80,000, and he has elected to defer 5% of compensation into the Section 401(k) plan. Including Ryan, 35 employees with a total covered payroll of $1.8 million participate in the plan and have elected to defer a total of $72,000 (4% of payroll). BMF’s matching contributions to the plan total $54,000 (3% of total covered payroll). What is the limit on annual additions to Ryan’s individual account?

A) $58,000

B) $13,500

C) $19,500

D) $26,000

A

A) Explanation
The limit on annual additions to Ryan’s account in 2021 is the lesser of 100% of compensation ($80,000) or $58,000.

LO 1.3.2

How well did you know this?
1
Not at all
2
3
4
5
Perfectly
14
Q

Which of the following statements regarding a top-heavy plan is CORRECT?

I. A top-heavy plan is one that provides more than 40% of its aggregate accrued benefits or account balances to key employees.

II. A top-heavy defined pension benefit plan must provide a minimum benefit accrual of 2% multiplied by the number of years of service (up to 20%).

III. For a top-heavy defined contribution plan, the employer must make a minimal contribution of 3% of annual covered compensation for each eligible non-key employee. If the contribution percentage for key employees is less than 3%, the contribution percentage to non-key employees can be equal to the key employees’ percentage.

IV. A top-heavy defined benefit pension plan must provide accelerated vesting.

A) I, II, and IV

B) II, III, and IV

C) I, II, III, and IV

D) I, II, and III

A

B) Explanation
Only Statement I is incorrect. A top-heavy plan is one that provides more than 60% of its aggregate accrued benefits or account balances to key employees.

LO 1.3.1

How well did you know this?
1
Not at all
2
3
4
5
Perfectly
15
Q

Velvet Lawns, Inc., employs 26 full-time workers and provides a money purchase pension plan for eligible employees. All 26 employees are plan participants this year. Jack, the owner of the company, has an account balance of $134,000. The total of the account balances of all plan participants amounts to $215,000.

Which of the following statements apply to this plan?

I. The plan would not pass the required coverage test and is therefore discriminatory.

II. The plan is top heavy.

III. ‘The plan may use either a five-year cliff or three- to seven-year graded vesting schedule.

IV. The plan must comply with requirements for minimum contributions to non-key employees.

A) I and III

B) II, III, and IV

C) I, II, and IV

D) II and IV

A

D) Explanation
If all 26 employees are participating in the plan, the coverage and nondiscrimination tests would be passed. The plan would, however, be top heavy (Jack’s $134,000 account balance is 62% of the $215,000 total of all account balances). Top-heavy plans must comply with minimum contribution requirements for non-key employees and use a top-heavy vesting schedule. All defined contribution plans are now required to vest no less rapidly than three-year cliff or two- to six-year graded.

LO 1.3.1

How well did you know this?
1
Not at all
2
3
4
5
Perfectly
16
Q

Which of the following are agencies that administer and ensure compliance with the federal laws that apply to qualified retirement plans?

I. Department of Labor (DOL)
II. Employee Retirement Income Security Agency (ERISA)
III. Internal Revenue Service (IRS)
IV. Pension Benefit Guaranty Corporation (PBGC)

A) I, II, and III

B) II and IV

C) I, III, and IV

D) I and III

A

C) Explanation
Under the reporting and disclosure requirements of the Employee Retirement Income Security Act of 1974 (ERISA), annual reports and summary plan descriptions are filed with the IRS (which provides copies to the DOL). An annual premium payment form is filed with the PBGC.

LO 1.1.1

How well did you know this?
1
Not at all
2
3
4
5
Perfectly
17
Q

In which situation(s) can the Pension Benefit Guaranty Corporation (PBGC) terminate a defined benefit pension plan?

I. The PBGC does not have the authority to terminate an employer’s defined benefit pension plan.

II. Joe’s Shoes Inc. has not met the minimum funding standards for the corporation’s defined benefit pension plan.

III. Ann has retired from her position and her former employer’s defined benefit pension plan cannot pay her retirement benefit because it lacks the funds.

IV. A professional service corporation with 15 active participants cannot meet the minimum funding standards of the defined benefit pension plan and has previously paid no PBGC premiums.

A) II, III, and IV

B) I only

C) II and III

D) III only

A

C) Explanation
Statements II and III are correct. The PBGC can terminate a defined benefit plan if:

minimum funding standards are not met;
benefits cannot be paid when due; and
the long-run liability of the company to the PBGC is expected to increase unreasonably.
A professional service corporation with 25 or fewer participants is not required to maintain PBGC coverage.

LO 1.1.1

How well did you know this?
1
Not at all
2
3
4
5
Perfectly
18
Q

Which of the following plans allow the excess method of permitted disparity?

I Money purchase pension plans
II. Defined benefit pension plans
III. Employee stock ownership plan (ESOP)
IV. Simplified employee pension (SEP)

A) I, II, and IV

B) II, III, and IV

C) I, III, and IV

D) I, II, and III

A

A) Explanation
All plans are allowed to integrate with Social Security except ESOPs, SIMPLEs, and SARSEPs. The excess method is allowed for all plans allowing integration, whereas the offset method is only allowed for defined benefit pension plans.

LO 1.3.3

How well did you know this?
1
Not at all
2
3
4
5
Perfectly
19
Q

Which of the following are included in the annual additions limit for defined contribution plans?

I. Investment gain
II. Employee elective deferral contributions
III. Employer contributions
IV. Forfeitures reallocated to the remaining participants in the plan

A) I, II, III, and IV

B) I and IV

C) II, III, and IV

D) II and III

A

C) Explanation
Only Statement I is incorrect. Investment gain is not considered in the calculation of annual additions.

LO 1.3.2

How well did you know this?
1
Not at all
2
3
4
5
Perfectly
20
Q

Which of the following are included in the annual additions limit for defined contribution plans?

I. Investment gain
II. Employee elective deferral contributions
III. Employer contributions
IV. Forfeitures reallocated to the remaining participants in the plan

A) I, II, III, and IV

B) I and IV

C) II, III, and IV

D) II and III

A

C) Explanation
Only Statement I is incorrect. Investment gain is not considered in the calculation of annual additions.

LO 1.3.2

How well did you know this?
1
Not at all
2
3
4
5
Perfectly
21
Q

ERISA requires reporting and disclosure of plan information to all of the following except

A) the Department of Labor (DOL).
B) plan participants.
C) plan sponsors.
D) the Internal Revenue Service (IRS).

A

C) Explanation
ERISA requires reporting and disclosure of plan information by plan sponsors to the IRS, DOL, Pension Benefit Guaranty Corporation (PBGC), and plan participants.

LO 1.1.1

How well did you know this?
1
Not at all
2
3
4
5
Perfectly
22
Q

Scott is the fiduciary of the BSB retirement plan. The entity responsible for monitoring his actions as a fiduciary is

A) the PBGC.
B) the DOL.
C) the SPD.
D) the ERISA.

A

B) Explanation
The Department of Labor (DOL) governs the actions of plan fiduciaries and ensures compliance with the ERISA plan reporting and disclosure requirements.

LO 1.1.1

How well did you know this?
1
Not at all
2
3
4
5
Perfectly
23
Q

Max is the finance director for Bland Foods, Inc. He is trying to implement a new qualified retirement plan for the company. There are numerous federal guidelines with which the company must comply. Which of the following federal agencies is tasked with supervising the creation of new, qualified retirement plans?

A) DOL
B) IRS
C) ERISA
D) PBGC

A

B) Explanation
The Internal Revenue Service (IRS) carries out the task of supervising the creation of new, qualified retirement plans.

LO 1.1.1

How well did you know this?
1
Not at all
2
3
4
5
Perfectly
24
Q

ERISA requirements for qualified plans include

A) participation and fiduciary requirements.
B) reporting and disclosure.
C) coverage and vesting.
D) all of these.

A

D) Explanation
All of these are ERISA requirements for qualified plans.

LO 1.1.1

How well did you know this?
1
Not at all
2
3
4
5
Perfectly
25
Q

Qualified retirement plans should do which of the following?

I. They must meet specific vesting requirements.
II. They have special tax advantages over nonqualified plans.
III. They must provide definitely determinable benefits.
IV. They require an annual profit to allow funding for the plan.

A) II and III
B) I, II, and III
C) I and IV
D) II, III, and IV

A

B) Explanation
Qualified plans must meet specific vesting schedules. Qualified plans are preferred to nonqualified plans because of the special tax advantages enjoyed by qualified plans. Qualified retirement plans must offer definitely determinable benefits. An annual profit is not required for a qualified plan to be funded.

LO 1.2.1

How well did you know this?
1
Not at all
2
3
4
5
Perfectly
26
Q

Qualified retirement plans are which of these?

I. They are subject to ERISA requirements.
II. They offer tax-deferred earnings to employees.
III. They can discriminate in favor of highly compensated employees.
IV. They provide a deferred tax deduction for employer funding.

A) I and II
B) I, II, and III
C) I, II, III, and IV
D) III and IV

A

A) Explanation
Statements I and II are correct. Qualified retirement plans are subject to ERISA requirements and provide tax deferral on investment earnings for employees. While qualified plans in general can provide different levels of benefits to different classes of employees, qualified plans cannot “discriminate in favor of highly compensated employees” in the sense that there is a legal limit to the amount of the difference. As long as the difference is inside the legal limits, the plan is not discriminatory (by definition). Qualified retirement plans provide an immediate tax deduction on employer contributions—not a deferred tax deduction, like a nonqualified deferred compensation plan.

LO 1.2.1

How well did you know this?
1
Not at all
2
3
4
5
Perfectly
27
Q

Which of the following describe differences between a tax-advantaged retirement plan and a qualified plan?

I. IRA-funded employer-sponsored tax-advantaged plans may not incorporate loan provisions.

II. Employer stock distributions from a tax-advantaged plan do not benefit from NUA tax treatment.

A) II only
B) Neither I nor II
C) I only
D) Both I and II

A

D) Explanation
Both statements are correct. IRA-funded employer-sponsored tax-advantaged plans are SEPs, SARSEPs, and SIMPLE IRAs.

LO 1.2.1

How well did you know this?
1
Not at all
2
3
4
5
Perfectly
28
Q

Jerry wants to establish a qualified plan for his business to provide employees of the company with the ability to save for retirement. Which of the following plans is a qualified plan?

I. Profit-sharing plan
II. Simplified employee pension (SEP) plan
III. SIMPLE IRA
IV. Section 457 plan

A) I, II, III, and IV
B) IV only
C) I only
D) II and III

A

C) Explanation
Of the plans listed, only the profit-sharing plan is a qualified plan. The SIMPLE IRA and the SEP plan are tax-advantaged plans, and the Section 457 plan is a nonqualified plan.

LO 1.2.1

How well did you know this?
1
Not at all
2
3
4
5
Perfectly
29
Q

Which of the following is an example of a qualified retirement plan?

A) Section 403(b) plan
B) Deferred compensation plan
C) Section 401(k) plan
D) SEP plan

A

C) Explanation
A Section 401(k) plan is a qualified plan. 403(b) plans and SEP plans are tax-advantaged plans, but are not ERISA-qualified retirement plans. A deferred compensation plan is a nonqualified plan. While tax-advantaged plans are very similar to qualified plans, there are some minor differences.

LO 1.2.1

How well did you know this?
1
Not at all
2
3
4
5
Perfectly
30
Q

Which of the following is NOT an example of a qualified retirement plan?

A) New comparability plan
B) Section 403(b) plan
C) Employee stock ownership plan (ESOP)
D) Section 401(k) plan

A

B) Explanation
A Section 403(b) plan is a tax-advantaged plan but not an ERISA-qualified retirement plan. While tax-advantaged plans are very similar to qualified plans, there are some minor differences. For example, a tax-advantaged plan is not allowed to have NUA treatment. NUA is covered later in the course. They are also not allowed to offer 10-year forward averaging or special pre-1974 capital gains treatment. Tax-advantaged plans also have less restrictive nondiscrimination rules. Otherwise, they are very similar to qualified plans.

LO 1.2.1

How well did you know this?
1
Not at all
2
3
4
5
Perfectly
31
Q

Which of these statements regarding top-heavy plans is CORRECT?

I. An accelerated vesting schedule is used when a defined benefit pension plan is top heavy.

II. A qualified plan is considered top heavy if it provides more than 50% of its aggregate accrued benefits or account balances to key employees.

III. Top-heavy defined benefit plans must provide a minimum benefit accrual of 2% per year of service for up to 10 years (20%) for all non-key employees.

IV. For a top-heavy plan, a key employee is an employee who owns more than 3% of the employer with compensation greater than $130,000 (2021).

A) I and II
B) II and III
C) II and IV
D) I and III

A

D) Explanation
An accelerated vesting schedule is used when a defined pension benefit plan is top heavy. A defined contribution plan always requires an accelerated vesting schedule. A qualified plan is considered top heavy if it provides more than 60% of its aggregate accrued benefits or account balances to key employees. A key employee is an employee who, at any time during the plan year, is the following: greater than a 5% owner; a greater than 1% owner with compensation greater than $150,000 (not indexed); or an officer of the employer with compensation greater than $185,000 in 2021.

LO 1.3.1

How well did you know this?
1
Not at all
2
3
4
5
Perfectly
32
Q

Which of the following are minimum coverage tests for qualified retirement plans?

I. Nondiscrimination test
II. Average benefits percentage test
III. Ratio test
IV. Maximum compensation test

A) II, III, and IV
B) I, II, and III
C) I and II
D) II and III

A

D) Explanation
The two minimum coverage tests for qualified retirement plans are the average benefits percentage test and the ratio test. To be qualified, a retirement plan must meet at least one of these tests if the plan does not meet the percentage (safe harbor) test.

LO 1.3.1

How well did you know this?
1
Not at all
2
3
4
5
Perfectly
33
Q

Nigel’s employer, Alpha, Inc., maintains a qualified defined benefit pension plan. There are 100 eligible employees working for Alpha, Inc. What is the minimum number of employees the retirement plan must cover to satisfy the 50/40 test?

A) 100
B) 80
C) 40
D) 50

A

C) Explanation
Under the 50/40 test, a defined benefit plan must cover the lesser of 50 employees or 40% of all eligible employees. In this case, the lesser of 50 employees or 40% of all eligible employees (100) is 40 employees. One way to remember the 50/40 test is the phrase people before percentages (50 people or 40%). Also, note that there are no qualifiers to the types of people. It is not 50 non-highly compensated people. It is just 50 individuals who work for the employer.

LO 1.3.1

How well did you know this?
1
Not at all
2
3
4
5
Perfectly
34
Q

Brad has been the sole owner and operator of Woodmasters, Inc., for the past 15 years. Brad is age 45, and his salary from the business is $130,000. Brad and his spouse, Laura, want to retire when Brad is age 65. Relevant information regarding the business is summarized as follows:

Financial performance fluctuated over the first 10 years.
Cash flow and profits have stabilized during the past five years and are expected to show modest but consistent growth in the future. Excess cash flow of approximately $150,000 is expected to be available this year. Future years should be about the same. Brad has expressed some concern about the company’s outdated equipment and is considering renovating the plant and replacing the outdated equipment over the next five years. The total cost should be about $300,000.
Total compensation for all employees (including Brad) is $245,000.
The four full-time rank-and-file employees range from age 19 to age 38, and have been with Woodmasters for periods ranging from four months to six years. Age and service information is shown as follows:
Employee Age Completed Years of Service Compensation
Brad 45 15 years $130,000
Beth 38 6 years $40,000
Todd 27 6 months $25,000
Carol 30 2 years $28,000
Jim 19 4 months $22,000
Which of the following statements can be made regarding any qualified plan that Brad installs at Woodmasters? (Assume that the plan admits any employee who is eligible under the ERISA age and one-year service requirements and that all eligible employees are participating.)

The plan passes the ratio percentage test.
Three employees (in addition to Brad) will be eligible to participate.
The plan passes the participation (50/40) test, although it may not be required to do so.
Information is insufficient to perform coverage and participation tests.
A) II and III
B) I and III
C) I, II, and III
D) II and IV

A

B) Explanation
Since the plan admits all ERISA-eligible employees, relevant percentages for the coverage and participation tests will be 100%; the plan will pass the ratio percentage test. Since the lesser of 50 participants or 40% of the ERISA-eligible employees is two and because three (100%) actually participate, the plan passes the 50/40 test. The 50/40 test only applies to defined benefit plans.

LO 1.3.1

How well did you know this?
1
Not at all
2
3
4
5
Perfectly
35
Q

Which of the following statements regarding a top-heavy plan is CORRECT?

I. A top-heavy plan is one that provides more than 50% of its aggregate accrued benefits or account balances to key employees.

II. A top-heavy defined pension benefit plan must provide a minimum benefit accrual of 2% multiplied by the number of years of service (up to 20%).

III. For a top-heavy defined contribution plan, the employer must make a minimal contribution of 3% of annual covered compensation for each eligible non-key employee. If the contribution percentage for key employees is less than 3%, the contribution percentage to non-key employees can be equal to the key employees’ percentage.

IV. A top-heavy defined benefit pension plan must provide accelerated vesting.

A) I, II, III, and IV
B) II and IV
C) II, III, and IV
D) I and III

A

C) Explanation
Only Statement I is incorrect. A top-heavy plan is one that provides more than 60% of its aggregate accrued benefits or account balances to key employees.

LO 1.3.1

How well did you know this?
1
Not at all
2
3
4
5
Perfectly
36
Q

The Jones Corporation has a profit-sharing plan with a 401(k) provision. The company matches dollar-for-dollar up to 5%. Pedro makes $150,000 and defers 5% into the 401(k) for 2021. The Jones Corporation has had a banner year and is considering a large contribution to the profit-sharing plan. What is the most that could be contributed to Pedro’s profit-sharing account this year?

A) $58,000
B) $43,000
C) $49,500
D) $19,500

A

B) Explanation
The maximum allowed contribution for 2021 is $43,000. The section annual additions limit for 2021 is $58,000. However, Pedro has already contributed $7,500, and this amount has been matched. Thus, $15,000 has already gone toward the $58,000 annual additions limit for 2021.

LO 1.3.2

How well did you know this?
1
Not at all
2
3
4
5
Perfectly
37
Q

Susan makes $400,000 working for Great Grapes, Inc. She defers 4% into the 401(k) and receives the 4% match. How much will go into her account in 2021?

A) $16,000
B) $32,000
C) $23,200
D) $19,500

A

Explanation
Only the first $290,000 of compensation may be used to determine contributions to qualified retirement plans in 2021. Thus, she contributes 4% of $290,000 in 2021. This amount is matched, so $290,000 × 0.08 = $23,200.

LO 1.3.2

How well did you know this?
1
Not at all
2
3
4
5
Perfectly
38
Q

In a Section 401(k) plan, which of these must be considered in complying with the maximum annual additions limit?

I. Employee contributions
II. Catch-up contributions for an employee age 50 or older
III. Dividends paid on employer stock held in the Section 401(k) plan
IV. Employer contributions

A) I and II
B) I and IV
C) II and IV
D) I, II, and V
Explanation
A

B) Statement I is correct. Employee contributions are counted against the annual additions limit. Statement II is incorrect. Catch-up contributions for an employee age 50 or older are not counted against the annual additions limit. Statement III is incorrect. Earnings on plan investments are not taken into account when computing the maximum annual additions limit. Statement IV is correct. For 2021, the annual additions limit is the lesser of 100% of the employee’s compensation, or $58,000.

LO 1.3.2

How well did you know this?
1
Not at all
2
3
4
5
Perfectly
39
Q

If a defined benefit pension plan is determined to be top heavy, what is one practical significance of this determination?

A) Different eligibility requirements come into effect.
B) One of two maximum contribution and benefit formulas must be used.
C) Different coverage requirements and nondiscrimination tests apply.
D) One of two accelerated vesting schedules must be used.

A

D) Explanation
If a defined benefit pension plan is top heavy, one of two accelerated vesting schedules must be used: either 100% vesting after three years of service or graded two- to six-year vesting. The same eligibility and nondiscrimination tests apply as for qualified plans that are not top heavy.

LO 1.3.1

How well did you know this?
1
Not at all
2
3
4
5
Perfectly
40
Q

Mac, age 39, works for the SLH Company. He has a salary of $28,000 and a 401(k) there. Mac also has another job with AKH, Inc., making $65,000. SLH and AKH are not related. Mac is deferring $9,500 into the 401(k) at SLH. How much can he defer into the 401(k) at AKH?

A) $58,000
B) $47,500
C) $10,000
D) $19,500

A

C) Explanation
Mac can defer $10,000 into his 401(k) at AKH, Inc. The total he can contribute to employer retirement plans in 2021 is $19,500. Since he is already doing $9,500 at one employer, he is limited to $10,000 at the other. The fact that the two employers are unrelated does not matter for how much a worker can defer into the two plans. However, when it comes to unrelated organizations, each would have an independent annual additions limit. In other words, the law expects workers to know how much they are contributing to various employer retirement plans and to stay below the limit. The same would be true for related employers. However, unrelated employers are not held responsible for the employee or employer contributions to the other employers.

LO 1.3.2

How well did you know this?
1
Not at all
2
3
4
5
Perfectly
41
Q

Window Washers, Inc., is establishing a profit-sharing plan using Social Security integration. The base contribution percentage for the profit-sharing plan will be 5%, and the owners have come to you with some questions about Social Security integration. Which one of the following statements is CORRECT?

A) The permitted disparity for the plan is 3%.
B) The permitted disparity for the plan is 5.7%.
C) The excess contribution percentage for the plan could be as high as 10%.
D) The excess contribution percentage for the plan could be as high as 26.25%.

A

C) Explanation
The excess contribution percentage for the plan could be as high as 10%. The excess contribution percentage is the base contribution percentage plus the permitted disparity. The permitted disparity for the defined contribution plan is the lesser of the base benefit percentage and 5.7%. Thus, in this case, the permitted disparity is 5% and the maximum the excess benefit percentage could be would be 10%.

LO 1.3.3

How well did you know this?
1
Not at all
2
3
4
5
Perfectly
42
Q

Jim, the president of East Dover Construction Company, has requested your advice in setting up a defined benefit pension plan for eligible employees in the company. Jim founded the company 17 years ago and now has 200 employees, most of whom are under 35 years of age. Due to the nature of the work and ongoing management difficulties, tenure among the employees has averaged under two years. Jim has just fired the managers who were creating problems, but turnover is likely to remain high due to ongoing morale problems. Jim’s current salary is $300,000, and he wants the plan to provide him with annual retirement income of $100,000 per year. He expects to retire in 13 years, at age 64.

Which of the following statements describes information you need to convey to Jim about factors that could affect the amount of his retirement benefit?

A) If the plan investments outperform expectations, he will receive the largest allocation of the excess earnings.

B) One method of increasing the retirement benefit paid to him is by integrating the plan using the offset method.

C) Using a flat benefit formula will always provide Jim a larger benefit than would a unit benefit formula.

D) The excess integration method could be used to increase the amount of his plan benefit in retirement.

A

D) Explanation
The excess integration method increases the retirement benefit for compensation above the integration level. The offset integration method reduces the retirement benefit due to the receipt of Social Security benefits. With a defined benefit plan, excess investment performance reduces the subsequent employer contributions. A flat benefit formula versus a unit benefit formula would depend on the details of the flat benefit versus the unit benefit formula, so more information would be required to determine which would give a larger benefit.

LO 1.3.3

How well did you know this?
1
Not at all
2
3
4
5
Perfectly
43
Q

Able Company is considering various types of qualified plans and seeks your advice. You are asked how a plan participant’s benefits at retirement are determined in a defined benefit plan with a flat benefit formula that uses the offset method of integration.

Which of these statements would best answer the company’s question?

A) The percentage of pay benefit specified by the plan is reduced by a specific percentage of the retired employee’s Social Security benefit.

B) The benefit paid from the employer plan is based only on employee compensation above the Social Security wage base.

C) The employee’s Social Security benefit is reduced by a specific percentage of the retired employee’s retirement plan benefit.

D) The benefit paid from the employer plan is reduced by the amount the employer pays into Social Security on behalf of the employee.

A

A) Explanation
The percentage of pay benefit specified by the plan is reduced by a specific percentage of the retired employee’s Social Security benefit. Offset integration reduces the defined benefit received due to the retiree also getting Social Security benefits.

LO 1.3.3

How well did you know this?
1
Not at all
2
3
4
5
Perfectly
44
Q

What is the permitted disparity for a defined contribution plan with a current base contribution percentage of 6%?

A) 12.0%
B) 11.7%
C) 5.7%
D) 6.0%

A

C) Explanation
The permitted disparity is the extra amount reached above the integration level. For an integrated defined contribution plan, the permitted disparity is the lesser of 5.7% and the base contribution percentage. Since the base benefit percentage is 6%, the permitted disparity is limited to 5.7%. That would mean the excess benefit percentage is 11.7% (6% + 5.7%).

LO 1.3.3

How well did you know this?
1
Not at all
2
3
4
5
Perfectly
45
Q

With an integrated defined contribution plan, what is the maximum permitted disparity?

A) The maximum permitted disparity is 25%, so if the base benefit percentage is greater than 25% then the permitted disparity would be capped at 25%.

B) In an integrated defined contribution plan, if the base contribution percentage is 5% then the permitted disparity is 5.7%.

C) The permitted disparity is 0.75% per year for up to 35 years.

D) For an integrated defined contribution plan, the permitted disparity is the lower of the base amount, or 5.7%.

A

D) Explanation
The permitted disparity is the lower of the base amount, or 5.7%. Thus, the maximum permitted disparity is 5.7% for integrated defined contribution plans. The number 5.7% is the percentage of an employee’s compensation that goes toward his Social Security retirement benefit for compensation below the taxable wage base ($142,800 in 2021).

LO 1.3.3

How well did you know this?
1
Not at all
2
3
4
5
Perfectly
46
Q

Which of these statements regarding prohibited transactions is false?

A) One category of prohibited transactions involves the sale, lease, or exchange of any property between the plan and a party in interest.

B) One category of prohibited transactions involves the investment in the sponsoring employer’s stock or real property.

C) One category of prohibited transactions bars a fiduciary from causing the plan to engage in a transaction if the fiduciary knows or should know that such a transaction constitutes a direct or indirect involvement between the plan and the parties in interest.

D) The lending of money or other extension of credit between the plan and a party in interest is a prohibited transaction exemption.

A

D) Explanation
Loans between the plan and a party in interest are prohibited transactions.

LO 1.4.1

How well did you know this?
1
Not at all
2
3
4
5
Perfectly
47
Q

Which of the following penalties apply to prohibited transactions?

I. A tax equal to 15% of the amount involved applies unless it can be demonstrated that the transaction satisfies ERISA’s fiduciary standards.

II. The transaction must be corrected and the plan placed in a financial position no worse than if the transaction had never occurred.

III. Plan participants who engage in prohibited transactions are subject to income tax on a judicially determined amount.

IV. Transactions that continue uncorrected into subsequent years are subject to additional penalties.

A) I and II
B) I, II, and IV
C) I and III
D) II and IV

A

D) Explanation
The law requires correction (i.e., undoing) of a prohibited transaction and restoring a plan to the position it would have been in, had the transaction never occurred. Ongoing transactions (e.g., loans, leases) create additional prohibited transactions in subsequent years (and additional penalties) until corrected.

Options I and III are incorrect for the following reasons. Once the prohibited transaction has taken place, the 15% penalty cannot be waived for extenuating circumstances. Income tax consequences may or may not apply depending on the nature of the underlying prohibited transaction. Usually, the IRS (not the courts) determines the amount of tax involved.

LO 1.4.1

How well did you know this?
1
Not at all
2
3
4
5
Perfectly
48
Q

Prohibited transactions are those that are not in the best interest of plan participants and include which of these?

I. A loan between the plan and any party in interest
The acquisition of employer securities or real property in excess of legal limits

II. A transfer of plan assets to or use of plan assets for the benefit of a party in interest

III. The sale, exchange, or lease of any property between the plan and a party in interest

A) I and II
B) I, II, and III
C) I, II, III, and IV
D) III and IV

A

C) Explanation
All of the statements are prohibited transactions. Self-dealing is also a prohibited transaction.

LO 1.4.1

How well did you know this?
1
Not at all
2
3
4
5
Perfectly
49
Q

Which of the following is the easiest type of retirement plan for an employer to adopt?

A) An individually designed plan
B) A Pension Benefit Guaranty Corporation (PBGC) plan
C) A prototype plan
D) A custom plan

A

C) Explanation
Master plans and prototype plans are easier to use than individually designed plans or custom plans because they are standardized plans approved as qualified in concept by the IRS. The PBGC is the governmental body that insures pension benefits; it is not a type of plan.

LO 1.4.1

How well did you know this?
1
Not at all
2
3
4
5
Perfectly
50
Q

In the administration of a qualified retirement plan, which of the following individuals is considered to be a fiduciary?

A) The marketing director of the plan sponsor
B) A financial planner handling the investment of plan assets
C) A CPA who prepares the plan’s Form 5500
D) A highly compensated employee who participates in the plan

A

B) Explanation
A person or corporation is considered a fiduciary under ERISA if that person or entity renders investment advice or services to the plan for direct or indirect compensation. Clearly, the financial planner-investment manager is within this definition.

LO 1.4.1

How well did you know this?
1
Not at all
2
3
4
5
Perfectly
51
Q

Which of the following is a characteristic of a target benefit pension plan?

A) Allows higher contribution levels for older plan participants

B) Allows the employee to select the amount of monthly benefit at retirement

C) Requires greater employee contributions for older employees

D) Has no limit on employee contributions

A

A) Explanation
Under such a defined contribution plan, the employer selects the target benefit. The employer contributes more for older employees. The fixed contribution formula is based on an initial actuarial determination of contributions required to meet a specific targeted benefit level.

LO 2.3.2

How well did you know this?
1
Not at all
2
3
4
5
Perfectly
52
Q

Which of these statements regarding target benefit pension plans are CORRECT?

I. The plans are covered by Pension Benefit Guaranty Corporation (PBGC) insurance.

II. Older participants who are new to the employer are favored in a target benefit pension plan.

III. Each employee has an individual account.

IV. Minimum funding standards apply.

A) I, II, III, and IV

B) I and III

C) II, III, and IV

D) II and IV

A

C) Explanation
Statements II, III, and IV are correct. Target benefit pension plans are a type of defined contribution plans and are not covered by PBGC insurance. Each plan participant has an individual account. Because target benefit plans are pension plans, minimum funding standards apply. Like defined benefit plans, the plans favor older participants in the sense that a new employee who is older than another employee and has the same compensation as the younger employee will get a larger target benefit contribution than the younger employee. This is true because both workers have the same target benefit, but the younger worker has more time for the balance to grow than an older worker.

LO 2.3.2

How well did you know this?
1
Not at all
2
3
4
5
Perfectly
53
Q

In a money purchase pension plan that utilizes plan forfeitures to reduce future employer plan contributions, which of the following components must be factored into the calculation of the maximum annual addition limit?

(CFP® Certification Examination, released 11/94)

I. Forfeitures that otherwise would have been reallocated

II. Annual earnings on all employer and employee contributions

III. Rollover contributions for the year

IV. Employer and employee contributions to all defined contribution plans

A) IV only

B) I, II, and III

C) I, II, III, and IV

D) I and III

A

A) Explanation
Because the forfeitures will be used to reduce future employer contributions, they will not count against the annual additions limit. Defined contribution plans do not consider earnings on investments when calculating contributions.

LO 2.3.1

How well did you know this?
1
Not at all
2
3
4
5
Perfectly
54
Q

Target benefit pension plans are defined contribution plans that do which of these?

I. They fund for a targeted benefit level.

II. They require a fixed contribution formula.

III. They provide a guaranteed retirement benefit.

IV. They adjust the contribution formula annually.

A) I and II

B) III and IV

C) II, III, and IV

D) I and III

A

A) Explanation
Statements I and II are correct. Target benefit pension plans have a fixed contribution formula that is based on an actuarial calculation and the participant’s age at plan inception or plan entrance. The plan allows higher contributions for older participants. Although the plan is technically a defined contribution plan, its intention is to fund for a targeted benefit at retirement, based on the participant’s age and number of years to retirement. The actual contributions may or may not achieve the targeted benefit; therefore, the retirement benefit is not guaranteed. The plan sponsor does not actuarially adjust the participant’s contribution percentage annually.

LO 2.3.2

How well did you know this?
1
Not at all
2
3
4
5
Perfectly
55
Q

Which of the following factors affect a target benefit plan participant’s retirement benefits?

I. The actuarial assumptions used to determine the contribution to the plan
II. The participant’s compensation for the plan year
III. The investment performance of the plan’s assets
IV. The age of the plan participant

A) I, II, III, and IV

B) II, III, and IV

C) I and III

D) I and II

A

A) Explanation
Options I, II, III, and IV are all correct. Actuarial assumptions about longevity and interest rates affect the amount contributed to a participant’s account. A participant’s compensation directly affects the size of her plan benefit. The value of a participant’s target benefit account balance (i.e., a separate account) depends, in part, on the investment performance (gains and losses) of the plan’s assets. Target benefit plans favor older employees who are closer to retirement.

LO 2.3.2

How well did you know this?
1
Not at all
2
3
4
5
Perfectly
56
Q

Which of the following is a basic provision of a money purchase pension plan?

A) Before-tax salary reductions or elective deferrals are subject to prescribed limitations on amounts.

B) Forfeitures from a nonvested participant’s account must be reallocated proportionately among remaining plan participants.

C) Forfeitures from a nonvested participant’s account must be applied to reduce the employer contribution to the plan.

D) In establishing such a plan, the employer typically agrees to make an annual contribution for each eligible employee as a fixed percentage of compensation.

A

D) Explanation
In establishing a money purchase pension plan, the employer agrees to make a fixed annual contribution, usually expressed as a percentage of compensation for each eligible employee.

LO 2.3.1

How well did you know this?
1
Not at all
2
3
4
5
Perfectly
57
Q

Which of the following statements regarding money purchase pension plans is CORRECT?

A) These plans typically favor younger employees.

B) They allow a maximum of 25% of employer stock.

C) The plan requires actuarial services.

D) Annual employer funding is optional.

A

A) Explanation
Money purchase plans favor younger employees. Money purchase pension plans have known funding costs, mandatory annual contributions, and limit the amount of company stock to a maximum of 10%.

LO 2.3.1

How well did you know this?
1
Not at all
2
3
4
5
Perfectly
58
Q

Which of the following are eligibility requirements that a defined benefit plan must satisfy to qualify for tax-favored status?

I. The plan must include employees who have attained 21 years of age.

II. The plan must include employees who have given one year of service to the employer during which they have worked a minimum of 1,000 hours.

III. If the employer uses the two-year 100% rule, participation requirements may be based on completion of two years of service.

IV. An employee’s service with a predecessor must count toward years of service in a successor’s plan.

A) I, II, and III

B) I, III, and IV

C) I, II, III, and IV

D) I, II, and IV

A

C) Explanation
All of these requirements must be met if the defined benefit plan is to qualify for tax-favored status.

LO 2.1.1

How well did you know this?
1
Not at all
2
3
4
5
Perfectly
59
Q

Which of the following statements regarding money purchase pension plans are CORRECT?

I. The employer makes annual mandatory contributions to each employee’s individual account.

II. The plan is relatively straightforward and easy to explain to participants.

III. Annual additions to each employee’s account are limited to the lesser of 100% of compensation, or $58,000 (for 2021).

IV. Generally, employer securities held by the plan cannot exceed 25% of the FMV of the plan assets at the times the securities are purchased.

A) I, II, III, and IV

B) I, II, and III

C) III and IV

D) I and II

A

B) Explanation
Only Statement IV is incorrect. Generally, employer securities held by the plan cannot exceed 10% of the FMV of the plan assets at the time the securities are purchased.

LO 2.3.1

How well did you know this?
1
Not at all
2
3
4
5
Perfectly
60
Q

Which of the following statements regarding fully insured Section 412(e)(3) plans is FALSE?

A) Section 412(e)(3) plans must meet minimum funding standards each plan year.

B) This type of plan is not required to be certified by an enrolled or licensed actuary.

C) A fully insured plan is inappropriate for an employer who cannot commit to regular premium payments.

D) A Section 412(e)(3) plan is a type of defined benefit pension plan.

A
A) Explanation
Section 412(e)(3) plans must only meet minimum funding standards if there is a loan outstanding against the insurance policy funding the plan.

LO 2.2.2

How well did you know this?
1
Not at all
2
3
4
5
Perfectly
61
Q

All of the following are characteristics of traditional defined benefit pension plans except

A) the employer is required to make annual contributions.

B) employees assume the risk of poor investment results.

C) they are complex to design and operate.

D) limited benefits are guaranteed by the Pension Benefit Guaranty Corporation (PBGC).

A

B) Explanation
A traditional defined benefit pension plan guarantees a specific benefit at retirement, and limited benefits are guaranteed by the PBGC. The employer assumes the risk of poor investment results in the plan. If investment results are poor, the employer may have to supply additional funding to assure that the guaranteed benefits are available.

LO 2.1.1

How well did you know this?
1
Not at all
2
3
4
5
Perfectly
62
Q

Which of the following statements is CORRECT in describing an accrued benefit?

A) The total benefit the participant has earned to date (amount is usually determined by multiplying the result of the participant’s years of service divided by the total potential years of service times the benefit formula in a flat percentage benefit plan)

B) The benefit that each employee receives when they leave the company

C) The dollar amount set aside in the defined benefit plan for the employee

D) The dollar amount distributed to the employee at termination of employment

A

A) Explanation
An accrued benefit is usually determined by multiplying the result of the participant’s years of service divided by the total potential years of service, times the benefit formula. The dollar amount to be distributed on termination is the “present value of the vested accrued benefit times the assumed annuity factor.”

LO 2.1.2

How well did you know this?
1
Not at all
2
3
4
5
Perfectly
63
Q

Which of the following statements is CORRECT in describing an accrued benefit?

A) The total benefit the participant has earned to date (amount is usually determined by multiplying the result of the participant’s years of service divided by the total potential years of service times the benefit formula in a flat percentage benefit plan)

B) The benefit that each employee receives when they leave the company

C) The dollar amount set aside in the defined benefit plan for the employee

D) The dollar amount distributed to the employee at termination of employment

A

A) Explanation
An accrued benefit is usually determined by multiplying the result of the participant’s years of service divided by the total potential years of service, times the benefit formula. The dollar amount to be distributed on termination is the “present value of the vested accrued benefit times the assumed annuity factor.”

LO 2.1.2

How well did you know this?
1
Not at all
2
3
4
5
Perfectly
64
Q

Which of the following statements regarding employer contributions to money purchase pension plans are CORRECT?

I. Aggregate employer deductible contributions may not exceed 25% of covered compensation.

II. Money purchase pension plans are not subject to a minimum funding standard.

III. Plan investment earnings and losses do not affect employer contributions.

IV. Forfeitures may be reallocated to remaining participants.

A) II and IV

B) I, III, and IV

C) II, III, and IV

D) I, II, and III

LO 2.3.1

A

B) Explanation
Money purchase pension plans require mandatory annual funding. Investment earnings and losses do not affect employer contributions. Most pension plans (like money purchase pension plans) are subject to minimum funding standards.

How well did you know this?
1
Not at all
2
3
4
5
Perfectly
65
Q

Your client, the chief financial officer of a new company, wishes to install a retirement plan in the company in which the pension benefits to employees are guaranteed by the Pension Benefit Guaranty Corporation (PBGC). Identify the plan(s) which must meet this requirement.

I. Profit-sharing plan
II. Money purchase plan
III. Target benefit plan
IV. Defined benefit plan

A) III and IV

B) II and III

C) IV only

D) I and II

A

C) Explanation
Only two defined benefit pension plans (defined benefit and cash balance) have PBGC insurance.

LO 2.1.1

How well did you know this?
1
Not at all
2
3
4
5
Perfectly
66
Q

Joe, 46, has owned his company for 18 years and wishes to retire at age 70. All of Joe’s employees are older than he is and have an average length of service with the company of eight years. Joe would like to adopt a qualified retirement plan that would favor him and reward employees who have rendered long service. Joe has selected a traditional defined benefit pension plan with a unit benefit formula. Which of these statements regarding Joe’s traditional defined benefit pension plan is CORRECT?

I. Increased profitability would increase both Joe’s and his employees’ pension contributions.

II. A unit benefit plan formula allows for higher levels of integration than other defined benefit pension plans.

III. A unit benefit plan formula rewards older employees who were hired in their 50s or 60s.

IV. A traditional defined benefit pension plan will maximize Joe’s benefits and reward long-term employees based on length of service.

A) III and IV

B) IV only

C) I, II, and IV

D) II only

A

B) Explanation
Statement I is incorrect. Contributions to traditional defined benefit pension plans are not dependent on the profitability of a company. Statement II is incorrect because a unit benefit plan formula will not allow higher integration levels. Statement III is incorrect because a flat percentage formula favors workers without much longevity.

LO 2.1.2

How well did you know this?
1
Not at all
2
3
4
5
Perfectly
67
Q

Which of the following types of qualified retirement plans are subject to the minimum funding requirements?

I. Defined benefit pension plans
II. Money purchase pension plans
III. Profit sharing plans
IV. Target benefit plans

A) I and II

B) II, III, and IV

C) I, III, and IV

D) I, II, and IV

A

D)
Explanation
The only qualified plans exempt from the minimum funding standard are profit sharing plans and stock bonus plans. Section 401(k) provisions are only permitted with profit sharing plans and stock bonus plans.

LO 2.3.2

How well did you know this?
1
Not at all
2
3
4
5
Perfectly
68
Q

Your client has a retirement plan with separate participant accounts and 40% of salary as a projected retirement benefit. Which of the following type of plans does your client most likely have?

A) Target benefit pension plan

B) Money purchase pension plan

C) Cash balance pension plan

D) Traditional defined benefit pension plan

A

A) Explanation
This plan is most likely a target benefit pension plan because of the individual account (DC) and projected (targeted) retirement benefit.

LO 2.3.2

How well did you know this?
1
Not at all
2
3
4
5
Perfectly
69
Q

Baxter and Smith is a law firm with a defined benefit pension plan. When would the plan be required to be covered by the Pension Benefit Guaranty Corporation (PBGC)?

A) If the firm employs 26 or more employees

B) If the firm employs 25 or fewer employees

C) A professional service employer is not required to be covered by PBGC

D) If the firm employs 10 or more employees

A

A) Explanation
Defined benefit plans maintained by certain professional service employers with 26 or more employees must be covered by the PBGC.

LO 2.1.1

How well did you know this?
1
Not at all
2
3
4
5
Perfectly
70
Q

Bart’s employer has a defined benefit pension plan. He has worked for the company for 20 years. His highest salaries occurred in the final three years before retirement: $255,000, $260,000, and $290,000.

Without considering the formula used to arrive at his retirement benefit amount, what is Bart’s maximum possible benefit from the defined benefit plan if he retires in 2021?

A) $285,000

B) $266,667

C) $230,000

D) $268,333

A

C) Explanation
Under Section 415 of the Tax Code, there is a limit on the projected annual benefit the plan can provide to the employee-participant at age 65. For 2021, this maximum benefit is the lesser of:

$230,000 of annual compensation; or
100% of the participant’s compensation averaged over his highest three consecutive years of earnings.
Bart’s last three years also contained his highest salaries, and averaged $266,667 [($255,000 + $260,000 + $285,000 maximum) ÷ 3], which is greater than the $230,000 maximum benefit allowed under Section 415. Compensation considered in any qualified plan benefit formula is capped at $290,000 (2021).

LO 2.1.2

How well did you know this?
1
Not at all
2
3
4
5
Perfectly
71
Q

What is the minimum number of employees a defined benefit plan must benefit to conform to IRS and ERISA regulations?

A) 50 employees

B) The lesser of 50 employees, or 40% of all eligible employees

C) The lesser of 50 employees, or 50% of all eligible employees

D) The lesser of 40 employees, or 50% of all eligible employees

A

B) Explanation
A defined benefit plan must meet an additional test to retain its qualified plan status. The 50/40 test states that the plan must benefit the lesser of 50 employees, or 40% of all eligible employees.

LO 2.1.1

How well did you know this?
1
Not at all
2
3
4
5
Perfectly
72
Q

Your client, the chief financial officer of a new company, wishes to install a retirement plan in the company in which the pension benefits to employees are guaranteed by the Pension Benefit Guaranty Corporation (PBGC). Identify the plan(s) that must meet this requirement.

(CFP® Certification Examination, released 12/96)

I. Profit-sharing plan
II. Money purchase plan
III. Target benefit plan
IV. Defined benefit plan

A) I and III
B) III and IV
C) IV only
D) I and II

A

C) Explanation
Only two defined benefit pension plans (defined benefit and cash balance) are covered by PBGC insurance.

LO 2.1.1

How well did you know this?
1
Not at all
2
3
4
5
Perfectly
73
Q

Assume a company’s goal is to maximize retirement benefits to the highly compensated employees, who also happen to be the oldest employees. Which of the following best accomplishes this goal if the company is installing a new plan?

A) An age-weighted profit-sharing plan
B) A money purchase pension plan
C) A defined benefit pension plan
D) A target benefit pension plan

A

C) Explanation
The defined benefit pension plan favors older participants and generally allows larger contributions than other plans. Age-based profit-sharing plans and target benefit pension plans also favor older participants. However, age-based profit-sharing plans and target benefit pension plans are both defined contribution plans that are subject to the annual additions limit (for 2021 a maximum contribution of no more than the lesser of 100% of an employee’s compensation or $58,000).

LO 2.1.1

How well did you know this?
1
Not at all
2
3
4
5
Perfectly
74
Q

Which of the following statements is CORRECT in describing the effects of actuarial methods or assumptions on contributions to a defined benefit pension plan?

A) Using salary scales tends to reduce the cost for funding younger employees’ benefits.

B) The lower the projected investment earnings on plan assets over the life of the plan, the lower the required employer contribution for the year.

C) The higher the turnover rate assumed by the actuary, the lower the required employer contribution for the year.

D) The higher the interest rate assumed by the actuary, the higher the required employer contribution for the year.

A

C) Explanation
Because defined benefit plans must apply forfeitures to reduce the employer contribution, an actuarial assumption of high turnover in a plan year will result in a lower required employer contribution.

LO 2.1.1

How well did you know this?
1
Not at all
2
3
4
5
Perfectly
75
Q

This year, its pension actuary informed Gear-It-Up Bicycles, Inc., that its required annual contribution to the company defined benefit pension plan needs to be significantly higher than last year. Which of the following factors could be responsible for this increase in annual contributions?

A) Lower-than-expected earnings on plan investments

B) Lower-than-expected number of employees nearing normal retirement age

C) Higher-than-expected employee turnover

D) Higher-than-expected forfeitures

LO 2.1.1

A

A) Explanation
Earnings on investment plans, which were lower than anticipated, will result in an increase to the required annual employer contributions to a defined benefit pension plan. The other factors noted may result in a decrease in plan contributions.

How well did you know this?
1
Not at all
2
3
4
5
Perfectly
76
Q

An increase in which of the following factors will increase plan costs for a defined benefit pension plan?

A) Mortality
B) Life expectancies
C) Investment returns
D) Employee turnover

A

Explanation
An increase in employee life expectancies will increase the potential costs of the plan, so life expectancies have a direct impact on plan costs.

LO 2.1.1

How well did you know this?
1
Not at all
2
3
4
5
Perfectly
77
Q

ERISA requires reporting and disclosure of defined benefit plan information to

I. the PBGC.
II. plan participants.
III. the IRS.
IV. the DOL.

A) I, II, and III
B) I, II, III, and IV
C) III and IV
D) I and II

LO 2.1.1

A

B) Explanation

All of these statements are correct.

How well did you know this?
1
Not at all
2
3
4
5
Perfectly
78
Q

Great Benx Corporation provides both a defined benefit plan and a money purchase plan for its employees. The defined benefit plan is covered by the PBGC. All employees participate in each plan. If the Section 415 limits apply, how do they pertain to this plan?

A) The Section 415 limits are applied separately for each plan. The annual additions limit for the money purchase plan in 2021 is 100% of the participant’s compensation or $58,000, whichever is less. The participant’s benefit in the defined benefit plan is limited in 2021 to 100% of the participant’s compensation or $230,000, whichever is less.

B) Each participant could receive a maximum contribution of 25% of the participant’s compensation for the money purchase plan and a maximum contribution of 100% of the participant’s compensation for the defined benefit plan.

C) The Section 415 limits no longer apply. These limits were repealed by the Economic Growth and Tax Relief Reconciliation Act of 2001 (EGTRRA) for qualified plan application.

D) Under the Section 415 limits, the total contributions to both accounts of a participant are limited to 25% of the participant’s compensation.

A

A) Explanation
The annual additions limit for the money purchase plan in 2021 is 100% of the participant’s compensation or $58,000, whichever is less. The participant’s benefit in the defined benefit plan is limited in 2021 to 100% of the participant’s compensation or $230,000, whichever is less. The Section 415 limits are applied separately for each plan.

LO 2.1.2

How well did you know this?
1
Not at all
2
3
4
5
Perfectly
79
Q

Joe, age 46, has owned his company for 18 years and wishes to retire at age 70. All of Joe’s employees are older than he is and have an average length of service with the company of eight years. Joe would like to adopt a qualified retirement plan that would favor him and reward employees who have rendered long service. Joe has selected a traditional defined benefit pension plan with a unit benefit formula. Which of the following statements regarding Joe’s traditional defined benefit pension plan is CORRECT?

I. Increased profitability would increase both Joe’s and his employees’ pension contributions.

II. A unit benefit formula allows for higher levels of integration than other defined benefit plans.

III. A unit benefit formula rewards older employees hired in their 50s or 60s.

IV. A traditional defined benefit pension plan will maximize Joe’s benefits and reward long-term employees based on length of service.

A) I, II, and IV
B) II and III
C) IV only
D) I and III

A

C) Explanation
Statement I is incorrect. Contributions to defined benefit pension plans are not dependent on the profitability of a company. Statement II is incorrect, because a unit benefit formula plan will not allow higher integration levels. Statement III is incorrect because a unit benefit formula favors workers with longevity. Employees hired in their 50s or 60s will not have significant years of service in the formula at retirement.

How well did you know this?
1
Not at all
2
3
4
5
Perfectly
80
Q

Fernando, age 45, participates in his employer’s defined benefit pension plan. This plan provides for a retirement benefit of 2% of earnings for each year of service with the company and, given Fernando’s projected service of 20 years, will provide him with a benefit of 40% of final average pay at age 65. What type of benefit formula is this plan using?

A) A flat benefit formula
B) A discretionary formula
C) A unit benefit formula
D) A flat percentage of earnings formula

A

C) Explanation
Because this formula considers both compensation and years of service, the defined benefit plan is using a unit benefit formula. This type of formula is frequently used to reward continued service with the same employer.

LO 2.1.2

How well did you know this?
1
Not at all
2
3
4
5
Perfectly
81
Q

Three common formulas for determining annual retirement benefits from a defined benefit plan include all of the following except

A) the flat percentage of earnings.
B) the percentage of earnings per year of service (unit benefit).
C) the flat dollar amount.
D) the dollar percentage matrix.

A

D) Explanation
The dollar percentage matrix formula does not exist.

LO 2.1.2

How well did you know this?
1
Not at all
2
3
4
5
Perfectly
82
Q

An employer must comply with restrictions on the use of more than one qualified retirement plan where employees participate in both plans. Which of the following are CORRECT about some of these limitations?

I. If a target benefit plan is a fully insured plan and covered under the PBGC, then the target benefit plan is not taken into account in applying the overall limit on deductions.

II. When an employer has more than one plan, the overall contribution limit will be the lesser of 25% of total participants’ compensations or the amount required to meet the minimum funding standard of the defined benefit plan. This limit applies only to the extent that the employer contribution to the defined contribution plan or plans is greater than 6%.

III. If an employer sponsors both a defined benefit plan and a defined contribution plan (and the defined benefit plan is exempt from the PBGC), the overall employer limit is the amount necessary to meet the minimum funding requirement of the defined benefit plan. A contribution of up to 6% of compensation is also allowed into the defined contribution plan.

IV. Under current law, if a defined benefit plan is covered under the PBGC, then the defined benefit plan is not taken into account in applying the overall limit on employer deductions.

A) II and IV
B) I and III
C) I and II
D) III and IV

A

D) Explanation
One exception to the overall deduction limit is for defined benefit plans that are covered by the PBGC. Under current law, if a defined benefit plan is covered under the PBGC, then the defined benefit plan is not taken into account in applying the overall limit on employer deductions.

LO 2.1.2

How well did you know this?
1
Not at all
2
3
4
5
Perfectly
83
Q

Which of the following is a defined benefit pension plan that promises a benefit based on a hypothetical account balance versus a traditional defined benefit pension plan, which promises a monthly retirement benefit for life?

A) Target benefit pension plan
B) Cross-tested plan
C) Cash balance pension plan
D) Age-weighted plan

A

C) Explanation
The plan described is a cash balance pension plan. Age-weighted plans allocate contributions to participants in such a way that when contributions are converted to equivalent benefit accruals (stated as a percentage of compensation), each participant receives the same rate of benefit accrual. Cross-tested plans are defined contribution plans that test whether the contribution formula discriminates in favor of the highly compensated employee by converting contributions made for each participant into equivalent benefit accruals. Target benefit pension plans are hybrid retirement plans that use a benefit formula like that of a defined benefit pension plan and the individual accounts like a defined contribution plan.

LO 2.2.1

How well did you know this?
1
Not at all
2
3
4
5
Perfectly
84
Q

Which of the following is CORRECT about contributions made to a cash balance pension plan?

A) Neither the employer nor the employee make contributions. Cash balance plans are funded entirely by payroll taxes.

B) Both the employer and the employee make contributions.

C) All contributions are made by the employee.

D) All contributions are made by the employer.

A

D) Explanation
All contributions to the account-based cash balance plan come from the employer and must reflect a uniform allocation formula based on compensation.

LO 2.2.1

How well did you know this?
1
Not at all
2
3
4
5
Perfectly
85
Q

Which of the following statements are disadvantages for the employer-sponsor of a cash balance pension plan?

I. A certain level of plan benefit is guaranteed by the PBGC.

II. The employer bears the investment risk in the plan.

III. Cash balance pension plans are less expensive for the employer than a traditional defined benefit pension plan.

IV. Retirement benefits may be inadequate for older plan entrants.

A) I and II
B) I and III
C) III and IV
D) II and IV

A

D) Explanation
Statements II and IV are disadvantages. Statements I and III are advantages of a cash balance pension plan.

LO 2.2.1

How well did you know this?
1
Not at all
2
3
4
5
Perfectly
86
Q

Tax implications of cash balance pension plans include which of these?

I. Employer contributions on behalf of employees grow tax deferred.
II. Employer contributions to the plan are deductible when made.
III. Annual retirement benefits are limited to $290,000 (2021).
IV. There are no penalties for distributions before age 59½.

A) I and II
B) III and IV
C) I, II, and III
D) I, II, III, and IV

A

A) Explanation
Statements I and II are correct. Section 415(b) limits the benefits provided under the plan to the lesser of $230,000 per year (2021) or 100% of the participant’s highest consecutive three-year average compensation. Distributions from the plan must follow the rules for qualified plan distributions and may be subject to penalty for early distribution.

LO 2.2.1

How well did you know this?
1
Not at all
2
3
4
5
Perfectly
87
Q

Which of the following statements are advantages for the employer-sponsor of a cash balance pension plan?

I. A certain level of plan benefit is guaranteed by the PBGC.

II. The employer bears the investment risk in the plan.

III. Cash balance pension plans are less expensive to administer for the employer than traditional defined benefit pension plans.

IV. Retirement benefits may be inadequate for older plan entrants.

A) I and III
B) I and II
C) III and IV
D) II and IV

A

A) Explanation
Statements I and III are advantages of a cash balance pension plan. Statements II and IV are disadvantages of a cash balance pension plan.

LO 2.2.1

How well did you know this?
1
Not at all
2
3
4
5
Perfectly
88
Q

Which of the following is a type of traditional defined benefit pension plan?

A) A money purchase pension plan
B) An employee stock ownership plan (ESOP)
C) A target benefit pension plan
D) A fully insured Section 412 (e)(3) pension plan

A

D) Explanation
A fully insured Section 412(e)(3) pension plan is a type of traditional defined benefit pension plan. The other types of plans noted are defined contribution plans, including the money purchase pension plan.

LO 2.2.2

How well did you know this?
1
Not at all
2
3
4
5
Perfectly
89
Q

Which of the following statements regarding fully insured Section 412(e)(3) plans is CORRECT?

I. A fully insured Section 412(e)(3) plan is a type of defined benefit plan.

II. All fully insured Section 412(e)(3) plans must meet minimum funding standards each plan year.

III. This type of plan is not required to be certified by an enrolled or licensed actuary.

IV. A fully insured plan is inappropriate for an employer who cannot commit to regular premium payments.

A) I, III, and IV
B) I and III
C) II and III
D) IV only

A

A) Explanation
Statements I, III, and IV are correct. Section 412(e)(3) plans are a type of defined benefit plan. This type of plan is not required to be certified by an enrolled or licensed actuary. A fully insured plan is inappropriate for an employer who cannot commit to regular premium payments. Statement II is incorrect. Fully insured Section 412(e)(3) plans must only meet minimum funding standards if there is a loan outstanding against the insurance policy funding the plan.

LO 2.2.2

How well did you know this?
1
Not at all
2
3
4
5
Perfectly
90
Q

Which of the following apply to money purchase pension plans?

I. The normal retirement benefit must be specified in the plan.

II. The formula is a specified percentage of each employee’s annual compensation.

III. While the amount of final benefit is not guaranteed or promised, there is an employer promise to contribute to the plan each year.

IV. A separate account must be maintained for each participant.

A) II, III, and IV
B) I and II
C) I, II, and III
D) I, II, and IV

A

A) Explanation
Statements II, III, and IV are correct. The first option states a requirement that applies to defined benefit pension plans but not to money purchase plans.

LO 2.3.1

How well did you know this?
1
Not at all
2
3
4
5
Perfectly
91
Q

Which of the following describe money purchase pension plans?

I. The participant’s final account balance determines the actual retirement benefit.

II. The maximum annual contribution is the lesser of 100% of eligible compensation or $58,000 (2021).

III. The employer bears the investment risk.

IV. The plan requires the services of an actuary.

A) I, II, and III
B) III and IV
C) II and IV
D) I and II

A

D) Explanation
Options I and II describe a money purchase pension plan, while options III and IV do not. The employee bears the investment risk and there is no actuary needed for this plan.

LO 2.3.1

How well did you know this?
1
Not at all
2
3
4
5
Perfectly
92
Q

Frank and his daughter, Joan, own and operate a gravel pit. Frank is age 55 and hopes to retire when he reaches age 65. Joan is age 33 and plans to continue operating the business after Frank retires. Currently, Frank owns 70% of the corporation and Joan owns the remaining 30%. Frank’s current income from the corporation is $72,000. They have established a target benefit plan; Frank’s account has been earning 8%, and the balance is currently $67,590. Based upon an analysis of Frank’s cash flow, you and he have determined that he will need annual retirement income of $54,000 in today’s dollars. With this information, which of the following best describes how the target benefit plan will provide for Frank’s and Joan’s retirement?

I. Considering Joan’s age, a money purchase plan would be more beneficial to her.

II. Frank can anticipate that the age-weighted nature of the target benefit plan will provide the bulk of his needed retirement income.

III. Based upon the present investment return, the target benefit plan may provide Frank with less than 40% of the income that he will need at retirement.

IV. For Frank, a target benefit plan will not provide adequate retirement income for him to meet his retirement income objective because of the 25% limit on deductions for employer contributions; additional methods of saving for retirement need to be explored.

A) II only
B) I and III
C) I and II
D) I, III, and IV

A

D) Explanation
Even if the company contribution is 25% of participant compensation, the current account balance and future contributions will not be adequate to meet his retirement income objective. Because of Joan’s age, the annual contributions into her account may be under 6%. Although she has time to accumulate savings, the level of the contribution will provide less retirement fund accumulation for her than a money purchase plan could.

LO 2.3.1

How well did you know this?
1
Not at all
2
3
4
5
Perfectly
93
Q

A money purchase pension plan is a defined contribution plan in which

A) the employer receives no tax deduction for the contribution.

B) the final retirement benefit amount is guaranteed.

C) the employer must contribute 25% of participant compensation each year.

D) the employer typically contributes a fixed percentage of participant compensation each year.

A

D) Explanation
In a money purchase pension plan, the employer typically contributes a fixed percentage of each participant’s compensation. The employer does receive a tax deduction for the amount of contribution to the plan. No guaranteed retirement benefit is provided in a defined contribution plan. The contribution is defined, not the benefit. Only defined benefit pension plans provide guaranteed retirement benefits.

LO 2.3.1

How well did you know this?
1
Not at all
2
3
4
5
Perfectly
94
Q

What is the maximum annual contribution by an employer to a plan participant’s money purchase pension plan account in 2021?

A) Lesser of 100% of employee compensation, or $58,000

B) $19,500

C) The amount required to pay the benefit promised without limit

D) 25% of the employee’s compensation

A

A) Explanation
The maximum annual contribution to the participant’s account under the plan is the lesser of 100% of the eligible employee’s compensation, or $58,000 (for 2021). The deduction for employer contributions to a defined contribution plan is limited to 25% of aggregate covered compensation.

LO 2.3.1

How well did you know this?
1
Not at all
2
3
4
5
Perfectly
95
Q

Ross, age 75, works for Financial Strategies, Inc. The company has a long-established retirement plan. The plan does not require an actuary or Pension Benefit Guaranty Corporation (PBGC) insurance, but the employer is required to make annual mandatory contributions to each employee’s account. What type of retirement plan was established by Financial Strategies?

A) Cash balance pension plan
B) Money purchase pension plan
C) Traditional defined benefit pension plan
D) Target benefit pension plan

A

B) Explanation
A money purchase pension plan requires annual mandatory employer contributions to each employee’s account, does not require an actuary, and does not require PBGC insurance.

The other choices are incorrect:

A cash balance pension plan requires an actuary and PBGC insurance.
A target benefit pension plan requires an actuary at the inception of the plan, but not on an annual basis.
A traditional defined benefit pension plan requires the services of an actuary annually as well as PBGC insurance.
LO 2.3.1

How well did you know this?
1
Not at all
2
3
4
5
Perfectly
96
Q

In which of the following ways are target benefit pension plans similar to money purchase pension plans?

I. Pension Benefit Guaranty Corporation (PBGC) insurance is required.

II. The actual dollar retirement benefit is guaranteed.

III. The employee bears the investment risk.

IV. Each employee has an individual account.

A) I, II, III, and IV
B) III and IV
C) I and II
D) I, II, and III

A

B) Explanation
Statements III and IV are correct. PBGC insurance is not available for target benefit or money purchase pension plans. Also, the actual dollar retirement benefit is not guaranteed in either of these plans.

LO 2.3.2

How well did you know this?
1
Not at all
2
3
4
5
Perfectly
97
Q

Under which of the following circumstances is a target benefit plan the most appropriate choice for small-business owners?

I. To simplify and reduce the cost of eliminating a defined benefit plan (without termination) by amending it into a target plan

II. Where the employer wants to provide larger retirement benefits for key employees who are significantly older than the other employees

III. To meet the employer’s goal of maximizing deductible contributions to provide benefits for older, highly compensated employees

IV. Where the employer is opposed to assuming the investment risk and prefers the simplicity of a separate account plan

A) I, II, and IV
B) I and III
C) II, III, and IV
D) II and IV

A

D) Explanation
Statements II and IV are correct for the following reasons. Target benefit plans have benefit formulas similar to those of defined benefit plans, which favor employees who are significantly older and higher paid than the average employee of the employee group. Because target benefit plans use separate accounts, the participants bear the risk of the plan’s investment performance. Statement I is incorrect because amending a defined benefit plan into a target plan will result in termination of the defined benefit plan. Statement III is not true since a DC plan is limited to 25%, and a DB plan is not limited by a set percentage. Therefore, the DB plan will usually provide a greater tax deduction if an age-weighted plan works best. The key word to Statement III is “maximizing.”

LO 2.3.2

How well did you know this?
1
Not at all
2
3
4
5
Perfectly
98
Q

In which of the following ways is a target benefit pension plan similar to a traditional defined benefit pension plan?

A) It benefits older employees
B) Investment returns have no bearing on benefit
C) Employer contribution is flexible
D) Employee assumes the investment risk

A

A) Explanation
A target benefit pension plan is similar to a traditional defined benefit plan in that they are both designed to benefit older employees. The employer bears the risk in the defined benefit plan only, and the employer contribution is not flexible in either plan. In a target benefit pension plan, the investment returns do affect the final benefit.

LO 2.3.2

How well did you know this?
1
Not at all
2
3
4
5
Perfectly
99
Q

Which of the following is a hybrid retirement plan that uses an actuarially designed benefit formula like that of a traditional defined benefit plan and individual accounts like that of a defined contribution plan?

A) Target benefit pension plan
B) Cash balance pension plan
C) Age-weighted profit-sharing plan
D) Money purchase pension plan

A

A) Explanation
The plan described is a target benefit pension plan. The contribution is derived from an actuarially designed benefit formula in a target benefit pension plan, but once determined, the plan resembles a money purchase pension plan in all other ways. Cash balance pension plans are defined benefit pension plans that promise a benefit based on a hypothetical account balance versus a traditional defined benefit plan, which promises a monthly retirement benefit for life. Age-weighted profit-sharing plans are defined contribution plans that allocate contributions to participants in such a way that when contributions are converted to equivalent benefit accruals (stated as a percentage of compensation), each participant receives the same rate of benefit accrual. A money purchase pension plan does not use an actuarially designed benefit formula.

LO 2.3.2

How well did you know this?
1
Not at all
2
3
4
5
Perfectly
100
Q

Which of the following is a retirement plan that requires the initial services of an actuary, the employee assumes the investment risk, and it favors older plan participants?

A) Target benefit pension plan
B) Money purchase pension plan
C) Traditional defined benefit plan
D) Traditional profit-sharing plan

A

A) Explanation
The question is describing a target benefit pension plan.

LO 2.3.2

How well did you know this?
1
Not at all
2
3
4
5
Perfectly
101
Q

Which of the following statements regarding target benefit pension plans is FALSE?

A) Each plan participant has an individual account in the plan.

B) Target benefit pension plans favor older participants.

C) The ultimate account balance for the employee’s account is guaranteed by the plan.

D) Target benefit pension plans are a type of defined contribution plan.

A

C) Explanation
A target benefit pension plan is a type of defined contribution plan and, as in all other defined contribution plans, there is no guaranteed ultimate account balance. The plan favors older participants by making a larger contribution for an older plan entrant than for a younger plan entrant with exactly the same salary.

LO 2.3.2

How well did you know this?
1
Not at all
2
3
4
5
Perfectly
102
Q

Netcyber, Inc., is a 10-year-old C corporation that has experienced dramatic growth during three of the past five years. There are currently 168 employees. The two owners, who are 35 and 31 years old, wish to establish a retirement plan. Which plan would probably be the best for them?

A) Profit-sharing plan
B)Target benefit pension plan
C) Age-weighted profit-sharing plan
D) Money purchase pension plan

A

A) Explanation
A profit-sharing plan would be most appropriate, as the owners could choose when to make contributions. Also, because the owners are young, they would have more time to receive contributions on their own behalf. A money purchase pension plan or target benefit pension plan would not be appropriate because this young company would not want to commit a large sum of money every year. The company probably would want the ability to reinvest the money in the firm if needed. The target benefit pension and age-weighted profit-sharing plans are also designed for older owner-employees.

LO 3.1.1

How well did you know this?
1
Not at all
2
3
4
5
Perfectly
103
Q

Under a profit-sharing plan

A) the company must make annual contributions.
B) up to 25% of the plan’s assets can be invested in the employer’s stock.
C) the company has flexibility regarding annual funding.
D) the employer bears investment risk.

A

C) Explanation
The company has funding flexibility. Pension plans can invest only up to 10% of plan assets in employer stock. Profit-sharing plans have no restrictions regarding investment in employer stock. The employer may deduct a contribution limited to only 25% of participating employees’ covered compensation. They must make substantial and recurring employer contributions, or the IRS will remove the plan’s qualified status. The employee bears investment risk.

LO 3.1.1

How well did you know this?
1
Not at all
2
3
4
5
Perfectly
104
Q

Which of these statements regarding profit-sharing plans is false?

A) The maximum tax-deductible employer contribution to a profit-sharing plan is 25% of total (aggregate) eligible employee covered compensation.

B) Profit-sharing plans are types of qualified defined contribution plans.

C) Profit-sharing plans are best suited for companies that have fluctuating cash flows.

D) A company that adopts a profit-sharing plan must make contributions each year.

A

D) Explanation
Profit-sharing plans are not required to make contributions each year.

LO 3.1.1

How well did you know this?
1
Not at all
2
3
4
5
Perfectly
105
Q

A client’s employer has recently implemented a traditional Section 401(k) plan as part of its profit-sharing plan. Which of these is CORRECT regarding the client’s participation in the plan?

A) The client will not pay current federal income taxes on amounts distributed from the plan.

B) The client is always immediately vested in all employer-matching contributions and their accrued benefits.

C) The client is immediately vested in all elective deferrals and their accrued earnings.

D) The client will not pay Social Security (FICA) taxes on amounts paid into the plan.

A

C) Explanation
Because a Section 401(k) plan is a qualified defined contribution plan, the employee is immediately vested (100%) in all elective deferrals and their accrued benefits. Such deferrals are, however, subject to FICA taxes. Vesting schedules may be used with employer-matching contributions.

LO 3.3.1

How well did you know this?
1
Not at all
2
3
4
5
Perfectly
106
Q

Valerie earns $320,000 annually from XYZ Corporation. The company profit-sharing plan provides for a contribution of 25% of participant compensation. What is the amount of the company’s contribution for Valerie for 2021?

A) $100,000
B) $58,000
C) $230,000
D) $71,250

A

B) Explanation
Valerie earns $320,000 annually in 2021. The plan provides for a contribution of 25% of participants’ compensation. However, the maximum compensation that can be taken into consideration is $290,000. Twenty-five percent of $290,000 is $72,500. Finally, the maximum annual additions limit is $58,000, making the maximum company contribution for Valerie $58,000.

LO 3.1.1

How well did you know this?
1
Not at all
2
3
4
5
Perfectly
107
Q

Which of the following statements regarding the characteristics or use of a profit-sharing plan is CORRECT?

A) The maximum tax-deductible employer contribution to a profit-sharing plan is 15% of covered payroll.

B) A company with a great number of older employees will find the implementation of a traditional profit-sharing plan to be the most beneficial for the older employee-participants.

C) Profit-sharing plans require a fixed, mandatory, annual contribution by the employer to the plan.

D) Profit-sharing plans are best suited for companies that experience fluctuating cash flow.

A

D) Explanation
A profit-sharing plan does not require a mandatory, annual employer contribution. Therefore, they are best suited for companies that experience fluctuating cash flow and would like the flexibility of a discretionary (though substantial and recurring) contribution. The maximum tax-deductible contribution is 25% of covered payroll. Because there is less time to accrue a retirement benefit by retirement age, a profit-sharing plan is not the best choice for a plan with a great number older employee-participants.

LO 3.1.1

How well did you know this?
1
Not at all
2
3
4
5
Perfectly
108
Q

In addition to meeting the financial needs and resources tests for hardship withdrawals, money may only be withdrawn from profit-sharing plan accounts for the following reasons:

I. Purchase of a primary residence

II. Payment of unreimbursed medical expenses

III. Payment necessary to prevent foreclosure on the participant’s primary residence

IV. Payment of higher education expenses for the participant, spouse, or dependent children

A) III and IV
B) I, II, and III
C) I, II, III, and IV
D) I and II

A

C) Explanation
All of the statements regarding hardship withdrawal requirements are correct.

LO 3.1.1

How well did you know this?
1
Not at all
2
3
4
5
Perfectly
109
Q

Jack, age 51, is the owner of an architectural firm with 23 employees, most of whom are younger than 40. The company’s cash flow varies from year to year, depending on their contracts. Jack wants to implement a qualified plan that is easy for employees to understand and that is administratively cost-effective. He also wants a plan with an incentive feature by which an employee’s account balance increases with company profits. Which of the following plans would be most appropriate for Jack’s firm?

A) Defined benefit pension plan
B) Money purchase pension plan
C) Section 403(b) plan
D) Traditional profit-sharing plan

A

D) Explanation
A traditional profit-sharing plan may be appropriate when an employer’s profits, or cash flow, fluctuate from year to year;
an employer wishes to implement a qualified plan with an incentive feature by which an employee’s account balance increases with employer profits; or
most employees are young (under age 50) and have substantial time to accumulate retirement savings, and the employees are, most likely, willing to accept a degree of investment risk in their individual accounts.
Jack’s company is not eligible to implement a Section 403(b) plan, as it is not a nonprofit organization. A defined benefit plan is not appropriate for the company’s employee demographics, requires stable cash flows, and can be expensive to administer. Money purchase plans also require a mandatory contribution each year and do not provide the incentive feature regarding company profits.

LO 3.1.1

How well did you know this?
1
Not at all
2
3
4
5
Perfectly
110
Q

The Acme Corporation has six owners, ranging in age from 30 to 60 years old, and 25 rank-and-file employees. The owners want to adopt a qualified retirement plan that will allow them to maximize the contributions to the owners’ accounts and minimize the contributions to the accounts of the rank-and-file employees. Which of the following plans would best meet the owners’ needs?

A) Section 401(k) plan
B) Self-employed Keogh plan
C) Age-based profit-sharing plan
D) New comparability plan

A

D) Explanation
A new comparability plan would allow the owners to divide the participants into two classes based on their compensation levels and allocate different contribution levels to the classes. An age-based profit-sharing plan wouldn’t meet their objectives because the owners’ ages are significantly different. Section 401(k) plans are subject to discrimination testing, and a self-employed Keogh plan is inappropriate because the owners aren’t self-employed.

LO 3.2.1

How well did you know this?
1
Not at all
2
3
4
5
Perfectly
111
Q

How is an age-weighted profit-sharing plan similar to a traditional defined benefit pension plan?

A) Contribution allocations to older participants may be maximized, while allocations to younger participants may be minimized.

B) Retirement benefits are determined by the participant’s final account balance.

C) Minimum vesting schedules are more liberal than in other types of plans.

D) Employer contributions are flexible from one year to another and, if resources are not available, the employer may choose not to contribute to the plan.

A

A) Explanation
Contribution formulas for both age-weighted profit-sharing plans and traditional defined benefit pension plans favor older employees. Retirement benefits are determined by the participant’s final account balance in defined contribution plans. Employer contributions can be flexible in an age-weighted profit-sharing plan, but not in a defined benefit pension plan. Vesting is not more liberal in these plans than in other types of plans.

LO LO 3.2.1

How well did you know this?
1
Not at all
2
3
4
5
Perfectly
112
Q

Which of the following plans is NOT a cross-tested plan?

I. New comparability plan
II. Employer stock ownership plan
III. Age-based profit-sharing plan
IV. Stock bonus plan

A) III and IV
B) II only
C) II and IV
D) I, II, III, and IV

A

C) Explanation
Of the plans listed, the employer stock ownership plan and the stock bonus plans are not cross-tested plans. Only the new comparability plan and the age-based profit-sharing plan are cross-tested plans.

LO 3.2.1

How well did you know this?
1
Not at all
2
3
4
5
Perfectly
113
Q

Which of the following plans is a cross-tested plan?

I. New comparability plan
II. Employee stock ownership plan (ESOP)
III. Age-based profit-sharing plan
IV. Stock bonus plan

A) I, II, III, and IV
B) III and IV
C) I only
D) I and III

A

D) Explanation
Of the plans listed, only the new comparability plan and the age-based profit-sharing plan are cross-tested plans. Cross-testing means a defined contribution plan is tested for nondiscrimination based on benefits rather than contributions.

LO LO 3.2.1

How well did you know this?
1
Not at all
2
3
4
5
Perfectly
114
Q

Bernie is a participant in his employer’s noncontributory employee stock ownership plan (ESOP). Two years ago, his employer contributed stock with a fair market value of $30,000 into Bernie’s account. Bernie retired one year later and took distribution of the stock when its fair market value was $40,000. Two years after his retirement, Bernie sold the stock for $50,000. What is the appropriate tax treatment available to Bernie upon sale of the stock?

A) $20,000 ordinary income
B) $20,000 long-term capital gain
C) $10,000 long-term capital gain
D) $50,000 ordinary income

A

B). Explanation
Employees are not taxed on stock in an ESOP until the stock is distributed. Upon distribution, the employee must pay ordinary income taxes on the fair market value of the stock when it was contributed to the plan on his behalf. Any net unrealized appreciation (NUA) at that time can be deferred until the stock’s subsequent sale. Upon the sale, the NUA portion will be treated as long-term capital gain. Additionally, the growth of the stock subsequent to the distribution will receive long-term capital gain treatment because Bernie held the stock longer than one year after distribution. Therefore, the appropriate tax treatment available to Bernie upon sale of the stock is a $20,000 long-term capital gain.

LO 3.2.2

How well did you know this?
1
Not at all
2
3
4
5
Perfectly
115
Q

ABC Corporation is a closely held company that wants to establish a qualified retirement plan for its employees. Also, the company wants to improve the marketability of its stock and give the employees an ownership stake in the company. Which of the following plans would help ABC meet all of these objectives?

A) SIMPLE 401(k)
B) Keogh (self-employed) plan
C) Target benefit pension plan
D) Employee stock ownership plan (ESOP)

A

D) Explanation
An ESOP could help ABC meet all of these objectives.

LO 3.2.2

How well did you know this?
1
Not at all
2
3
4
5
Perfectly
116
Q

An employee stock ownership plan (ESOP) is a defined contribution plan that may provide the employer with which of the following advantages?

I. Increased corporate cash flow
II. The ability to borrow money to purchase corporate stock
III. A market for employer stock
IV. Financial resources to expand the business

A) I, II, and III
B) I and II
C) III and IV
D) I, II, III, and IV

A

D) Explanation
All of these statements are ESOP advantages to the employer.

LO 3.2.2

How well did you know this?
1
Not at all
2
3
4
5
Perfectly
117
Q

Which of the following legal requirements apply to employee stock ownership plans (ESOPs)?

I. ESOPs must permit participants who have reached age 55 and have at least 10 years of service the opportunity to diversify their accounts.

II. ESOPs cannot be integrated with Social Security.

III. An employer’s deduction for ESOP contributions and amounts made to repay interest on an ESOP’s debt cannot exceed 25% of the participants’ payroll.

IV. The mandatory 20% income tax withholding requirement does not apply to distributions of employer stock from an ESOP.

A) II and III
B) I, II, and IV
C) I, II, and III
D) I and II

A

B) Explanation
Options I and II correctly state the diversification rule and the rule that prohibits integrated ESOPs. There is no limit on amounts used to pay interest on ESOP debt. ESOP distributions of employer stock only are not subject to the 20% income tax withholding requirement.

LO 3.2.2

How well did you know this?
1
Not at all
2
3
4
5
Perfectly
118
Q

An employer is permitted to make what level of contributions to an employee stock ownership plan (ESOP)?

A) An ESOP is a defined contribution plan that limits employer contributions to 25% of covered payroll.

B) An ESOP is a defined benefit plan. Thus the contribution limit is the amount required to fund the projected benefits.

C) An ESOP is a stock bonus plan that permits a maximum contribution to a non-leveraged ESOP of 35% (rather than the 25% that would otherwise apply).

D) An ESOP is a stock bonus plan that permits a maximum contribution to a leveraged ESOP of 27% (rather than the 25% that would otherwise apply).

A

A) Explanation
Defined contribution plans limit employer contributions to 25% of covered payroll. With a leveraged ESOP, the employer may deduct both the principal (up to 25% of compensation paid to covered employee-participants) and additional amounts paid as interest as the loan is paid off. An ESOP is not a defined benefit plan.

LO 3.2.2

How well did you know this?
1
Not at all
2
3
4
5
Perfectly
119
Q

Which of these are basic provisions of an IRC Section 401(k) plan?

I. Employee elective deferrals are exempt from income tax withholding and FICA and FUTA taxes.

II. An employer’s deduction for contributions to a Section 401(k) plan cannot exceed 25% of covered payroll, which is not reduced by the employees’ elective deferrals.

III. A Section 401(k) plan cannot require as a condition of participation that an employee complete a period of service longer than one year.

IV. Employee elective deferrals may be made from salary or bonuses.

A) II, III, and IV
B) I and IV
C) I and III
D) II and IV

A

A) Explanation
Options II, III, and IV correctly describe the 25% employer deduction limitation, eligibility requirement, and potential sources of employee elective deferrals for Section 401(k) plans. Option I is incorrect because employee elective deferrals (i.e., salary deferrals) are subject to FICA and FUTA taxes. Option III is correct. There can be a two-year period of service requirement if the participants are 100% immediately vested for some other employer retirement plans. However, this does not apply for 401(k) plans.

LO 3.3.1

How well did you know this?
1
Not at all
2
3
4
5
Perfectly
120
Q

Which of these statements regarding Section 401(k) plans is CORRECT?

I. Employer contributions and their accrued earnings are always immediately 100% vested in the plan.

II. A Section 401(k) plan is a qualified profit-sharing or stock bonus plan.

III. The maximum pretax elective deferral for a participant age 35 is $19,500 in 2021.

IV. Elective deferrals in a cash or deferred arrangement (CODA) are not subject to FICA and FUTA taxes.

A) II and III
B) I and IV
C) III only
D) I, II, III, and IV

A

A) Explanation
Statements II and III are correct. Statement I is incorrect. Employee contributions and investment earnings are always 100% vested, not the employer contributions. Statement IV is incorrect. Elective deferrals in a CODA are subject to FICA and FUTA taxes.

LO 3.3.1

How well did you know this?
1
Not at all
2
3
4
5
Perfectly
121
Q

What is the maximum elective deferral a participant can make to a Section 401(k) plan in 2021, assuming no catch-up provisions apply?

A) $13,500
B) $6,000
C) $57,000
D) $19,500

A

D) Explanation
The elective deferral limit for 2021 to a Section 401(k) is $19,500. The maximum annual additions limit is $58,000.

LO 3.3.1

How well did you know this?
1
Not at all
2
3
4
5
Perfectly
122
Q

A Section 401(k) plan allows plan participants the opportunity to defer taxation on a portion of regular salary or bonuses simply by electing to have such amounts contributed to the plan instead of receiving them in cash. Which of these is a rule that applies to Section 401(k) plan elective deferrals?

I. Section 401(k) plan elective deferrals are immediately 100% vested and cannot be forfeited.

II. In-service withdrawals are to be made only if an individual has attained age 62.

III. An extra nondiscrimination test called the actual deferral percentage test applies to elective deferral amounts.

A) III only
B) I and II
C) I only
D) I and III

A

D) Explanation
Statement II is incorrect. A Section 401(k) plan may allow in-service distributions before age 62.

LO 3.3.2

How well did you know this?
1
Not at all
2
3
4
5
Perfectly
123
Q

All of these statements regarding a traditional Section 401(k) plan are CORRECT except

A) a Section 401(k) plan is a qualified profit-sharing or stock bonus plan.

B) employer contributions and their accrued earnings are always immediately 100% vested in the plan.

C) the maximum pretax elective deferral for a participant age 35 is $19,500 in 2021.

D) elective deferrals in a cash or deferred arrangement (CODA) are subject to FICA and FUTA taxes.

A

B) Explanation
Employee contributions and investment earnings are always 100% vested, not the employer contributions. The vesting of employer contributions depends on the type of employer contribution to the plan.

LO 3.3.1

How well did you know this?
1
Not at all
2
3
4
5
Perfectly
124
Q

In 2021, what is the maximum amount an employee under the age of 50 may contribute to a traditional Section 401(k) plan as an elective deferral?

A) $19,500
B) The lesser of 100% of compensation or $57,000 annually
C) $26,000
D) $13,500

A

A) Explanation
The maximum amount of elective deferrals possible in 2021 to a traditional profit-sharing Section 401(k) plan for an individual under age 50 is $19,500 (it is $26,000 for an individual age 50 or older). The lesser amount of $13,500 applies to the SIMPLE 401(k) form.

LO 3.3.1

How well did you know this?
1
Not at all
2
3
4
5
Perfectly
125
Q

Hardship withdrawals are only allowed from Section 401(k) plans if specifically stated in the plan document and typically for expenses such as

I. vacation costs.
II. medical expenses.
III. college tuition costs.
IV. insurance premiums.

A) I and III
B) II, III, and IV
C) I and II
D) II and III

A

D) Explanation
Hardship withdrawals are typically allowed for unreimbursed medical expenses, college tuition and fees, to purchase a principal residence, burial expenses for spouse or dependents, and to prevent eviction from one’s principal residence or foreclosure on the mortgage of such residence.

LO 3.1.1

How well did you know this?
1
Not at all
2
3
4
5
Perfectly
126
Q

A Section 401(k) plan does not have to satisfy the ADP and ACP tests if it meets one of the safe harbor provisions. All of these about the safe harbor provisions are correct except

A) a plan that satisfies the safe harbor provisions is not subject to the top-heavy rules.

B) an employer may satisfy the safe harbor provisions by making non-elective contributions for all eligible employees.

C) an employer may satisfy the safe harbor provisions by making certain matching contributions for non-highly compensated employees who participate in the plan.

D) mandatory employer contributions under the safe harbor provisions may be subject to a three-year cliff vesting requirement.

A

D) Explanation
Employees must be immediately 100% vested in mandatory employer contributions that are made under the safe harbor provisions. The mandatory contributions can take the form of either a non-elective contribution to all eligible employees or matching contributions to participating non-highly compensated employees who participate in the plan.

LO 3.3.1

How well did you know this?
1
Not at all
2
3
4
5
Perfectly
127
Q

Which of these statements regarding the safe harbor rules for Section 401(k) plans is CORRECT?

I. The employer can avoid ACP and ADP testing if it matches 100% up to 4% of compensation for nonhighly compensated employees.

II. The employer can avoid ACP and ADP testing if it makes contributions of 3% or more of compensation for all employees who are eligible to participate in the plan, whether or not the employee chooses to participate.

III. To meet the safe harbor requirements, the matching and non-elective contributions must be immediately 100% vested.

IV. The employer must provide notice to each eligible employee about rights and obligations under the plan.

A) II and III
B) III and IV
C) IV only
D) I, II, III, and IV

A

D) Explanation
All of these statements regarding Section 401(k) plan safe harbor rules are correct.

LO 3.3.1

How well did you know this?
1
Not at all
2
3
4
5
Perfectly
128
Q

Northwest Instruments Corp. made matching contributions to its SIMPLE 401(k) in the last three years. Assume all eligible employees earn at least the maximum includible compensation limit and all defer the maximum amount allowed. Due to extensive capital expenses anticipated this year, the company is considering how to reduce expenses. It will not be able to continue to make the 3% matching contribution and has called you to discuss their options. Which of these could you recommend?

A) Because they have satisfied the 3% matching contribution for three years, Northwest Instruments Corp. could reduce the matching contribution to 1%.

B) By providing adequate notice, Northwest Instruments Corp. could move to the 2% non-elective contribution this year, although the savings would be minimal.

C) Employer contributions under a SIMPLE plan are discretionary, and Northwest Instruments Corp. could provide notice that they will not provide any contribution this year.

D) With SIMPLE 401(k) plans, employers who begin using the 3% matching contribution do not have any option available to modify the company’s contribution.

A

B) Explanation
Under a SIMPLE 401(k), an employer does not have the option to change the percentage used for the matching contribution. A SIMPLE IRA would be able to move the match down to 1% for two out of five years. However, in this case the employer can use the 2% non-elective contribution instead of the 3% matching contribution. Under the scenario described, however, the savings would be minimal.

LO 3.3.3

How well did you know this?
1
Not at all
2
3
4
5
Perfectly
129
Q

Grant, age 51, made an initial contribution of $10,000 to a Roth 401(k) in 2013. He made subsequent contributions of $6,000 annually for the next four years. In 2021, Grant separated from service and took a $50,000 distribution from his Roth 401(k) to purchase a boat. Which of these statements regarding this distribution is CORRECT?

A) It is income tax free because it was made after five taxable years from the date of initial contribution.

B) It is taxable because it was within 10 taxable years from the date of initial contribution.

C) It is income tax free because it was made after five taxable years, and Grant is over age 50.

D) It is partially taxable because Grant was not age 59½ or older, disabled, or deceased.

A

D) Explanation
Although Grant took the distribution after five taxable years from the date of initial contribution, he did not meet one of the other requirements for a qualified distribution (it must be made after the individual attained age 59½, the individual must be disabled, or the distribution must be made to a beneficiary or estate of an individual on or after the individual’s death). In this case, the first $34,000 is counted against his contributions. Thus, there is no tax or penalty on $34,000. There are no conversions, so the rest of the distribution is earnings and thus subject to income tax and the 10% early distribution rules.

LO 3.3.3

How well did you know this?
1
Not at all
2
3
4
5
Perfectly
130
Q

Ryan wants to take a distribution from his SIMPLE 401(k) account balance from his previous employer and deposit it in an IRA. Which of these statements regarding his transfer is CORRECT?

I. The distribution from the SIMPLE 401(k) plan is not subject to the mandatory 20% income tax withholding requirement.

II. A direct transfer from Ryan’s SIMPLE 401(k) to an IRA is not subject to the mandatory 20% income tax withholding requirement.

A) I only
B) II only
C) Neither I nor II
D) Both I and II

A

B) Explanation
Statement I is incorrect. SIMPLE 401(k) plans are qualified plans and are subject to mandatory 20% income tax withholding for a distribution that is not a direct trustee-to-trustee transfer. When there is a direct transfer of a distribution from a qualified plan to an IRA, the mandatory 20% withholding rule does not apply.

LO 3.3.3

How well did you know this?
1
Not at all
2
3
4
5
Perfectly
131
Q

Ellen participates in a SIMPLE 401(k) maintained by her employer. If she has completed two years of service, to what extent is she vested in the employer contributions to her account?

A) 100%
B) 40%
C) 0%
D) 20%
Explanation
A

A) Vesting schedules are not permitted in SIMPLE 401(k)s. Employees are always 100% vested in employer contributions.

LO 3.3.3

How well did you know this?
1
Not at all
2
3
4
5
Perfectly
132
Q

The actual deferral percentage (ADP) test is generally applicable to which of these?

I. Traditional Section 401(k) plans
II. Money purchase pension plans
III. SIMPLE 401(k)s

A) III only

B) I and II

C) II only

D) I only

A

D) Explanation
The ADP test applies only to traditional Section 401(k) plans. The test does not apply to SIMPLE 401(k)s or money purchase pension plans. The ADP deals with worker deferrals. A money purchase plan is essentially an employer contribution only plan.

LO 3.3.3

How well did you know this?
1
Not at all
2
3
4
5
Perfectly
133
Q

Which one of the following would NOT be an acceptable course of action for a 401(k) plan that has failed the actual deferral percentage (ADP) test?

A) A qualified nonelective contribution for all eligible nonhighly compensated employees

B) A qualified matching contribution for all eligible nonhighly compensation employees

C) A corrective distribution of a portion of the eligible nonhighly compensated employees’ deferrals

D) A corrective distribution to the eligible highly compensated employees

A

C) Explanation
The ADP test fails when too much is being saved or contributed on behalf of highly compensated employees. In order to bring the plan into compliance it would be necessary to make a corrective distribution to a portion of the highly compensated employee deferrals, not the nonhighly compensated employee deferrals. The other three options are stated correctly.

LO 3.3.2

How well did you know this?
1
Not at all
2
3
4
5
Perfectly
134
Q

Which plan is appropriate for a business with a fluctuating cash flow and an owner who is significantly older than the rank-and-file employees?

A) Cash balance
B) Target benefit
C) Defined benefit
D) Age-weighted profit sharing

A

D) Explanation
An age-weighted formula can be used by a profit-sharing plan. This arrangement will work best for a business with a fluctuating cash flow and key employees who are older than the other employees. Pension plans (i.e., cash balance plans and target plans) are subject to minimum funding requirements and may create a problem for a business with a fluctuating cash flow, even though they will maximize contributions for older employees. A service-based formula will favor employees who have many years of service with the business.

LO LO 3.2.1

How well did you know this?
1
Not at all
2
3
4
5
Perfectly
135
Q

George Cobb owns Cobb Construction Inc., which is worth $3 million with an annual payroll of $1.3 million. George receives a salary of $300,000. The company initiated a profit-sharing plan last year and makes contributions at year-end. The plan has about $120,000 in assets. George’s account has a balance of $28,000. He is 60 and wants to retire this year and sell his company. His problem is that the construction industry is very slow this year, and the major companies are retrenching, not expanding. George’s company is very profitable from year to year, earning a profit of $550,000 to $650,000 each year. George has an excellent management team in place that he has carefully built and trained over the years. His bank approached him recently, offering to loan him almost any amount, up to the value of his company, at 8%. Typically such loans are amortized over no more than 10 years.

How can George use a qualified plan to accomplish his goal?

A) George should have the company profit-sharing plan borrow $3 million and buy his stock.

B) George should borrow $3 million from the bank and retire.

C) George should amend the company profit-sharing plan and make it a leveraged employee stock ownership plan (LESOP), then have the plan borrow $3 million and buy his stock.

D) George should amend the company profit-sharing plan and make it an employee stock ownership plan (ESOP) and have the ESOP buy his stock.

A

C) Explanation
George should amend the profit-sharing plan, making it a LESOP. Then, as the LESOP trustee, he can borrow $3 million in the name of the LESOP from a bank familiar with LESOP loans. The LESOP will receive $3 million from the loan and it will use the proceeds to buy George’s $3 million worth of stock and become the owner of the company. George will receive the $3 million in cash and retire. Only a LESOP can borrow money. Borrowing $3 million doesn’t extract the value of his company for his benefit; it only creates debt and a nondeductible interest cost.

LO 3.2.2

How well did you know this?
1
Not at all
2
3
4
5
Perfectly
136
Q

Total annual contributions to an individual participant in a traditional Section 401(k) plan are limited in 2021 to

A) $19,500.

B) the lesser of 100% of compensation, or $58,000.

C) $290,000.

D) $13,500.

A

B) Explanation
Total annual contributions to a participant’s account are limited in 2021 to the lesser of 100% of employee compensation, or $58,000, with only the first $290,000 of employee compensation considered in the contribution formula. The total contribution is made up of the worker contribution, the employer contribution, and reallocated forfeitures. The worker contribution alone is limited to $19,500 in 2021 for those 49 and younger.

LO 3.3.1

How well did you know this?
1
Not at all
2
3
4
5
Perfectly
137
Q

Which statements describe one or more characteristics of hardship withdrawals from Section 401(k) plans?

I. Hardship withdrawals may be taken from elective deferral amounts and the investment income from such deferrals.

II. The participant must demonstrate “immediate and heavy financial need” and not have any other resources that are “reasonably available.”

III. A hardship withdrawal is exempt from the 10% early withdrawal penalty if the participant has not reached age 59½.

IV. Hardship conditions must be established for hardship withdrawals from profit sharing or stock bonus 401(k) plans.

A) I and II

B) II, III, and IV

C) I and III

D) II and IV

A

D) Explanation
Hardship withdrawals from profit sharing or stock bonus Section 401(k) plans require that hardship conditions (“immediate and heavy financial need” and other resources not “reasonably available”) be demonstrated. Only elective deferral amounts—not the associated earnings—are available for hardship withdrawals.

LO 3.3.1

How well did you know this?
1
Not at all
2
3
4
5
Perfectly
138
Q

Tina is a participant in her company’s employee stock ownership plan (ESOP). The company transfers 1,000 shares of employer stock at $5 per share to her account. Several years later, when the stock is $11 per share, Tina retires at age 61. If she elects to receive the stock in a lump sum at retirement and later sells the stock for $12,000, what are the tax consequences to Tina?

A) Tina will be taxed on $11,000 of ordinary income when she receives the stock at retirement.

B) Tina will have ordinary income of $12,000 when she sells the stock.

C) Tina will have a capital gain of $7,000.

D) Tina will have a capital gain of $1,000.

A

C) Explanation
Because of the net unrealized appreciation rule, at distribution, Tina will be taxed only on $5,000, the original cost of the stock when it was contributed to the plan. Tina was taxed on the $5,000 basis in the stock when she received her distribution. Therefore, her capital gain will be $7,000 if she sells the stock for $12,000.

LO 3.2.2

How well did you know this?
1
Not at all
2
3
4
5
Perfectly
139
Q

The Acme Corporation has six owners, ranging in age from 30 to 60 years old, and 25 rank-and-file employees. The owners want to adopt a qualified retirement plan that will allow them to maximize the contributions to the owners’ accounts and to minimize the contributions to the accounts of the rank-and- file employees. Which of the following plans would best meet the owners’ needs?

A) New comparability plan

B) Keogh plan

C) Section 401(k) plan

D) Age-based profit-sharing plan

A

A) Explanation
A new comparability plan would allow the owners to divide the participants into two classes based on their compensation levels, and to allocate different contribution levels to the classes. An age-based profit-sharing plan wouldn’t meet their objectives because the owners’ ages are significantly different. Section 401(k) plans are subject to discrimination testing, and a Keogh plan is inappropriate because the owners are not self-employed.

LO LO 3.2.1

How well did you know this?
1
Not at all
2
3
4
5
Perfectly
140
Q

The actual deferral percentage (ADP) test looks at the

A) employee deferrals (ADP).

B) age 50 and over catch-up contribution amounts.

C) amount of employer contributions.
Failed

D) amount of employee after-tax contributions.

A

A) Failed
Explanation
The ADP test looks at employee deferrals (ADP).

LO 3.3.2

How well did you know this?
1
Not at all
2
3
4
5
Perfectly
141
Q

Which of the following is a test that must be met before a hardship withdrawal from a profit-sharing plan?

I. The financial needs test requires the hardship be due to an immediate significant financial need of the participant-employee.

II. The resources test requires that the participant must not have other financial resources sufficient to satisfy the need.

A) II only

B) Neither I nor II

C) Both I and II

D) I only

A

Explanation
Both statements describe the tests that must be met to qualify for a hardship withdrawal.

LO 3.1.1

How well did you know this?
1
Not at all
2
3
4
5
Perfectly
142
Q

Jason, a CFP® professional, has a corporate client whose only two shareholders, ages 57 and 61, have asked for assistance with selecting a corporate retirement plan. His clients have annual compensation from the corporation of $145,000 each. They have nine employees, ages 21–40, with annual compensation of $18,000–$55,000. The clients want a qualified plan that offers maximum benefits for themselves, minimum benefits for the employees, contribution flexibility, and low administrative costs. Which of the following retirement plans should Jason recommend?

A) Defined benefit pension plan

B) Traditional profit-sharing plan

C) Simplified employee pension (SEP) plan

D) Age-weighted profit sharing

A

D) Explanation
An age-weighted profit-sharing plan is a qualified plan that would offer maximum benefits for the shareholders, who are older. An age-weighted profit-sharing plan would also offer flexible contributions and low administrative costs. A defined benefit pension plan would not have contribution flexibility or low administrative costs. Although a profit-sharing plan would offer contribution flexibility and low administrative costs, it would not allow maximum benefits for the principals. A SEP is not a qualified plan.

LO LO 3.2.1

How well did you know this?
1
Not at all
2
3
4
5
Perfectly
143
Q

If the actual deferral percentage (ADP) for nonhighly compensated employees is 9%, what is the maximum deferral percentage for highly compensated employees?

A) 11.00%

B) 18.00%

C) 11.25%

D) 12.50%

A

C) Explanation
For amounts higher than 8%, the allowable spread between NHCEs and HCEs is 1.25 (125%). So for this question, 9% × 1.25 = 11.25%. Besides the 1.25 times test, the other test is the 200% rule with a maximum difference of 2%. In this case, 200% is 18%, but the maximum difference of 2% brings this test down to 11%. Since 11.25% is higher, that test decides how much highly compensated employees could contribute.

LO 3.3.2

How well did you know this?
1
Not at all
2
3
4
5
Perfectly
144
Q

Which one of the following types of plan formulas should be considered by a relatively new business with a fluctuating cash flow and key employees who are significantly older than the rank-and-file employees?

A) Target benefit
B) Age weighted
C) Service based
D) Cash balance

A

B) Explanation
An age-weighted formula can be used by a profit-sharing plan. This arrangement will work best for a business with a fluctuating cash flow and key employees who are older than the other employees. Pension plans (i.e., cash balance plans and target plans) are subject to minimum funding requirements and may create a problem for a business with a fluctuating cash flow, even though they will maximize contributions for older employees. A service-based formula will favor employees who have many years of service with the business.

LO 3.2.1

How well did you know this?
1
Not at all
2
3
4
5
Perfectly
145
Q

If a tax-sheltered annuity (TSA) plan provides for employer-matching contributions, which of the following tests apply?

A) Actual deferral percentage (ADP) test
B) Neither test applies
C) Both the ACP and ADP tests
D) Actual contribution percentage (ACP) test

A

D) Explanation
While the ACP test applies to TSA plans, both the ACP and the ADP tests apply to 401(k) plans.

LO 3.3.2

How well did you know this?
1
Not at all
2
3
4
5
Perfectly
146
Q

Albert, 52, is the sole proprietor of a consulting business that will have Schedule C net income of $80,000 this year. The business maintains a profit-sharing Keogh plan and Albert is the only plan participant. What is the maximum that can be contributed to the plan this year on Albert’s behalf? The self-employment tax for Albert is $11, 304.

A) $20,000
B) $16,680
C) $23,500
D) $14,870

A
D) Explanation
Schedule C net income	$ 80,000
Less 7.65%	(6,120)
Self-employment income subject to self-employment taxes	$ 73,880
Times 15.3%	
Self-employment tax	$ 11,304

One-half of the self-employment tax is deductible as an adjustment to income. In this example, the deductible portion is $5,652 ($11,304 ÷ 2).

Determine the adjusted contribution percentage for the Albert, the business owner.

Maximum percentage contribution for participants

(employee %)

.25
Divided by (1 + employee %) 1.25
Equals adjusted contribution percentage for owner = .20

Schedule C net profit (business profit) $80,000
Less income tax deduction allowed (1/2 self-employment tax) ($5,652)
Net earnings from self-employment $74,348
Multiply by .20 =
Owner’s contribution $14,870

Note that $14,870 is 25% of $59,478 and $59,478 is the amount you get from taking the net earnings of $74,348 and subtracting $14,870. Thus, the IRS would argue that the self-employed owner is getting the same 25% contribution as the worker after all the accounting is factored in.

LO 3.3.3

How well did you know this?
1
Not at all
2
3
4
5
Perfectly
147
Q

The ABC Company’s 401(k) has just been changed to allow Roth elective deferrals. Which of the following does a Roth 401(k) plan have in common with a Roth IRA?

They both are phased out for higher income earners.
Withdrawals after age 59½ are tax free if the account is at least five years old.
Contributions are not deductible.
They are both subject to required minimum distributions (RMDs) for retirees starting after attaining age 72.

A) II and IV

B) I and IV

C) II and III

D) I and II

A

C) Explanation
Although Roth IRAs have an additional reason for qualified distributions (up to $10,000 of first-time homebuyer expenses), a distribution from either type of Roth account after five years and over age 59½ would be a qualified distribution. No type of Roth contribution is deductible. Roth IRAs are not subject to RMDs while the original owner is alive, but employer Roth accounts for retirees are subject to the normal RMD rules. Roth IRAs are phased out for higher earners, but not for employer Roth accounts.

LO 3.3.3

How well did you know this?
1
Not at all
2
3
4
5
Perfectly
148
Q

Which of the following are provisions of qualified stock bonus plans?

I. Taxation of the capital gain on employer stock held in the plan may be deferred beyond the distribution date.

II. Like profit-sharing plans, stock bonus plans allow for flexible employer contributions.

III. Social Security integration is not allowed in stock bonus plans.

IV. Participants must be allowed to receive their distributions in shares of employer stock that is publicly traded.

V. If the employer’s securities are not readily tradable on an established market, a participant who separates from service must be provided a put option that will be available for at least 60 days after distribution of the stock.

A) I, II, IV, and V

B) I, II, and III

C) II and IV

D) I, III, and V

A

A) Explanation
Assuming the participant elects NUA treatment, the unrealized gain in the value of the stock is not taxed until the stock is sold. Stock bonus and profit-sharing plans both have the flexible employer contribution characteristics. Participants have the option to take the employer’s stock under either. When the employer’s securities are not readily tradable on an established market, a participant who separates from service must be provided the right to have the employer—not the plan—repurchase the employer securities under a fair valuation formula (i.e., a put option). Although Social Security integration is allowed in stock bonus plans, it is not permitted in employee stock ownership plans (ESOPs).

LO 3.2.2

How well did you know this?
1
Not at all
2
3
4
5
Perfectly
149
Q

Which of the following statements regarding stock bonus plans and employee stock ownership plans (ESOPs) are CORRECT?

I. They allow employees to defer all income taxes on distributed stock until the stock is sold.

II. They can be costly and administratively cumbersome.

III. They always decrease corporate cash flows.

IV. They create a market for employer stock.

A) II and III

B) I and II

C) I, II, III, and IV

D) II and IV

A

D) Explanation
Statements II and IV are correct. Both ESOPs and stock bonus plans give employees a stake in the company through stock ownership. They may increase company cash flow because employers make cashless contributions to the retirement plan and create a market for employer stock. In addition, they both limit the availability of retirement funds to employees if an employer’s stock falls drastically in value and create an administrative and cash-flow problem for employers by requiring them to offer a repurchase option (put option) if their stock is not readily tradable on an established market. Employees must pay income tax on the value of the stock contributed to the plan at the time of distribution. Gain on the stock before a lump-sum distribution is net unrealized appreciation and is not taxed until the employee-participant sells the stock.

LO 3.2.2

How well did you know this?
1
Not at all
2
3
4
5
Perfectly
150
Q

Which of the following is a type of defined contribution profit-sharing plan?

A) Money purchase pension plan

B) Traditional Section 401(k) plan

C) Cash balance pension plan

D) Target benefit pension plan

A

B) Explanation
Of the choices offered, only a Section 401(k) is a defined contribution profit-sharing plan. The other choices are pension plans.

LO 3.1.1

How well did you know this?
1
Not at all
2
3
4
5
Perfectly
151
Q

Which of the following types of defined contribution plans may borrow money in the name of the plan?

A) A tandem profit-sharing plan and a traditional profit-sharing plan

B) A profit-sharing plan with Section 401(k) provisions

C) An age-weighted profit-sharing plan

D) A leveraged employee stock ownership plan (ESOP)

A

D) Explanation
For qualified plans, an ESOP or LESOP is unique in its ability to borrow money in the plan’s name without violating prohibited transaction rules.

LO 3.2.2

How well did you know this?
1
Not at all
2
3
4
5
Perfectly
152
Q

George, age 55, earns $250,000 and participates in his employer’s SIMPLE IRA. The employer match is on a dollar-for-dollar basis, up to 3% of each participating employee’s compensation for the year. What is the maximum amount of employee and employer contributions that can be contributed to George’s account in 2021?

A) $16,500

B) $19,500

C) $13,500

D) $24,000

A

D) Explanation
The total of the employee and employer contributions that may be made to George’s account in 2021 is $24,000.

$13,500 employee contribution
+ $7,500 employer contribution ($250,000 × 0.03)
+ $3,000 employee age-50-and-over catch-up contribution
$24,000

LO 4.2.1

How well did you know this?
1
Not at all
2
3
4
5
Perfectly
153
Q

A simplified employee pension plan (SEP)

A) is a very complex retirement plan for employees.

B) provides for mandatory funding.

C) allows employers to make contributions to one general account.

D) can be established by any type of employer.

A

D) Explanation
A SEP plan is a very simple retirement plan that provides for flexible funding and allows employers to make contributions directly to participants’ individual IRAs and can be established by any type of employer. The percentage contributed by the employer can fluctuate each year as long as it is the same for every employee. Up to 25% of an employee’s covered compensation is allowable for the tax benefit, up to a maximum of $58,000 (2021).

LO 4.2.2

PREV

How well did you know this?
1
Not at all
2
3
4
5
Perfectly
154
Q

Are the investment options available to 401(k) and 403(b) plans different in any way?

A) No, because both a 401(k) and a 403(b) are qualified plans.

B) No, because both a 401(k) and a 403(b) have unlimited access to all the same investment products.

C) Yes, because 403(b) plans are limited to investing in annuity contracts and/or mutual funds.

D)
Yes, because 401(k) plans can only be invested in mutual funds and/or annuity contracts.

A
C) Explanation
A 403(b) plan is limited to investing in mutual funds and/or annuity contracts. Conversely, the Internal Revenue Code does not specify which instruments 401(k) funds can be invested in. A 403(b) plan is technically not a qualified plan under the Employee Retirement Income Security Act of 1974 (ERISA).

LO 4.3.1

PREV

How well did you know this?
1
Not at all
2
3
4
5
Perfectly
155
Q

Which of the following statements is CORRECT in describing requirements that must be met for a plan to be considered a Section 457 plan?

I. To avoid adverse income tax effects, the agreement must be executed during the same month as the participant’s services are provided.

II. Eligible participants include employees of agencies, instrumentalities, and subdivisions of a state as well as Section 501 tax-exempt organizations.

III. The deferral limit for employees younger than age 50 is the lesser of $19,500 in 2021, or 100% of compensation.

A) II only

B) I and II

C) I and III

D) II and III

A

D) Explanation
All of these organizations may establish a Section 457 plan. Deferrals are limited to the lesser of $19,500 in 2021, or 100% of compensation. Statement I is incorrect because the agreement must be executed before the first day of the month in which the services will be performed. The result will be a loss of the tax deferral for that month if the agreement is made during the month.

LO 4.3.2

PREV

How well did you know this?
1
Not at all
2
3
4
5
Perfectly
156
Q

Dorothy is a 36-year-old jewelry designer who owns her own small gallery. For most of the year, she works alone, handling the designing and sales herself. However, during the busy holiday season, she hires Yvonne and Mirabelle, two part-time sales clerks to help her. Each of these employees works approximately 300 hours, earning an average salary of $4,000. Dorothy would like to establish a retirement plan that would allow her to save for her own retirement, but not require her to cover the part-time employees. She also doesn’t want to pay expensive administrative costs. Which one of the following plans would be most appropriate for Dorothy?

A) SIMPLE IRA

B) Traditional Section 401(k) plan

C) SEP plan

D) Section 457 plan

A

A)Explanation
A SEP plan would not be appropriate because it would require coverage of Dorothy’s part-time employees. A traditional Section 401(k) is not appropriate because it would involve special nondiscrimination testing and annual filing of the Form 5500 series. A Section 457 plan is not appropriate because it is available only for certain, private tax-exempt organizations. A SIMPLE IRA would be the most appropriate plan because it involves little administrative costs and would meet Dorothy’s retirement planning goals.

LO 4.2.1

PREV

How well did you know this?
1
Not at all
2
3
4
5
Perfectly
157
Q

Shock Limited just established a simplified employee pension (SEP) plan for the benefit of its employees. The company has more than 500 employees, 60% of whom are highly compensated. This year, Shock contributed 6% of each eligible employee’s salary to the SEP plan. Several of the employees of the company are unfamiliar with the provisions of SEP plans and have come to you requesting information. Which of the following statements regarding the basic provisions of SEP plans is CORRECT?

I. Employees can roll money that is distributed from a SEP plan into a different IRA within 60 days without withholding or penalty as long as it is not a required minimum distribution.

II. Contributions must be made for any employee, age 21 or over, who has performed services for the company in three of the past five years and has earned at least $650 during 2021.

III. Employer contributions are 100% vested immediately.

IV. A SEP plan may exclude members of unions if the unions have their own retirement plan.

A) I, II, and III

B) I, III, and IV

C) II, III, and IV

D) I, II, III, and IV

PREV

A

D) Explanation
All of the statements are correct.

LO 4.2.2

How well did you know this?
1
Not at all
2
3
4
5
Perfectly
158
Q

Which of the following statements regarding the disadvantages for employees participating in SEP plans is NOT correct?

A) If an employer maintains a SEP plan and a qualified plan, contributions to the SEP plan reduce the amount that may be deducted for contributions to the qualified plan.

B) The special rule for calculating deductible contributions on behalf of an owner-employee also applies to a SEP plan.

C) Employees cannot rely on a SEP plan alone to provide an adequate retirement benefit.

D) The employer bears the investment risk under the plan.

A

D) Explanation
The employee bears the investment risk under the plan, which is a disadvantage to the employee. All of the other choices are disadvantages of a SEP plan.

LO 4.2.2

PREV

How well did you know this?
1
Not at all
2
3
4
5
Perfectly
159
Q

Paul, age 64, is retiring next year. He participates in an eligible Section 457 plan through his governmental employer. His employer’s plan has a normal retirement age of 65. Which of the following statements regarding Paul’s governmental Section 457 plan is false?

A) Section 457 plan distributions are not eligible for net unrealized appreciation treatment.

B) A lump-sum distribution from a governmental Section 457 plan may be rolled over to a qualified plan.

C) Because he is over age 50 and within the last three years of employment before retirement, Paul can defer the maximum amount under a special catch-up provision and an additional amount under the over-age-50 catch-up provision this year.

D) During the last three years of employment before the plan’s normal retirement age, Paul’s elective deferral may be increased up to $39,000 (2021).

A

C)Explanation
A participant in a governmental Section 457 plan may not use both the special catch-up allowance and the over-age-50 catch-up allowance in the same tax year.

LO 4.3.2

PREV

How well did you know this?
1
Not at all
2
3
4
5
Perfectly
160
Q

Paul, age 64, is retiring next year. He participates in an eligible Section 457 plan through his governmental employer. His employer’s plan has a normal retirement age of 65. Which of the following statements regarding Paul’s governmental Section 457 plan is false?

A) Section 457 plan distributions are not eligible for net unrealized appreciation treatment.

B) A lump-sum distribution from a governmental Section 457 plan may be rolled over to a qualified plan.

C) Because he is over age 50 and within the last three years of employment before retirement, Paul can defer the maximum amount under a special catch-up provision and an additional amount under the over-

D) During the last three years of employment before the plan’s normal retirement age, Paul’s elective deferral may be increased up to $39,000 (2021).
Failed

A

C) Explanation
A participant in a governmental Section 457 plan may not use both the special catch-up allowance and the over-age-50 catch-up allowance in the same tax year.

LO 4.3.2

PREV

How well did you know this?
1
Not at all
2
3
4
5
Perfectly
161
Q

Tom, a sole proprietor, is interested in implementing a retirement plan. He may have employees in the future. He also wants a plan that is easily understood, where the employees bear the investment risk, does not favor older participants, and allows elective deferrals. Which of the following plans is best suited to Tom’s goals?

A) Target benefit pension plan

B) SIMPLE IRA

C) Age-weighted profit-sharing plan

D) SEP plan

A

B) Explanation
Tom’s goals describe attributes of a SIMPLE IRA. The SEP plan and the target benefit plan do not allow worker contributions. The age-weighted profit-sharing plan is not as easily understood. Also, the target benefit plan and the age-weighted profit-sharing plan favor older employees.

LO 4.2.1

PREV

How well did you know this?
1
Not at all
2
3
4
5
Perfectly
162
Q

A government employer would choose to establish a Section 457 plan for all of the following reasons except

A) income tax credit for certain taxpayers regarding elective contributions.

B) tax deductibility of contributions.

C) no early withdrawal penalty on distributions.

D) tax-deferred growth.

A

B) Explanation
Because Section 457 plans are sponsored by tax-exempt entities, deductibility of plan contributions is not an issue.

LO 4.3.2

PREV

How well did you know this?
1
Not at all
2
3
4
5
Perfectly
163
Q

Which of the following statements regarding SIMPLE IRA employer contributions is CORRECT?

I. A 2% (of employee compensation) nonelective employer contribution can be made for all eligible employees.

II. A dollar-for-dollar matching contribution up to 3% of compensation can be contributed solely for participating employees who have elected to make contributions.

III. The employer’s contribution must remain the same for each year the plan is maintained.

IV. The employer must annually communicate its contribution level to employees before the beginning of the employees’ 90-day election period.

A) III and IV

B) I and II

C) I, II, and III

D) I, II, III, and IV

A

B) Explanation
Statements I and II are correct. An employer can make a 2% nonelective employer contribution in a SIMPLE IRA, where employees eligible to participate receive an employer contribution equal to 2% of their covered compensation (limited to $290,000 in 2021 and subject to cost-of-living adjustments for later years), regardless of whether they make their own contributions. Alternatively, an employer may make a dollar-for-dollar match up to 3% of compensation, where only the participating employees who have elected to make contributions will receive an employer-matching contribution. Each year, the employer can choose which contribution level it will use for the following year’s contributions. The company is required to communicate this information to employees before the beginning of the 60-day election period.

LO 4.2.1

PREV

How well did you know this?
1
Not at all
2
3
4
5
Perfectly
164
Q

George has been participating in his employer’s SIMPLE IRA for one year. He is 45 years old. If he withdraws $1,000 from this plan this year and the withdrawal is not covered by an exception to the penalty tax on premature withdrawals, he will owe a penalty tax of

A) $0.

B) $500.

C) $250.

D) $100.

A

C) Explanation
George will incur a $250 penalty tax on the $1,000 withdrawal. Premature withdrawals made from a SIMPLE IRA within two years of initial participation are subject to a penalty tax of 25%.

LO 4.2.1

PREV

How well did you know this?
1
Not at all
2
3
4
5
Perfectly
165
Q

Bonnet Company has 50 employees, of which 40 earned at least $5,000 in the prior year and are expected to earn at least that much in the current year. Bonnet Company does not currently maintain a retirement plan. If Bonnet implements a SIMPLE IRA, which of the following statements is NOT correct?

A) Employees may make elective deferrals into the SIMPLE IRA as a percentage of compensation of up to at least $13,500 in 2021.

B) The effective covered compensation limit for workers younger than age 50 for SIMPLE IRAs is $450,000 for 2021 when the employer elects the 3% match.

C) The covered compensation limit is $290,000 if Bonnet Company decides on the 2% nonelective employer contribution.

D) All of the employees of Bonnet Company are eligible to participate in the plan.

PREV

A

D) D) Explanation
Only the employees of Bonnet Company who earned at least $5,000 in the prior year and expect to earn at least that amount in the current year are eligible to participate in the plan (40 employees). Employees may make an elective deferral into the SIMPLE IRA as a percentage of compensation of up to at least $13,500 in 2021. Those age 50 and above can add an additional $3,000. The unofficial, but mathematically set, effective covered compensation limit for SIMPLE IRAs for employees under age 50 is $450,000 in 2021 when the employer elects the 3% match ($13,500 divided by 0.03). The covered compensation limit is $290,000 if Bonnet Company decides on the 2% nonelective employer contribution.

LO 4.2.1

How well did you know this?
1
Not at all
2
3
4
5
Perfectly
166
Q

Sharon Bender, age 52, has been a teacher in the Lammer County School District for 18 years. Recently, she inherited a large sum of money and wants to minimize her income tax. What is her maximum 403(b) deferral in 2021?

A) $26,000

B) $29,000

C) $19,500

D) $58000

A

B) Explanation
Sharon can defer the basic $19,500 allowed in 2021, plus $6,500 for the age 50 catch-up and $3,000 for the long service catch-up (15 years of service or more).

LO 4.3.1

PREV

How well did you know this?
1
Not at all
2
3
4
5
Perfectly
167
Q

Which of the following statements is CORRECT in describing a Section 457 catch-up provision?

A) During the final three years before retirement, a participant in a Section 457 plan could contribute up to the lesser of 100% of compensation, or $58,000. The other catch-up is for those who have attained age 50. They can increase their deferrals by $6,500 (in 2021) in all but the last three years before retirement.

B) During the final three years before the plan document normal retirement date, an employee can double the usual age 50 catch-up of $6,500 in 2021.

C) The final three-year catch-up provision allows participants to make contributions up to twice the maximum deferral allowed for a 457 plan. The additional deferral amount is available only from prior unused deferrals, and is to make up for those years when deferrals were less than the maximum allowed. The other catch-up is for those who have attained age 50. They can increase their deferrals by $6,500 (in 2021) in all but the last three years before retirement if they use the final three-year catch-up.

D) The final three-year catch-up provision allows all participants of a Section 457 plan to contribute an additional $19,500 during the three years preceding retirement, regardless of previous contributions. The other catch-up is for those who have attained age 50. They can increase their deferrals by $6,500 (in 2021) in all but the last three years before retirement.

A

C) Explanation
The additional deferral amount is available only from prior unused deferrals and is to make up for those years when deferrals were less than the maximum allowed. The other catch-up is for those who have attained age 50. They can increase their deferrals by $6,500 (in 2021) in all but the last three years before retirement if they are using the final three-year catch-up.

LO 4.3.2

PREV

How well did you know this?
1
Not at all
2
3
4
5
Perfectly
168
Q

All of the following statements regarding the basic provisions of a Section 457 plan are correct except

A) the contribution limit is doubled in the 3 years before an individual’s retirement.

B) in 2020, an individual who has attained age 50 may make additional catch-up contributions of up to $6,500.

C) distributions from a Section 457 plan are not subject to an early withdrawal penalty.

D) a Section 457 plan is a qualified plan of governmental units or agencies, and non-church-controlled, tax-exempt organizations.

A

D) Explanation
A Section 457 plan is not a qualified plan. This plan is a deferred compensation plan that may be established by governmental units or agencies and non-church-controlled, tax-exempt organizations. Section 457 plans have special catch-up rules.

LO 6.3.1

PREV

How well did you know this?
1
Not at all
2
3
4
5
Perfectly
169
Q

Bland Foods, Inc., wants to establish a retirement benefit for the company’s executives that is separate from its qualified plan. The plan will be unfunded and pay benefits only as needed from the company’s assets. Bland Foods wants to provide the benefits without requiring the executives to reduce their current salary. Which of these types of plans was most likely chosen by Bland Foods, Inc.?

A) Stock bonus plan

B) Defined benefit pension plan

C) Top-hat plan/nonqualified plan

D) Section 401(k) plan

A

C) Explanation
A top-hat plan would meet all of the company’s objectives. The other choices are all qualified plans and would be subject to the limitations on funding, contributions, and benefits.

LO 4.1.1

PREV

How well did you know this?
1
Not at all
2
3
4
5
Perfectly
170
Q

Which of the following are characteristics that simplified employee pensions (SEPs) share with qualified profit sharing plans?

I. Limitation on employer contributions
II. Application of controlled group rules
III. Nondiscrimination requirements
IV. Statutory eligibility requirements (age 21, one year of service)

A) I, II, and III

B) II, III, and IV

C) I and II

D) II and IV

A

A) Explanation
SEPs and profit sharing plans are both subject to the 25% limit on deductible employer contributions, the controlled group rules, and nondiscrimination requirements. Eligibility requirements are different for a SEP.

LO 4.2.2

PREV

How well did you know this?
1
Not at all
2
3
4
5
Perfectly
171
Q

Christopher works for the Ex-March Company, a small business with 75 employees. Ex-March has decided to establish a SIMPLE IRA plan for all of its employees and will make a 2% nonelective contribution for each of its eligible employees. Christopher’s annual salary is $40,000, and he has determined that he cannot afford to make an elective deferral to his SIMPLE IRA. Which of the following statements regarding Christopher’s SIMPLE IRA contribution is CORRECT?

A) Ex-March Company must make a nonelective contribution of $800 for Christopher.

B) Ex-March is not required to make a contribution for Christopher.

C) Christopher must make a 2% minimum contribution this year.

D) Ex-March must make a nonelective contribution of $1,200 for Christopher.

A

A) Explanation
Ex-March must make a nonelective contribution of $800 for Christopher. Though Christopher does not make a contribution this year, Ex-March must make a nonelective contribution of $800 (2% × $40,000) on his behalf. Under the employer contribution election Ex-March has selected, even if an eligible employee does not contribute to the SIMPLE IRA, that employee would still receive an employer nonelective contribution to his SIMPLE IRA equal to 2% of compensation.

LO 4.2.1

PREV

How well did you know this?
1
Not at all
2
3
4
5
Perfectly
172
Q

Which of the following are reasons a small business might choose the SIMPLE over a Section 401(k) plan?

A) Because a SIMPLE is less costly to operate, it is generally the better choice if the employer is not concerned about the design constraints of the plan.

B) A Section 401(k) plan would be top heavy (benefits for key employees will exceed 60% of total benefits), and the employer wants to minimize employer contributions.

C) The employer expects that it could not satisfy the Section 401(k) nondiscrimination test.

D) All of these are reasons.

A

D) Explanation
All of these statements are reasons a small business might choose the SIMPLE over a Section 401(k) plan.

LO 4.2.1

How well did you know this?
1
Not at all
2
3
4
5
Perfectly
173
Q

Which of the following describes a basic provision of a SIMPLE IRA?

A) Only employers that average fewer than 20 employees can establish a SIMPLE IRA.

B) A SIMPLE IRA must satisfy both the ADP and ACP nondiscrimination tests.

C) An employer may add a SIMPLE IRA plan to an existing defined benefit plan to allow employees to make elective deferrals.

D) SIMPLE IRA plans can be arranged to allow for in-service loans for up to 50% of the account balance, but not to exceed $50,000.

E) One contribution formula an employer can use under a SIMPLE IRA is to make a 2% nonelective contribution on behalf of each eligible employee with at least $5,000 in current compensation.

A

E) Explanation
SIMPLE IRA plans are available to employers with 100 or fewer employees and with no other qualified retirement plan. The employer contribution requirement may be satisfied by either a 3% matching contribution formula or a 2% nonelective contribution for each employee with current-year compensation of $5,000 or more. IRA-funded employer-sponsored tax-advantaged plans may not incorporate loan provisions. These plans do not require ADP or ACP nondiscrimination tests.

LO 4.2.1

How well did you know this?
1
Not at all
2
3
4
5
Perfectly
174
Q

Tom, age 54, is the sole proprietor of a small business. He is interested in adopting a retirement plan for the business. His primary goals are to make large contributions to his own retirement account and to minimize the expense and paperwork associated with the plan. Which of the following retirement plans would you recommend for Tom’s business? (He makes $50,000 of self-employment income and has three part-time employees who earn $15,000 each.)

A) Section 401(k) plan funded by employee elective deferrals

B) SIMPLE IRA

C) SEP plan

D) Traditional defined benefit pension plan

A

B) Explanation
Qualified plans require a significant amount of expense and paperwork. Because Tom’s business is small, a SEP or SIMPLE IRA is preferred. Tom can, however, defer more in a SIMPLE than in a SEP. Based on 2021 plan contribution limits, in the SIMPLE, he can defer $16,500 ($13,500 salary deferral, plus $3,000 as a catch-up contribution) plus receive a 3% match. In a SEP, he would be limited to $9,293 as follows:

$50,000 Schedule C income
− 3,533 deductible half of self-employment tax (SECA)
$46,467 net Schedule C SE income
× 20% (the 20% is calculated by 0.25 ÷ 1.25 = 20%)
$9,293
LO 4.2.1

How well did you know this?
1
Not at all
2
3
4
5
Perfectly
175
Q

For which of the following plans does the employee bear the investment risk in the plan?

I. SIMPLE 401(k)
II. Traditional profit-sharing plan
III. SEP IRA
IV. SIMPLE IRA

A) I, II, III, and IV
B) II only
C) I, III, and IV
D) I and II

A

A) Explanation
The employee bears the investment risk for all of these plans.

LO 4.2.1

How well did you know this?
1
Not at all
2
3
4
5
Perfectly
176
Q

Gary was just hired by an employer that maintains a SIMPLE IRA for its employees. Which of these statements regarding Gary’s participation in the SIMPLE IRA is CORRECT?

A) Gary may only defer $6,000 into the SIMPLE IRA if he is younger than age 50.

B) Covered compensation is limited to $290,000 for a SIMPLE IRA in 2021 if the employer elects a 3% match.

C) When Gary participates in the plan, he will be 100% vested in his employer’s contributions.

D) The annual employer match may be limited to 1% of employee compensation each year.

A

C) Explanation
Gary will be 100% vested in his employer contributions to his account. He may defer up to $13,500 (2021) into the SIMPLE IRA if he is younger than age 50. An employer may only limit the matching contribution to 1% of employee compensation in no more than two out of every five years. Effective covered compensation for employees under age 50 is limited to $450,000 ($13,500 ÷ 0.03) for a SIMPLE IRA (2021) in which the employer elects to make 3% matching contributions. The covered compensation limit of $290,000 (2021) does not apply in this instance because there is a special rule for SIMPLE IRAs.

LO 4.2.1

How well did you know this?
1
Not at all
2
3
4
5
Perfectly
177
Q

Max, age 47, has been participating in his employer’s SIMPLE IRA for one year. If he withdraws $1,000 from this plan this year and the withdrawal is not covered by an exception to the penalty tax on premature withdrawals, he will owe a penalty tax of

A) $400.
B) $250.
C) $0.
D) $100.

A

B) Explanation
Premature withdrawals made from a SIMPLE IRA within two years of initial participation are subject to a penalty tax of 25%. Max’s penalty would be $250.

LO 4.2.1

How well did you know this?
1
Not at all
2
3
4
5
Perfectly
178
Q

Which of the following regarding a SIMPLE is CORRECT?

I. A SIMPLE requires ADP testing of employee elective deferral contributions.

II. SIMPLE IRAs are subject to top-heavy rules.

III. A 25% early withdrawal penalty may apply to distributions taken within the first two years of participation in the SIMPLE IRA plan.

IV. The maximum annual elective deferral contribution to a SIMPLE is $19,500 (2021) for an employee who has not attained age 50.

A) I, II, and IV
B) III only
C) I, II, III, and IV
D) II and III

A

B) Explanation
Neither a SIMPLE IRA nor a SIMPLE 401(k) requires ADP testing. A SIMPLE is not subject to top-heavy rules. Statement III is correct. The early withdrawal penalty is 25% for distributions taken within the first two years of participation in a SIMPLE IRA. The maximum elective deferral to a SIMPLE is $13,500 (2021). Employees who have attained age 50 by the end of the tax year will also be eligible for a catch-up contribution of $3,000.

LO 4.2.1

How well did you know this?
1
Not at all
2
3
4
5
Perfectly
179
Q

A savings incentive match plan for employees (SIMPLE) can be which of these?

I. It can be established as an IRA.
II. It can be established as a Section 401(k) plan.
III. It can be established as a Section 403(b) plan.
IV. It can be offered by employers who have 100 or fewer employees.

A) I and II
B) I, II, and IV
C) II and IV
D) I, II, and III

A

B) Explanation
Employers with more than 100 employees (who earn more than $5,000) may not offer a SIMPLE. In addition, the employer may not usually offer any other type of qualified retirement plan in conjunction with a SIMPLE. Union plans and governmental 457 plans are the exceptions to the employer having a second active plan. However, this is not to be applied to a test question unless specifically mentioned in the question. Basically, can a SIMPLE have a second active plan for its workers? No, because that is as far as the test is going. What about a union plan or a 457 for qualified state and local government employers? Well, okay, for those rare exceptions, but the question would have to bring up one of these rare exceptions. The point is that SIMPLEs were intended to be the only active plan for the small employer. Congress was afraid the employer would try to be discriminatory with the other retirement plan, so the rules was made that there could not be another retirement plan. Then governmental 457 plans and union plans snuck in, but these rare exceptions are not a part of the question unless specifically mentioned.

LO 4.2.1

How well did you know this?
1
Not at all
2
3
4
5
Perfectly
180
Q

Margaret is a 29-year-old attorney with her own law practice. She has hired four part-time employees over the past five years to assist her. Each of these employees works approximately 200 hours per year, earning an average annual salary of $4,000. Margaret would like to establish a retirement plan that would allow her to begin saving for her own retirement, with little administrative costs. Which one of the following plans would be most appropriate for Margaret in 2021?

A) SIMPLE IRA
B) Traditional Section 401(k) plan
C) SEP plan
D) Section 457 plan

A

A) Explanation
A SIMPLE IRA would be the most appropriate plan because it involves little administrative costs and would meet Margaret’s retirement plan goals. Notice the workers are making more than $650 and less than $5,000. If a SEP would be chosen, she would have to contribute for employees who made more than $650; are at least age 21; and who have worked for her for three of the preceding five years.

LO 4.2.1

How well did you know this?
1
Not at all
2
3
4
5
Perfectly
181
Q

Which of the following statements describes a basic provision or use of a savings incentive match plan for a SIMPLE IRA?

A) A SIMPLE IRA must satisfy special nondiscrimination tests in addition to general rules.

B) A SIMPLE IRA is primarily suitable for large, corporate-type employers.

C) Only employers that average fewer than 200 employees can establish a SIMPLE IRA.

D) One contribution formula an employer can use in a SIMPLE IRA is to make a 2% nonelective contribution on behalf of eligible employees.

A

D) Explanation
A 2% nonelective contribution formula on behalf of eligible employees is one of two formulas an employer may use in making contributions to a SIMPLE IRA. A 3% match is the other. For a SIMPLE IRA only, the 3% match can be lowered to 1% for two out of five years.

LO 4.2.1

How well did you know this?
1
Not at all
2
3
4
5
Perfectly
182
Q

Which one of the following requirements is a possible disadvantage of a simplified employee pension (SEP) for an employer?

A) The vesting requirements for a SEP prohibit forfeitures.

B) Employer contributions to a SEP are subject to payroll taxes.

C) The SEP’s trustee is subject to ERISA’s prohibited transaction excise tax penalties.

D) A SEP must have a fixed contribution formula that is nondiscriminatory.

A

A) Explanation
SEP contributions must be 100% vested (i.e., nonforfeitable). SEPs consist of individual IRAs; there is no trustee for a SEP plan. The contribution formula of a SEP is not required to be fixed. Employer contributions to a SEP are not subject to payroll taxes.

LO 4.2.2

How well did you know this?
1
Not at all
2
3
4
5
Perfectly
183
Q

Which of the following statements regarding the disadvantages to the employer of SEP plans is CORRECT?

I. Employees cannot rely on a SEP plan alone to provide an adequate retirement benefit, which may hinder appreciation of the plan by employees.

II. The employer bears the investment risk under the plan.

III. If an employer maintains a SEP plan and a qualified plan, contributions to the SEP plan reduce the amount that may be deducted for contributions to the qualified plan.

IV. The special rule for calculating deductible contributions on behalf of an owner-participant in a qualified plan also applies to a SEP plan.

A) III only
B) II only
C) I, III, and IV
D) I, II, III, and IV

A

C) Explanation
Statements I, III, and IV are disadvantages of a SEP plan to an employer. Statement II is false. The employee bears the investment risk under the plan.

LO 4.2.2

How well did you know this?
1
Not at all
2
3
4
5
Perfectly
184
Q

How does simplified employee pension (SEP) plan participation affect an employee’s IRA contributions?

I. The deductibility of an active participant’s IRA contribution depends upon his MAGI.

II. SEP plan participation does not reduce or eliminate an employee’s ability to fund an IRA.

III. Employees who participate in a SEP plan are considered active participants in an employer-sponsored retirement plan for the tax year in which an employer contribution is made.

IV. Employees who participate in a SEP plan are not considered active participants in an employer retirement plan for the tax year in which an employer contribution is made.

A) I, II, and III
B) I and II
C) II and III
D) I, II, and IV

A

A) Explanation
Statements I, II, and III are correct. Employees who participate in a SEP plan are considered active participants in an employer retirement plan for the tax year in which an employer contribution is made. The deductibility of an active participant’s IRA contribution depends upon his MAGI and can be phased out or eliminated at certain income levels. SEP plan participation does not reduce or eliminate an employee’s ability to fund an IRA. The IRA can be funded, but not necessarily deducted from gross income.

LO 4.2.2

How well did you know this?
1
Not at all
2
3
4
5
Perfectly
185
Q

Which of the following statements with respect to simplified employee pension (SEP) contributions made by an employer is CORRECT?

A) Contributions are currently excludible from the employee’s gross income.
B) Contributions are capped at $19,500 for 2021.
C) Contributions are subject to income tax withholding.
D) Contributions are subject to FICA and FUTA.

A

A) Explanation
Contributions are currently excludible from the employee’s gross income. Employer contributions to a SEP are not subject to FICA, FUTA, or income tax withholding. The SEP contribution limit for 2021 is the lesser of 25% of covered compensation or $58,000.

LO 4.2.2

How well did you know this?
1
Not at all
2
3
4
5
Perfectly
186
Q

George is a plumbing contractor and has implemented a retirement plan for his employees. The plan must cover all employees who are at least age 21 and have worked for George for three of the last five years (including part-time employees). Contributions must be made for employees who earned at least $600 in the prior year. The plan can exclude union members if they have their own retirement plan. Which type of plan has George selected?

A) Section 403(b) plan
B) Simplified employee pension (SEP) plan
C) SARSEP plan
D) SIMPLE IRA

A

B) Explanation
The requirement that contributions must be made for employees who earned at least $600 in the prior years identifies the plan as a SEP plan.

LO 4.2.2

How well did you know this?
1
Not at all
2
3
4
5
Perfectly
187
Q

All of the following statements regarding simplified employee pension (SEP) plans are correct except

A) all part-time employees can be excluded.
B) SEP plans can be established by any form of business entity.
C) the major advantage is the simplicity of the plan.
D) employer contributions are discretionary.

A

A) Explanation
A SEP plan must cover any employee who is at least 21 years old and who worked for the employer, even on a part-time basis, for three of the preceding five years. In addition, contributions must be made on behalf of any eligible employee whose compensation for the year is at least $650 (2021). These provisions make most part-time employees eligible to participate.

LO 4.2.2

How well did you know this?
1
Not at all
2
3
4
5
Perfectly
188
Q

Which of the following types of plans is typically used by religious, charitable, educational, and other Section 501(c)(3) entities or public school systems?

A) SIMPLE
B) Section 403(b) plan/TSA
C) Section 457 plan
D) SEP plan

A

B) Explanation
A Section 403(b) plan/TSA (tax-sheltered annuity) is a tax-deferred employee retirement plan that is adopted only by certain tax-exempt organizations and certain public schools and colleges. Employees have individual accounts to which employers contribute (or employees contribute through salary reductions).

LO 4.3.1

How well did you know this?
1
Not at all
2
3
4
5
Perfectly
189
Q

Which of the following statements regarding Section 403(b) plans is CORRECT?

I. Section 403(b) plans must comply with many of the same reporting and auditing requirements that apply to 
Section 401(k) plans.

II. Certain eligible participants in a Section 403(b) plan may defer as much as much as $29,000 into the plan in 2021.

III. Section 403(b) plans may provide for plan loans to participants.

IV. Funding for Section 403(b) plans is limited to mutual funds and annuities.

A) I and II
B) III and IV
C) I, II, and III
D) I, II, III, and IV

A

D) Explanation
All of the statements are correct. The $29,000 maximum elective deferral for 2021 includes the $19,500 basic limit; the extra $3,000 per year for up to $15,000 lifetime for employees of not-for-profit health care, education, and church employers; and the $6,500 catch-up for those age 50 and older.

LO 4.3.1

How well did you know this?
1
Not at all
2
3
4
5
Perfectly
190
Q

Which of the following statements regarding the basic provisions of tax-sheltered annuities (TSAs)/Section 403(b) plans is NOT correct?

A) An eligible employee may be able to use both a special catch-up provision and an over-age-50 catch-up provision in the same year.

B) TSAs are available to all eligible employees of Section 501(c)(3) organizations who adopt such a plan.

C) If an employee has at least 15 years of service with an eligible employer, an additional catch-up contribution may be permitted.

D) The special catch-up provision for eligible Section 403(b) participants allows up to a $39,000 (2 × $19,500 in 2021) elective deferral in the last 3 years of employment before retirement.

A

D) Explanation
The special catch-up provision for eligible Section 403(b) participants allows a maximum additional $3,000 per year elective deferral until a lifetime maximum catch-up is reached. The ability to double the worker contribution in the last three years before the retirement plan document’s normal retirement age applies to governmental 457 plans.

LO 4.3.1

How well did you know this?
1
Not at all
2
3
4
5
Perfectly
191
Q

Which of the following statements regarding TSAs/Section 403(b) plans is CORRECT?

I. The sponsor must be a tax-exempt organization that meets the requirements of Section 501(c)(3), a governmental organization, or public educational organization.

II. Lump-sum distributions may be eligible for special long-term capital gain treatment.

III. In-service withdrawals may be permitted.

IV. The plan can invest in individual stocks and bonds, but not options or futures.

A) I and III
B) I, II, and III
C) II, III, and IV
D) I, III, and IV

A

A) Explanation
Statement II is incorrect because distributions from a TSA are treated as ordinary income. Statement IV is incorrect. Only annuities and mutual funds are permitted investments in a TSA.

LO 4.3.1

How well did you know this?
1
Not at all
2
3
4
5
Perfectly
192
Q

For tax-exempt employers that do not want to implement a Section 457 plan and desire a plan funded strictly by employee elective deferrals, a good alternative would be

A) a SEP plan.
B) a Section 403(b) plan.
C) a profit-sharing plan.
D) a SARSEP plan.

A

B) Explanation
The Section 403(b) plan, like the Section 457 plan, can be used as an employee-deferred contribution plan. Certain tax-exempt employers can implement Section 403(b) plans. With a SEP plan or a profit-sharing plan, there are also employer contribution considerations. New SARSEP plans can no longer be established.

LO 4.3.1

How well did you know this?
1
Not at all
2
3
4
5
Perfectly
193
Q

Which of the following statements regarding the basic provisions of Section 403(b) plans is CORRECT?

I. A special catch-up provision is available to employees of Section 501(c)(3) organization employers who have at least 1 year of service.

II. An eligible employee may be able to use both a special catch-up provision and an over-age-50 catch-up provision in the same year.

III. TSAs are available to all eligible employees of Section 501(c)(3) organizations who adopt such a plan.

IV. If an employee has at least 15 years of service with an eligible employer, an additional catch-up contribution may be permitted.

A) I and II
B) II only
C) II, III, and IV
D) I, II, and III

A

C) Explanation
Statement I is incorrect. The special or additional catch-up provision requires at least 15 service years.

LO 4.3.1

How well did you know this?
1
Not at all
2
3
4
5
Perfectly
194
Q

Which of the following are permitted investments in tax-sheltered annuity (TSA) plans?

I. Individual stocks
II. Group or individual annuity contracts
III. Custodial accounts invested in mutual funds

A) I and II
B) I only
C) II and III
D) I, II, and III

A

C) Explanation
Investments in TSAs are limited to annuity contracts, either group or individual, and custodial accounts invested in mutual funds. Individual stocks and bonds are not permitted.

LO 4.3.1

How well did you know this?
1
Not at all
2
3
4
5
Perfectly
195
Q

Ronald, age 44, works for two private tax-exempt employers. One has a Section 403(b) plan and one maintains a nongovernmental Section 457 plan. If Ronald defers $10,000 into the Section 403(b) plan in 2021, what is the maximum amount he may defer into the Section 457 plan?

A) $19,500
B) $13,500
C) $9,000
D) $48,000

A

A) Explanation
Ronald can separately defer the maximum of $19,500 (2021) into the Section 457 plan because Section 457 plan limits are not aggregated with the Section 403(b) plan limits.

LO 4.3.2

How well did you know this?
1
Not at all
2
3
4
5
Perfectly
196
Q

Which of the following retirement plans, maintained by an employer, would also permit an eligible employer to establish a SIMPLE?

A) Section 401(k) plan
B) Section 457 plan
C) Section 403(b) plan
D) SEP plan

A

B) Explanation
To establish a SIMPLE, an employer cannot maintain another plan. However, a Section 457 plan is a nonqualified deferred compensation plan; therefore, it does not constitute a plan for purposes of establishing a SIMPLE.

LO 4.2.1

How well did you know this?
1
Not at all
2
3
4
5
Perfectly
197
Q

Which of the following retirement plans, maintained by an eligible employer, would also permit the employer to establish a SIMPLE IRA?

A) Money purchase pension plan
B) Traditional Section 401(k) plan
C) Section 403(b) plan
D) Union plan bargained in good faith

A

D) Explanation
To establish a SIMPLE (IRA or Section 401(k)), an employer cannot generally maintain another retirement plan. However, a plan bargained in good faith with a union is allowed because Congress is more worried about powerful businessowners using a retirement plan in a discriminatory manner.

LO 4.3.2

198
Q

Which of the following statements regarding TSAs and Section 457 plans is CORRECT?

I. Both plans may be funded entirely by participant contributions.

II. Participation in either a TSA or a Section 457 plan will cause an individual to be considered an active participant for purposes of phasing out the deductibility of traditional IRA contributions.

III. Both plans allow net unrealized appreciation tax treatment for lump-sum distributions.

IV. Both plans must meet minimum distribution requirements that apply to qualified plans.

A) II, III, and IV
B) I and IV
C) I only
D) I, III, and IV

A

B) Explanation
Statements I and IV are correct. Statement II is incorrect because a Section 457 plan is a deferred compensation arrangement that will not cause a participant to be considered an active participant. Statement III is incorrect because net unrealized appreciation (NUA) tax treatment is not permitted for distributions from either plan.

LO 4.3.2

199
Q

David is age 47 and qualifies for a Section 457 plan through his job with the state. David’s salary is $37,500. What is the maximum salary deferral that David can contribute to the Section 457 plan in 2021?

A) $6,500
B) $13,500
C) $19,500
D) $9,375

A

C) Explanation
The dollar limit applicable to Section 403(b), 457, and 401(k) plans is $19,500 in 2021.

LO 4.3.2

200
Q

Which one of the following types of employers can offer either a 403(b) plan or a 457(b) plan?

A) Any employer that can offer a Section 403(b) plan could also sponsor a Section 457(b) plan.

B) A state university or college could offer either or both a Section 403(b) and a Section 457(b) plan.

C) Churches and qualified church-controlled organizations may offer either or both a Section 403(b) and a Section 457(b) plan.

D) Any employer with less than 1,000 employees can offer either or both a Section 403(b) and a Section 457(b) plan.

A

B) Explanation
State and local governments and organizations exempt from federal income tax are eligible to offer Section 457 deferred compensation plans, but churches and church-controlled organizations cannot. No for-profit organizations can offer either of these plans.

LO 4.3.2

201
Q

All of the following retirement plans permit employees to make elective deferrals except

A) profit-sharing plans with Section 401(k) provisions.
B) SEP plans.
C) SIMPLE 401(k)s.
D) Section 401(k) plans.

A

B) Explanation
Employee elective deferrals are permitted in Section 401(k) plans, profit-sharing plans with Section 401(k) provisions, and SIMPLEs, but not in SEP plans.

LO 4.2.2

202
Q

Which of these statements regarding the Section 72(t) early distribution penalty is false?

A) A 10% penalty is imposed on the taxable amount of a distribution made to a participant that has not yet attained age 59½, unless a specific exception applies. Taxpayers must include the appropriate amount of the distribution in their taxable income.

B) The 10% tax usually applies only to the taxable portion of the distribution. If a distribution includes amounts that have been previously subject to tax, such as after-tax employee contributions, the nontaxable portion of the distribution is exempt from the 10% penalty tax.

C) The 10% penalty applies to distributions that are made from a qualified plan, a Section 403(b) plan, a traditional IRA, or a simplified employee pension (SEP) plan.

D) The tax does not apply to any distribution from a Roth IRA.

A

D) Explanation
The early distribution penalty does apply to certain nonqualified distributions from a Roth IRA. All other statements are correct.

LO 5.4.2

203
Q

Which of the following is NOT subject to 20% mandatory federal income tax withholding if a rollover is not a direct transfer?

IRA rollovers
Section 403(b) plan/tax-sheltered annuity (TSA) rollovers
Money purchase pension plan rollovers
Profit-sharing plan rollovers
A) I only

B)
I, II, III, and IV

C)
II, III, and IV

D)
I, II, and III

A

A) Explanation
Of those listed, only IRAs are not subject to 20% withholding (regardless of the type of rollover). In contrast, all qualified plan and Section 403(b) plan distributions require withholding in the absence of a direct trustee-to-trustee transfer. SEP plans, while not listed, are a type of IRA and thus do not require 20% withholding.

LO 5.3.1

204
Q

Based on IRS regulations, the minimum distribution rules for IRAs and qualified retirement plans do what?

They impose a 10% excise tax on the amount by which a distribution falls short of the required minimum distribution (RMD).
They have a required beginning date of age 59½.
They make it easier to calculate the minimum distribution amount based on the participant’s life expectancy.
They require the participant to determine a beneficiary by the required minimum distribution (RMD) beginning date.

A)
III only

B)
I and II

C)
I, II, and III

D)
I and IV

A

A)Explanation
Based on IRS regulations, minimum distribution rules have been simplified by providing a uniform table that can be used by all participants (except when a spouse is more than 10 years younger) to determine the minimum distribution required during their lifetimes. This makes it easier to calculate the RMD because participants would neither need to determine their beneficiary by the required minimum distribution beginning date, nor would they have to decide whether to recalculate their life expectancies each year. The excise tax is 50%, not 10%. The required beginning date is April 1 of the year after the year the account owner turns age 72 for IRAs and qualified plan participant owners of 5% or more of the company. The required beginning date for Section 403(b) plans, Section 457 plans, and qualified plans is April 1 of the year after the employee retires. It is advisable for the owner of an IRA or qualified plan account to name an individual as beneficiary, but there is no requirement that such a beneficiary be determined before the owner attains age 72.

LO 5.4.2

205
Q

Which of the following statements is CORRECT regarding a nondeductible IRA?

A)
If a person is an active participant, qualification for contributions to nondeductible IRAs would depend upon AGI and filing status of the taxpayer.
Failed
B)
A person who is eligible to deduct an IRA contribution may choose to make a nondeductible contribution instead.
Achieved
C)
To qualify for a nondeductible IRA, a person’s AGI must be below a specified amount. If the AGI is within the phaseout range, they may make a partial contribution. If the AGI is above certain limits, which vary depending upon the filing status of the taxpayer, contributions to a nondeductible IRA are prohibited.
Failed

A

B) Explanation
A person may always choose not to deduct her IRA contribution, regardless of AGI or whether or not the person is an active participant.

LO 5.1.2

206
Q

Clara, age 50, opened a Roth IRA in Year 1 and made a $5,000 contribution for that year. In Years 2, 3, and 4, she made additional annual contributions of $5,000. Clara died in Year 4 after making her contribution for that year. The beneficiary is her son, Josh, who has asked his financial planner when he may take a total distribution from Clara’s Roth IRA penalty free and tax free. What should the planner tell him?

A)
Josh may roll over Clara’s Roth IRA to his own Roth IRA and is not required to take distributions during his lifetime.

B)
Because the distribution is made to Josh as a beneficiary upon Clara’s death, the distribution can be taken immediately without incurring a penalty or income taxes.

C)
The required five-year holding period that began in Year 1 with Clara’s first contribution to the Roth IRA must be satisfied for a distribution to be penalty free and tax free.

D)
A new five-year holding period began with the year of Clara’s death (Year 4), and Josh must wait until January 1 of Year 9 for a tax-free and penalty-free distribution.

A

C) Explanation
Any distribution must satisfy the five-year holding period that began in Year 1 with Clara’s first contribution to the Roth IRA to be tax free and penalty free. A nonspouse beneficiary cannot roll over the inherited Roth IRA into his own Roth IRA and forego the required distributions.

LO 5.4.2

207
Q

Which of the following are CORRECT statements pertaining to IRA rollover requirements?

I. If an IRA account is distributed directly to the IRA participant, the original IRA custodian must withhold 20% from the proceeds.

II. An IRA rollover must be completed within 60 days following the distribution date.

III An IRA account may be rolled over to another IRA once a year.

IV Direct trustee-to-trustee IRA transfers may be made as often as the IRA owner wishes.

A) I, II, and III

B) II and IV

C) I, III, and IV

D) II, III, and IV

A

D) Explanation
Once a year, an IRA owner may roll over his account to a new IRA within 60 days after distribution from the original IRA. Direct trustee-to-trustee transfers may be made as often as desired. The 20% withholding requirement does not apply to IRA distributions; the 20% mandatory withholding applies to eligible rollover distributions made directly to the plan participant from employer-sponsored qualified retirement plans or TSAs.

LO 5.2.1

208
Q

Which one of the following reasons for withdrawal would NOT allow the participant to avoid the IRS 10% early withdrawal penalty?

A) Certain medical expenses

B) Disability

C) The purchase of an automobile

D) Death

A

C) Explanation
Money withdrawn to purchase an automobile would be subject to the 10% early withdrawal penalty. If a participant dies or becomes disabled, there is no penalty for an early withdrawal. Distributions for certain serious medical expenses allow the participant to avoid the 10% early withdrawal penalty. Generally, the expenses that avoid the penalty are those that exceed the threshold of 10% of a participant’s adjusted gross income for individuals who are under age 59½. A withdrawal for any medical or any other expenses for those over age 59½ would not be subject to the early withdrawal penalty because the person is over 59½ anyway.

LO 5.3.1

209
Q

Tom’s ex-spouse, Dorinda, wants to save for her retirement years. What is the amount Dorinda can contribute to a Roth IRA for 2021? Her only income is $24,000 in child support and $3,600 of alimony from her 2018 divorce.

A) $3,600

B) $0

C) $7,000

D) $6,000

A

A) Explanation
Of the money Dorinda receives, only $3,600—the portion of the payments that can be attributed to alimony from a pre-2019 divorce settlement—qualifies as earned income for the purposes of a Roth IRA contribution. The definition for earned income is the same as for a traditional IRA. Please note that divorces finalized during and after 2019 will fall under the new rules, which will not include alimony as tax deductible to the payer and taxable to the receiver. As such, this will not be considered earned income. This divorce happened before this Tax Reform Act, and thus follows the old rules.

LO 5.4.1

210
Q

Ted and Margaret, ages 40 and 38, plan to contribute $12,000 to their IRAs for the current tax year. They are both employed and file a joint tax return. Ted is an active participant in his employer’s qualified retirement plan, but Margaret is not eligible for her employer’s plan. Their MAGI for the current tax year is $112,000. What amount, if any, can they deduct for their IRA contributions?

A) $9,900

B) $0

C) $6,000 (Margaret’s contribution only)

D) $12,000

A

A)Explanation
Since Ted is an “active participant” and their AGI falls within the phaseout range for married joint filers ($105,000–$125,000), their $12,000 IRA contribution will be only partially deductible. The upper AGI limit of $125,000 minus their AGI of $112,000 equals $13,000. $13,000 ÷ $20,000 = .65. And .65 × $6,000 (the maximum contribution) = $3,900 for Ted + $6,000 for Margaret’s spousal IRA. For test taking purposes, there is no need to actually calculate the number for this question. They are in the phaseout range for active participants, but only one spouse is an active participant. Thus

the nonactive participant spouse can deduct the full amount, but the active participant spouse cannot do the full amount; and
because of this, the answer must be higher than $6,000 and lower than $12,000 because they are both younger than 50 in 2021. There is only one answer in this range, so it must be the correct answer.
If there were two or more answers within the range, the exact number must be calculated.

Finally, if a person is not allowed to deduct an IRA contribution, the next option would be to contribute to a Roth IRA if income allows. Why would anyone want to make a nondeductible IRA contribution if they were eligible to make a Roth IRA contribution?

LO 5.1.2

211
Q

Which one of the following is subject to the 10% penalty tax on premature distributions from an IRA?

A) A series of substantially equal periodic payments

B) A distribution to a 55-year-old employee following separation from service

C) A distribution following the owner’s death

D) A distribution following disability

A

B) Explanation
This type of distribution is subject to the 10% penalty for an IRA. However, the exemption from the premature distribution penalty does apply to qualified plan distributions.

LO 5.3.1

212
Q

John has been contributing to his IRA for 17 years because his former employer did not offer a retirement plan. If his new employer offers a tax-sheltered annuity (TSA), simplified employee pension (SEP), or 457 plan, into which of these could he potentially roll his IRA?

Another IRA
His TSA
A SEP
A 457 plan

A) I only

B) I, II, III, and IV

C) II and III

D) I and II

A

B) Explanation
An IRA may be rolled to another IRA, TSA, SEP, qualified plan, or governmental 457 plan that accounts for such rollovers separately. The reason a governmental 457 would need to account for an IRA rollover separately is that a 457 plan is not normally subject to the 10% EWP. However, a distribution of money from a 457 plan that came from an account that was subject to the 10% EWP would be subject to the EWP.

LO 5.2.1

213
Q

Grant, age 51 today, made an initial contribution of $10,000 to a Roth 401(k) during 2013. He made subsequent contributions of $6,000 annually for the next four years. In 2021, Grant took a $50,000 distribution from his Roth 401(k) to purchase a boat. There were no deductible employer or employee contributions to his 401(k). Which of these statements regarding this distribution is CORRECT?

A)
It is partially taxable because Grant was not age 59½, disabled, the distribution was not made to a beneficiary or Grant’s estate after his death, or used for a first-time home purchase.

B)
It is taxable because it was within 10 taxable years from the date of initial contribution.

C)
It is income tax free because it was made after 5 taxable years from the date of initial contribution.

D)
It is income tax free because it was made after 5 taxable years, and Grant is over age 50.

A

A)
Explanation
Although Grant took the distribution after five taxable years from the date of initial contribution, he did not meet one of the other requirements for a qualified distribution (made after the individual attained age 59½, attributed to being disabled, made to a beneficiary or estate of an individual on or after the individual’s death, or used for a first-time home purchase). In this case, the first $34,000 is counted against his contributions. Thus, there is no tax or penalty on $34,000. There are no conversions, so the rest of the distribution is earnings and thus subject to income tax and the 10% early distribution rules.

LO 5.4.2

214
Q

\Which of the following statements is CORRECT regarding penalties related to IRAs?

I. For contributions in excess of the annual limits, the IRS imposes a penalty of 10% of the excess amount until the excess is withdrawn.

II. One of the exceptions from the 10% early withdrawal penalty (pre-59½) is a distribution to fund a first-time home purchase (subject to a lifetime maximum of $10,000). Income taxes still apply.

III. The 10% early withdrawal penalty does not apply if withdrawals are made to pay for qualified higher education expenses.

IV. Failure to begin receiving distributions by December 31 of the year following the year the taxpayer attains age 72 will result in a penalty tax of 50%.

A) I, II, III, and IV

B) II and III

C) I only

D) II and IV

A

B) Explanation
The penalty for excess contributions to an IRA is 6%. The 10% penalty for distributions before age 59½ does not apply to withdrawals made for qualified higher education expenses or first-time home purchase ($10,000 lifetime max). The required beginning date for minimum distributions is April 1 of the following year following the year in which the account holder attains age 72, not December 31.

LO 5.3.1

215
Q

Distributions from a Roth IRA are qualified distributions if the five-year holding period has been met as well as what else?

I. If the distributions are due to the owner’s death or disability

II. If the distributions are used for buying, building, or rebuilding a first-time home (up to $10,000 lifetime maximum)

III. If the owner has attained the age of 59½

IV. If the distributions are used for college tuition costs

A) I, II, III, and IV

B) I, II, and III

C) II and III

D) I and IV

A

B)Explanation
Only Statement IV is incorrect. A qualified distribution from a Roth IRA is a distribution after the five-year holding period has been met and the distribution satisfies one of the following four requirements: attainment of the age of 59½; death; disability; or first-time home purchase up to $10,000. College tuition is not a qualified special purpose distribution. However, it is an exception for the 10% early withdrawal penalty. Thus, with a Roth distribution for college costs, only the earnings would be subject to income tax after all contribution and conversion amounts were withdrawn.

LO 5.4.2

216
Q

All of the following are considerations for converting distributions from qualified plans or a traditional IRA to a Roth IRA except

A) one advantage of a conversion to the Roth IRA is that the Roth IRA will not be subject to required minimum distributions (RMD) during the life of the original owner.

B) the Roth IRA conversion is more appropriate when the income tax rate is lower at the time of distribution than at the time of conversion.

C) any portion of an IRA can be converted to a Roth IRA.

D) the Roth IRA conversion becomes more and more appropriate the longer the period of distributions.

A

B) Explanation
The Roth IRA conversion is more appropriate when the income tax rate is the same or higher at the time of distribution than at the time of conversion.

LO 5.4.3

217
Q

Nicholas is a 45-year-old single taxpayer with MAGI of $90,000 in 2021. Nicholas participates in his employer’s Section 457 plan and has deferred $19,500 of his salary to the plan this year. How much of a tax-deductible traditional IRA contribution can Nicholas make this year?

A)$2,500

B)$5,000

C)$6,000

D)$0

A

C) Explanation
A participant in a Section 457 plan is not considered an active participant for IRA tax deductibility purposes. Because Nicholas is not an active participant, he can make a fully deductible $6,000 contribution to a traditional IRA in 2021. If Nicholas were an active participant in a qualified plan, SEP, SARSEP, SIMPLE, or Section 403(b) plan, his deduction would be fully phased out at his MAGI level. However, his MAGI would still be low enough to allow contributions to a Roth IRA. Contributions to a Roth IRA begin to be phased out at $125,000 for unmarried individuals in 2021.

LO 5.1.2

218
Q

A financial planner’s client has an IRA with a balance of $140,000 as of January 1. On April 15 of the same year, the client withdraws the entire amount from the IRA and places it in a non-IRA CD for 60 days, earning 9% interest. On the 60th day, the client promptly and timely reinvests the principal of the CD in an IRA containing an aggressive growth fund. On September 15 of the same year, the client becomes dissatisfied with the return and the variability of the investment. The client wants a less risky investment and wants assurance that any IRA distribution will not be taxed at the time of the change. Which of these are acceptable alternatives for the client?

I. Withdraw the funds and reinvest them within 60 days in an IRA that invests exclusively in Treasury instruments.

II. Direct the trustee of the IRA to transfer the funds to another IRA that invests exclusively in Treasury instruments.

III. Withdraw the funds and reinvest within 60 days in an IRA that is an index mutual fund holding common stocks with portfolio risk equal to the S&P 500.

A) II only

B) II and III

C) I, II, and III

D) I only

A

A) Explanation
The client has already made a nondirect withdrawal (60-day rollover) this year; therefore, Statement I is incorrect, because he is not permitted to have another 60-day rollover during the same year. Statement II is a permitted alternative. Statement III is not permitted without the proceeds being included in taxable income.

LO 5.3.1

219
Q

Teresa, 37, recently terminated her employment with Applied Dynamicals Inc. The company had no qualified plan, but she has a Roth IRA created by rolling over $25,800 from her traditional IRA 20 months ago. Her Roth account has a balance of $30,000. As a result of her layoff, she needed some cash, and so she requested that her mutual fund company send her $5,500 by check, made in her name. Which one of the following statements best describes the distribution requirements that will directly affect the amount she receives on her distribution check?

A)The distribution will be income tax free because it is less than her conversion contribution.

B)The 20% mandatory withholding requirement will apply.

C)Teresa will owe income tax and the 10% early withdrawal penalty on the $5,500.

D)The distribution will be income tax free and without any premature distribution penalty because it is less than her conversion contribution.

A

A) Explanation
In this case, the mandatory withholding requirement will not apply because the payment is made from an IRA, not an employer retirement plan. The distribution will be income tax free because it is less than her conversion contribution and she paid the tax at the time of conversion. However, she will be assessed a 10% early withdrawal penalty because the distribution is from a converted IRA, five years has not elapsed, and the withdrawal does not meet any of the exceptions to the early withdrawal penalty from an IRA.

LO 5.4.2

220
Q

Sam has a Roth IRA valued at $100,000 on December 31, 2020. His original contribution was nine years ago. If Sam dies in 2021, when is the distribution to his beneficiary considered a qualified distribution?

A) 2022

B) 2025

C) 2021

D) 2024

A

C) Explanation
The beneficiary can take a qualified distribution immediately. The five-taxable-year period is not restarted when the owner of a Roth IRA dies. Thus, the beneficiary of the Roth IRA would have to wait only until the end of the original five-taxable-year period for the distribution to be a qualified distribution. The five-year period was satisfied for this Roth IRA four years ago.

LO 5.4.2

221
Q

Which of the following statements regarding the tax effects of converting a traditional IRA to a Roth IRA is CORRECT?

I. The converted amount is treated as a taxable distribution from the traditional IRA.

II. The 10% premature penalty applies if the owner is not at least 59½ years old.

A) Both I and II

B) II only

C) I only

D) Neither I nor II

A

C)Explanation
Only Statement I is correct. When a traditional IRA is converted to a Roth IRA, the converted amount is treated as a taxable distribution and is included in the owner’s gross income. The 10% penalty does not apply to the conversion amount when converted, regardless of the owner’s age. However, if the taxable portion of the converted amount is withdrawn within five years of the conversion, then the taxable portion of the conversion is treated as coming out first when the converted amount is withdrawn. This taxable amount would be subject to the early withdrawal rules and penalized 10% unless an exception applies. The point of this rule is to protect Roth conversions from being sham transactions intended to get around the 10% early withdrawal penalty.

The law treats withdrawals of converted amounts that are more than five years past the conversion date the same as contributions.

LO 5.4.2

222
Q

Which of these is considered an active participant for determining the deductibility of traditional IRA contributions this year?

I. A participant in a defined benefit pension plan who has just satisfied the eligibility requirements and entered the plan in the past six months

II. A participant in a traditional Section 401(k) plan who is currently not making elective deferrals but has $100 of forfeitures reallocated to their account this year

III. A highly compensated employee with a $500,000 account balance in a profit-sharing plan for which the plan earnings this year are $35,000 but no employer contributions, employee contributions, or reallocated forfeitures were added this year

IV. A self-employed professional with no employees maintaining a simplified employee pension (SEP) with a $10,000 account balance funded by a 20% contribution two years ago plus earnings

A) II and III
B) I, II, III, and IV
C) I and II
D) I and IV

A

C) Explanation
Active participation for purposes of determining deductibility of IRA contributions differs from being covered under a qualified plan. Specifically, an employee must be contributing to the plan, having employer contributions or forfeitures reallocated on his behalf, or accruing a defined benefit before he is considered an active participant in a qualified plan for IRA purposes. The participant in Statement I has begun to accrue a benefit in the pension plan. The participant in Statement II is considered an active participant because of the forfeiture relocation. The taxpayers in Statements III and IV are not considered active participants this year because no contributions were made on their behalf this year.

LO 5.1.1

223
Q

Which of these statements regarding prohibited transactions by a fiduciary or an individual associated with traditional IRA accounts is CORRECT?

I. Generally, if an individual or the individual’s beneficiary engages in a prohibited transaction with the individual’s IRA account at any time during the year, it will not be treated as an IRA as of the first day of the year.

II. If an individual borrows money against an IRA annuity contract, the individual must include in gross income the fair market value of the annuity contract as of the first day of the tax year.

III. Selling property to an IRA by a fiduciary or an individual owner of the IRA is not prohibited.

IV. A 50% penalty will be assessed against an IRA owner who borrows money against their IRA.

A) I, III, and IV
B) I and II
C) I, II, and III
D) II and III

A

B) Explanation
Statements I and II correctly describe prohibited transactions. Statement III is incorrect. Selling property to an IRA by a fiduciary or an individual owner of the IRA is a prohibited transaction. Statement IV is incorrect. The individual may have to pay the 10% additional tax on premature distributions.

LO 5.1.1

224
Q

Gordon is the fiduciary for a traditional IRA. He has several different investments available to him to invest the IRA assets. All of the following investments are permitted investments for a traditional IRA except

A) a real estate investment trust (REIT).
B) mutual funds.
C) stock in Bottle, Inc., which is an S corporation.
D) corporate bonds.

A

C) Explanation
Traditional IRAs cannot own S corporation stock, as an IRA is not a permissible shareholder under the S corporation rules. All of the other choices are permitted investments.

LO 5.1.1

225
Q

Stewart and Abby, both age 35, plan to contribute a total of $12,000 to their IRAs for this tax year. They both work outside the home, and they file a joint income tax return. Stewart is a teacher at the local high school and participates in a 403(b) plan. Abby’s employer does not provide a retirement plan. They expect that their MAGI in 2021 will be $129,000.

What amount, if any, can they deduct for their IRA contributions?

A) $12,000
B) $7,000
C) $0
D) $6,000

A

D) Explanation
An individual is not denied a deduction for his IRA contribution simply because of the other spouse’s active participation, unless the couple’s combined AGI exceeds $208,000 (2021). Based on their AGI, Abby will be able to deduct a contribution of up to $6,000 to an IRA. Since their combined AGI is too high for Stewart to make a deductible IRA contribution, he should consider contributing to a Roth IRA.

LO 5.1.2

226
Q

Scott and Gayle, who are both age 45, are married and file a joint income tax return for the current year. Scott is a self-employed architect who earns $110,000 of Schedule C income and pays $15,543 in self-employment tax. Gayle is not employed outside the home. What is the maximum deductible IRA contribution Scott and Gayle can make, if any, for 2021?

A) $0
B) $14,000
C) $6,000
D) $12,000

A

D) Explanation
Neither Scott nor Gayle is an active participant in an employer-sponsored retirement plan. Therefore, they can establish a traditional IRA for Scott and a spousal IRA for Gayle and contribute a deductible total of $12,000 ($6,000 each) to traditional IRAs for 2021.

LO 5.1.2

227
Q

John and Mary, both age 49, are married and file a joint income tax return for the current year (2021). John is self-employed as an engineering consultant and reports $120,000 of Schedule C net income and pays $16,955 in self-employment tax. Mary is not employed outside the home. What is the maximum deductible IRA contribution John and Mary can make this year?

A) $0
B) $12,000
C) $7,000
D) $6,000

A

Explanation
Neither John nor Mary is an active participant in an employer-sponsored retirement plan, qualified retirement plan, SEP plan, SIMPLE, or Section 403(b) plan; therefore, they can contribute and deduct $12,000 ($6,000 each) to traditional IRAs for the current year (2021).

LO 5.1.2

228
Q

Which of the following persons can make a deductible contribution to an IRA for 2021?

Person	Marital Status	AGI	Covered by Pension Plan
I. Jane	Single	$61,000	Yes
II. Joe	Married	$100,000	No
III. Betty	Single	$20,000	Yes
IV. Mary Sue	Married	$40,000	Yes

A) III and IV
B) II only
C) I, II, III, and IV
D) II, III, and IV

A

D) Explanation
Each of these persons can make deductible contributions to an IRA for 2021.

LO 5.1.2

229
Q

George and Mabel each put $6,000 into their respective IRAs. George’s employer does not provide a qualified retirement plan. Mabel participates in a 401(k) plan at work. Their AGI is $201,000 in 2021, and they file jointly. How much of their IRA contributions will be deductible?

A) $12,000
B) $6,000
C) $0
D) $4,200

A

D) Explanation
The IRA rules allow an IRA deduction for individuals who are not active participants but whose spouses are, in some cases. However, that option is phased out if the couple’s AGI is between $198,000 and $208,000 in 2021. With a combined AGI of $201,000, George would be able to deduct $208,000 − $201,000 = $7,000; ($7,000 ÷ $10,000) × $6,000 = $4,200.

LO 5.1.2

230
Q

Martha has inherited a traditional IRA that contained no after-tax contributions. She would rather not take the required minimum distributions but instead roll the distributions over into her own IRA to save for her own retirement and avoid paying income tax. Which of the following statements is CORRECT?

I. Martha should direct the IRA trustee to make an annual direct transfer to her own traditional IRA of the required minimum distributions so the distributions remain nontaxable.

II. To minimize current taxation, Martha should execute a direct transfer of the entire IRA into an inherited IRA.

A) II only
B) Neither I nor II
C) I only
D) Both I and II

A

A) Explanation
Only Statement II is correct. Required minimum distributions may not be rolled over. To decrease current taxation, Martha should execute a direct transfer to an inherited IRA. She, will, however, be required to begin required minimum distributions from the inherited IRA.

LO 5.2.1

231
Q

Maria has a traditional IRA valued at $500,000. She named her daughter, Faith, as beneficiary of the account. If Maria dies prematurely, which of the following statements is CORRECT?

I. Faith inherits the IRA.
II. Faith can transfer the inherited funds to an inherited IRA via a direct trustee-to-trustee transfer and name her own beneficiary.
III. Because Faith is a nonspouse beneficiary, she is not allowed to roll over the IRA.
IV. Faith can roll over the IRA into her own Section 401(k) plan.

A) I, II, and III
B) III and IV
C) I, II, III, and IV
D) I and II

A

D) Explanation
Statements I and II are correct. A nonspouse beneficiary (such as an adult child) may use a trustee-to-trustee transfer of the decedent’s balance from a qualified plan, Section 403(b) plan, governmental Section 457 plan, or IRA to her own inherited IRA. However, the nonspouse beneficiary must generally begin receiving the distributions from the deceased participant’s IRA immediately, whereas a surviving spouse beneficiary may continue to defer payouts until she attains age 72.

LO 5.2.1

232
Q

Which of the following best describes the purpose of establishing a stretch IRA?

A) To allow the owner to borrow money from the IRA without committing a prohibited transaction
B) To extend the period of tax-deferred earnings beyond the original owner’s lifetime
C) To allow the owner to continue making contributions to a traditional IRA beyond age 72
D) To enable the owner to take distributions before age 59½ without paying the 10% penalty

A

B) Explanation
A stretch IRA is used to stretch the period of tax-deferred earnings on the IRA beyond the lifetime of the original owner. The goal is to delay the distribution of assets from the IRA for as long as possible.

LO 5.2.1

233
Q

Under the IRA minimum distribution rules, if the IRA account owner dies before distribution payments begin, what occurs?

A) All of these occur.
B) The funds revert to the government if no beneficiary is named.
C) If the beneficiary is a named individual who is not the spouse, the beneficiary must distribute the entire account over a period not exceeding five years.
D) The beneficiary can begin receiving distributions.

A

D) Explanation
If no beneficiary is named, the funds revert to the account owner’s estate and are distributed according to the will or the state’s intestacy laws. The spouse has the option of rolling over the IRA to the beneficiary-spouse’s own account, and a nonspouse beneficiary may use a direct trustee-to-trustee transfer of the IRA into an inherited IRA. A nonspouse named beneficiary is not required to distribute the entire IRA balance within five years.

LO 5.2.1

234
Q

Distributions from IRAs must begin by April 1 of the year following the year in which an individual reaches age

A) 59½.
B) 75.
C) 72.
D) 65.

A

C) Explanation
Penalties are imposed upon distributions that commence before age 59½ or after April 1 following the year the account owner turns age 72.

LO 5.3.1

235
Q

Which of the following constitutes an exception to the imposition of the 10% premature distribution penalty for distributions made from an IRA owned by an individual who is currently age 40?

A) A distribution in payment of qualified higher education expenses

B) A distribution paid as a series of substantially equal payments over a five-year period

C)A distribution following separation from service at age 56

D) A distribution taken after demonstration of a financial hardship

A

A) Explanation
IRA distributions are exempt from the early distribution penalty if made in payment of qualified higher education expenses. The other distributions are either exempt only if made from a qualified plan or not exempt, regardless of the source. The exception for a distribution paid as a series of substantially equal payments must extend over the greater of five years or age 59½.

LO 5.3.1

236
Q

Assuming the account holder is age 40, which of these withdrawals from a traditional IRA is subject to the 10% early withdrawal penalty?

A) $10,000 to pay for medical expenses in excess of the 10% (2021) of AGI threshold

B) $10,000 donated directly to a qualified 501(c)(3) charity

C) $10,000 to pay for a qualified first-time home purchase

D) $10,000 to pay for college tuition for either the account holder or a dependent

A

B) Explanation
For tax purposes, the account holder must be at least age 70-1/2 to make a qualified charitable distribution from an IRA. The other answer options are expressly exempt from the 10% early withdrawal penalty, regardless of the account holder’s age.

LO 5.3.1

237
Q

Which of these reasons for an early distribution from an IRA is NOT an exception to the 10% penalty?

A) Early distributions made for qualifying medical expenses exceeding 10% of the account owner’s AGI

B) A distribution made after age 55 and separation from service with an employer

C) The plan owner becomes totally and permanently disabled

D) Made on or after the account owner attains age 59½

A

Explanation
A distribution made after age 55 and separation from service with an employer is not a qualified IRA early distribution penalty exception.

LO 5.3.1

238
Q

Which of the following is subject to the required minimum distribution (RMD) requirements after the account owner/plan participant dies?

Traditional IRAs
Roth IRAs
Qualified plans

A)
II and III
B)
I and II
C)
I, II, and III
D)
I only
A

C) Explanation
All of these retirement accounts are subject to RMD requirements after the account owner/plan participant dies. However, RMD requirements do not apply to Roth IRAs while the owner is alive.

LO 5.4.1

239
Q

Which of the following groups would NOT benefit from using Roth IRAs?

A) Low-income wage earners who are active participants in their employers’ plans and are not eligible to make deductible IRA contributions

B) Low-income wage earners who need current deductions

C) High-income wage earners who have exhausted the tax benefits of other tax-favored vehicles

D) Taxpayers who anticipate being in a higher tax bracket in their retirement years

A

B) Explanation
Roth IRAs offer no current deductions. Low-income wage earners needing current deductions are better served by traditional IRAs.

LO 5.4.1

240
Q

Question #19 of 30
Question ID: 1340322
Guy and Dotty, who are both age 42, are married and file a joint tax return. Their modified adjusted gross income (MAGI) for 2021 is $130,000. Dotty has already made a $6,000 contribution to her traditional IRA and has also made a $2,000 contribution to their son’s Coverdell Education Savings Account this year. What is the maximum amount that may be contributed, if any, to a Roth IRA for Guy and Dotty this year given these facts?

A)
$6,000
B)
$2,000
C)
$11,000
D)
$0
A

A) Explanation
For 2021, the maximum combined contribution to traditional and Roth IRAs (for an owner younger than age 50) is $6,000 per person annually. Dotty has already contributed the maximum amount for her traditional IRA, but they can make a further contribution of $6,000 to a Roth IRA for Guy. The $2,000 Coverdell contribution is never relevant to any retirement account contribution. Their AGI is below the Roth IRA phaseout range for a married couple ($198,000–$208,000) in 2021.

LO 5.4.1

241
Q

Which of these is CORRECT about a Roth IRA?

A) Withdrawals of earnings up to $10,000 from a Roth IRA for the purchase of a first home can be penalty free if the five-year holding period has been met.

B) If a nonqualifying distribution is made before age 59½, the principal is subject to the 10% penalty, but it is not considered to be taxable income.

C) An individual can contribute $6,000 annually to a regular IRA and $6,000 annually to a Roth IRA.

D) As with conventional IRAs, distributions from a Roth IRA must begin by April 1 of the year following the year the participant reaches age 72.

A

A) Explanation
Withdrawals of earnings from a Roth IRA are not penalized under these circumstances if the five-year holding period has been met.

LO 5.4.2

242
Q

Which of the following statements most accurately describes the tax treatment of contributions to and distributions from a Roth IRA?

I. Contributions are made with pretax dollars.

II. A withdrawal from the account will not be subject to tax if the account has been established for at least three years and the funds (up to $10,000) are being used for a first-time home purchase.

III. Distributions are not taxable if they are attributable to disability and the account has been established for at least five years.

IV. If the account has been open for at least five years and the account owner is age 59½, distributions are penalty free and income tax free.

A) III and IV
B) I only
C) I, II, and III
D) II, III, and IV

A

A) Explanation
Contributions to a Roth IRA are made with after-tax dollars. Distributions from a Roth IRA are income tax and penalty free if the owner has maintained the account for at least five years and the distribution is attributed to one of the following:

Death
Disability
First-time home purchase ($10,000 lifetime maximum)
Attainment of age 59½
LO 5.4.2
243
Q

In considering whether to convert a traditional IRA to the Roth IRA form, which of the following is a valid consideration?

A) If the taxpayer anticipates being in a lower tax bracket at date of distribution from the Roth IRA, it generally makes sense to convert.

B) It is generally advantageous if the converted assets will remain in the Roth IRA for a relatively short time period before withdrawal.

C) If the taxpayer files as married filing separately and thereby splits income, it generally makes sense to convert.

D) If the source of payment for taxes due upon conversion comes from an outside source, it generally is advantageous to convert.

A

Explanation
The Roth IRA yields greater after-tax benefits than a traditional deductible IRA if the front-end tax due upon conversion is paid from funds outside the Roth IRA and an equivalent amount of funds is, thereby, available for investment.

LO 5.4.2

244
Q

Myra, age 35, converted an $80,000 traditional IRA to a Roth IRA last year. Her adjusted basis in the traditional IRA is $20,000. She also made a contribution of $5,000 to the same Roth IRA last year. Myra is in a combined 30% marginal tax rate. If Myra takes a $4,000 distribution from her Roth IRA this year, how much total federal tax, including penalties, is due as a result of the distribution?

A) $400
B) $0
C) $1,200
D) $1,600

A

B) Explanation
Although the distribution is not a qualified distribution, it will not be taxable income because it is treated as a distribution from the Roth IRA regular contributions first. Because the $4,000 distribution is not includible in gross income, nor does it relate to a conversion within the last five years, the distribution is not subject to regular income tax or the 10% early withdrawal penalty.

LO 5.4.2

245
Q

Sherry, who is currently age 50, made only one contribution during her lifetime to her Roth IRA in the amount of $5,000 in 2014. If she were to receive a total distribution of $6,500 from her Roth IRA in 2021 to take a vacation, how would she be taxed?

A)
Because Sherry waited at least five years, the distribution will be classified as a qualified distribution and will not be taxable or subject to the 10% early distribution penalty.
B)
Because Sherry waited more than five years, the distribution will be classified as a qualified distribution and will not be taxable, but will be subject to the 10% early distribution penalty.
C)
Although Sherry waited more than five years, the distribution will not be classified as a qualified distribution, it will be taxable to the extent of earnings, and it will be subject to the 10% early distribution penalty on the taxable amount.
D)
Although Sherry waited at least five years, the distribution will not be classified as a qualified distribution and will be fully taxable and will be subject to the 10% early distribution penalty.

A

C) Explanation
A distribution from a Roth IRA is not subject to taxation if it is a qualified distribution or to the extent that it is a return of the owner’s contributions or conversions to the Roth IRA. A qualified distribution is one that meets both of the following tests:

The distribution was made after a five-year holding period.
The distribution was made for one of the following reasons:
Owner has attained age 59½
Distribution was made to a beneficiary or the estate of the owner on or after the date of the owner’s death
Distribution was attributable to the owner’s disability
Distribution was for a first-time homebuyer expense purchase
The 10% early withdrawal penalty only applies to a distribution from a Roth IRA that is includable in gross income. The 10% early withdrawal penalty also applies to a nonqualified distribution, even if it is not then includable in gross income, to the extent it is allocable to a conversion contribution made within the five-year period beginning with the first day of the individual’s taxable year in which the conversion contribution was made.

LO 5.4.2

246
Q

Marian, age 62, converts $30,000 from a traditional IRA to a Roth IRA in 2014. In 2019, she converts another traditional IRA with a fair market value of $35,000 to a Roth IRA. She makes no other IRA contributions. In 2021, Marian takes a $40,000 distribution from her Roth IRA. This distribution is treated as $30,000 from the 2014 conversion contribution and $10,000 from the 2019 conversion contribution, both of which were includable in her gross income when converted. As a result, for 2021,

A)
$10,000 is includable in Marian’s gross income.
B)
the $10,000 from the 2019 conversion is subject to the 10% penalty tax.
C)
the $10,000 withdrawal from the 2019 conversion is not subject to the 10% penalty tax.
D)
$20,000 is includable in Marian’s gross income.

A

C) Explanation
The conversion amounts were already included in Marian’s gross income when converted. Therefore, they will not be subject to income taxes again when withdrawn. The distribution allocable to the $10,000 conversion contribution made in 2019 (less than five taxable years ago) starts out as being subject to the early distribution penalty because it was withdrawn less than five years after the conversion. However, it is not subject to the 10% penalty tax under Section 72(t) in this case because Marian is over age 59½, which is one of the exceptions to the 10% penalty. The withdrawal of the conversion amount from the first conversion is not subject to the 10% early withdrawal penalty because the conversion is more than five years old. Thus, if the owner would have been younger than age 59½, the withdrawal of that money would not have been subject to the early withdrawal penalty.

LO 5.4.2

247
Q

Which of the following statements regarding the tax effects of converting a traditional IRA to a Roth IRA is CORRECT?

A)
The converted amount is treated as a taxable distribution from the IRA to the extent the distribution does not represent a return of basis.
B)
The 20% premature distribution penalty applies if the owner is less than age 59½.
C)
The 10% premature distribution penalty applies if the owner is less than age 59½.
D)
The amount converted is not considered a taxable distribution from the IRA to the extent the distribution does not represent a return of basis.

A

A) Explanation
When a traditional IRA is converted to a Roth IRA, the converted amount is treated as a taxable distribution to the extent the distribution does not represent a return of basis and is included in the owner’s gross income. A penalty does not apply for amounts converted from a traditional IRA to a Roth IRA, regardless of the owner’s age. However, the amount that was taxable at the conversion will be subject to the 10% early distribution penalty if it is withdrawn within five years of the conversion and the withdrawal does not qualify for an exception to the 10% early withdrawal penalty. This rule protects against people converting to a Roth and then taking a distribution as a way of escaping the 10% early withdrawal penalty.

LO 5.4.2

248
Q

Which of these statements is false regarding the conversion of a traditional IRA to a Roth IRA?

A) An amount in a traditional IRA may be transferred to a Roth IRA maintained by the same trustee.

B) An amount in a traditional IRA may be transferred in a trustee-to-trustee transfer from the trustee of the traditional IRA to the trustee of the Roth IRA.

C) An amount distributed from a traditional IRA can be rolled over to a Roth IRA within 60 days of the distribution.

D) The IRA owner’s modified adjusted gross income (MAGI) cannot exceed $100,000 in the year of the conversion.

A

D) Explanation
There is no MAGI limit for a taxpayer in the year in which there is a conversion.

LO 5.4.2

249
Q

Jane Paschal has contributed $1,000 each year to a Roth IRA, beginning with an initial payment of $500 on December 31, 2016. She wants to know when she can begin making qualified distributions. Which one of the following statements represent what you should tell her?

A) Qualified higher education expenses are one of the reasons a withdrawal may be a qualified distribution.

B) After December 31, 2020, the five years will have elapsed, and she could begin making qualified distributions as soon as she attains age 59½.

C) Any distributions for medical expenses in excess of 10% would qualify as a tax-free distribution after satisfying the five-year holding period even if she has not attained age 59½.

D) Any distribution she takes after January 1, 2021, will meet the five-year requirement.

A

D)Explanation
The clock started on January 1, 2016, so five years will have elapsed on January 1, 2021. A Roth IRA owner is required to hold the account for a minimum of five years to qualify for tax-free distributions. In addition, the owner must be at least age 59½, dead, disabled, or withdrawing up to $10,000 of Roth IRA earnings for qualified first-time homebuyer expenses.

LO 5.4.2

250
Q

All of these are considerations for converting distributions from qualified plans or a traditional IRA to a Roth IRA except

A) the Roth IRA conversion is more appropriate when the income tax rate is lower at the time of distribution than at the time of conversion.

B) one advantage of a conversion to the Roth IRA is that the Roth IRA will not be subject to required minimum distributions (RMD) during the life of the original owner.

C) any portion of an IRA can be converted to a Roth IRA.

D) the Roth IRA conversion becomes more appropriate the longer the period of distributions.

A

A) Explanation
The Roth IRA conversion is more appropriate when the income tax rate is the same or higher at the time of distribution than at the time of conversion.

LO 5.4.2

251
Q

Which of these statements is false about Roth 401(k) accounts?

A) Just as with any Roth IRA account, there are no required minimum distributions (RMDs) that must be made from a Roth 401(k) account.

B) Roth 401(k) accounts allow larger contributions than Roth IRAs.

C) Unlike traditional Roth IRA accounts that have phaseouts based upon income, there are no income restrictions applicable to participation in a Roth 401(k).

D) Even if a Roth 401(k) participant already has a Roth IRA account, a new five-year clock is required for the Roth 401(k).

A
A) Explanation
Roth 401(k) accounts, just as with traditional 401(k) accounts, have RMD rules that apply, meaning that distributions must start in the year the participant reaches age 72 (unless they are still working). This can be avoided by rolling the Roth 401(k) over into a Roth IRA since there are no RMDs with Roth IRAs. There is a new clock that is started for a Roth 401(k) account, even if the participant already has a Roth IRA account. However, if a participant transfers the Roth 401(k) into a Roth IRA, then the Roth IRA clock will be the one that applies, not the Roth 401(k) clock.

LO 5.4.3

252
Q

Charles and Lucy Brown each had $100,000 in their respective IRAs on December 31 of last year. Each has named the other as beneficiary. They need to determine the amount each must withdraw once withdrawals are required. This year, Charles turned age 72 on January 2 and Lucy turned age 72 on March 4.

What is the required minimum distribution (RMD) for each? (Assume the IRS RMD Joint Life Table expected return for two individuals age 72 is 19.1. The Uniform Table factor is 25.6 at age 72 and 24.7 at age 73.)

A) $5,284

B) $3,750

C) $3,906

D) $4,854

A

C) Explanation
$100,000 ÷ 25.6 = $3,906 (The first distribution year results in the same amount for each.)

LO 6.2.2

253
Q

Tony, age 65, is a nonowner employee of Widget, Inc. He wants to defer his retirement from Widget, Inc., until age 75 and continue to work. Tony contributes 6% of his pay to the Section 401(k) plan, and his employer matches 100%. Which of the following statements regarding Tony’s distribution options is CORRECT?

A) Tony will be required to take minimum distributions from his Section 401(k) plan beginning April 1 of the year after he attains age 72.

B) Tony will be subject to a 10% early withdrawal penalty on distributions received from his Section 401(k) plan.

C) Tony will be required to take minimum distributions from his Section 401(k) plan beginning April 1 of the year after he retires if he does retire after age 72.

D) Tony cannot contribute to his Section 401(k) plan after age 72.

A

C) Explanation
Generally, an individual must receive his first minimum distribution by April 1 following the year the individual attains age 72. However, if the individual remains employed beyond age 72, he may defer minimum distributions until April 1 of the year following the year of retirement. This exception to the general rule only applies to the employer’s qualified plan (not IRAs). Therefore, the other choices are incorrect. Also, this exception is not available if the individual is a greater-than-5% owner of the company sponsoring the retirement plan.

LO 6.2.1

PREV

254
Q

On December 31 of last year, Samuel Herman had $360,000 in his IRA. He has named Trudy Herman, his wife, as beneficiary. He wants you to determine the amount he must withdraw for his RMD this year and the date by which the withdrawal must be made. This year, he turned age 72 on October 17, and Trudy turned age 35 on January 8. (Assume the IRS RMD Joint Life Table divisor for two individuals ages 72 and 35 is 47.5; ages 73 and 36 is 46.6, and ages 74 and 37 is 45.6. The IRS Table I divisor for an individual age 72 is 15.5, age 73 is 14.7, and age 74 is 14.1. The Uniform Table factor is 25.6 at age 72 and 24.7 at age 73.)

What is the smallest required minimum distribution (RMD) that Samuel can take, and when must distributions begin?

A) He must begin distributions on April 1 of next year in the amount of $7,579.

B) He must begin distributions on April 1 of next year in the amount of $8,072.

C) He must begin distributions on April 1 of this year in the amount of $13,277.

D) He must begin distributions on April 1 of next year in the amount of $14,063.

A

A) Explanation
Because they are married and their ages differ by more than 10 years, the joint life factor results in a longer life expectancy and smaller required distributions, so the joint life factor may be used. A joint distribution is calculated as follows: $360,000 ÷ 47.5 = $7,579.

LO 6.2.2

PREV

255
Q

Under the required minimum distribution (RMD) rules for IRAs, a penalty tax of

A) 50% is assessed on the amount of required minimum distribution not taken before the required date.

B) 10% is assessed on excess distributions.

C) 10% is assessed on the amount of required minimum distribution not taken before the required beginning date.

D) 50% is assessed on excess distributions.

A

A) Explanation
The IRS requires the owner to take minimum distributions from a traditional IRA no later than April 1 of the year following the owner’s attaining age 72. If the amount distributed is less than the required minimum, a 50% excise tax is assessed on the amount of the shortage. The minimum distribution is determined by dividing the IRA account balance (or aggregate account balances) on December 31 of the prior year by the owner’s remaining life expectancy (or joint life expectancy of the owner and beneficiary) shown in the table. The required distribution may be taken from one IRA, but the calculation must be based on the totals in all IRAs.

LO 6.2.1

256
Q

Parrish Products would like to implement a retirement plan for its employees. The company has over 2,000 employees and would like to help them in saving for their retirement. The company chairman, Roger Parrish, is concerned about the administrative costs of the plan. He was recently informed by one of his colleagues that certain types of retirement plans are required to provide annuities to the participants and their beneficiaries. Which of the following qualified plans must provide qualified joint and survivor annuities (QJSAs) and qualified preretirement survivor annuities (QPSAs)?

I. Simplified employee pension (SEP)
II. Target benefit pension plan
III. Profit-sharing plan
IV. Defined benefit pension plan

A) I and III

B) II and IV

C) I, II, III, and IV

D) I, II, and IV

A

B). Explanation
SEPs and profit-sharing plans do not have to provide for preretirement or post-retirement joint and survivor annuities because they are not pension plans. In this case, a “pension plan” is a category, not a single type of plan. Pension plans require mandatory annual funding each and every year. The other category besides pension plans is called “profit-sharing plans.” In this usage, a profit-sharing plan is a plan that does not require mandatory annual contributions. You can remember the pension plans as the “Be my cashtarget plans”: “B” for benefit in defined benefit plans, “M” for money purchase plans, “Cash” for cash balance plans, and “Target” for target benefit plans. This mnemonic device not only separates the pension plans and profit-sharing plans, but it can be extremely helpful in selecting retirement plans for a business. For example, if a firm cannot commit to mandatory annual funding, then the “Be my cash target plans” are eliminated.

Profit-sharing plans pay nonforfeitable balances to surviving spouses as an alternative provision to preretirement and postretirement joint and survivor annuities. Qualified pension plans must provide QJSA and QPSA benefits.

LO 6.4.2

PREV

257
Q

Jerry and Cindy are divorced, and Cindy obtained a qualified domestic relations order (QDRO) assigning her 50% of Jerry’s qualified retirement plan benefit. How will the QDRO affect the benefits received from the plan?

I. If Cindy receives an early distribution from the plan pursuant to the QDRO, the distribution is tax exempt.

II. If Cindy receives an early distribution from the plan pursuant to the QDRO, the distribution is exempt from the 10% early distribution penalty.

III. Jerry becomes an alternate payee of the plan under the QDRO.

IV. The QDRO may specify when Cindy receives the plan benefit.

A) II and IV

B) I and III

C) I, II, III, and IV

D) II and III

A

A) Explanation
Statement I is incorrect. Cindy will be required to pay income taxes on the distribution to the extent the distribution would have ordinarily been taxable. Statement II is correct. An early distribution from the plan that Cindy receives pursuant to the QDRO is exempt from the 10% early distribution penalty. Statement III is incorrect. Cindy becomes an alternate payee of the plan under the QDRO. Statement IV is correct. The QDRO may specify when Cindy may receive the plan benefit.

LO 6.5.1

PREV

258
Q

Which of the following statements regarding distributions from retirement plans where death has occurred before the required beginning date (RBD) of required minimum distributions is NOT correct?

A) An estate beneficiary must take distribution from the plan using the 5-year rule.

B) A spouse beneficiary may roll the distribution over into her own IRA and is not required to take a distribution until the year following the year the spouse attains 72.

C) A spouse must take distributions over the spouse’s remaining single life expectancy, beginning in the year following the year of death.

D) A non-spouse beneficiary in a qualified plan, Section 403(b) plan, governmental Section 457 plan, or IRA also may use a direct trustee-to-trustee transfer of an inherited amount to create an inherited IRA.

A

C) Explanation
When an account owner dies prior to his required beginning date (RBD), a surviving spouse is not required to commence distributions in the year following the death of the owner. A spouse beneficiary may elect to treat the IRA as his own and defer distributions until attainment of age 72.

LO 6.4.1

259
Q

Mark attained age 72 this year. He does not plan to retire from his position with Big Trucks Inc. until his birthday on December 1 of next year, when he will be 73. Mark is an 8% shareholder in Big Trucks. When must Mark begin to receive required minimum distributions (RMDs) from his qualified retirement plan at Big Trucks?

A) Because Mark is a greater than 5% shareholder in Big Trucks, he must receive his first RMD by April 1 of next year.

B) Mark is not required to begin his RMDs until April 1 of the year following his actual retirement from Big Trucks.

C) Mark is not required to receive his first RMD until December 31 of the year following his actual retirement date from Big Trucks.

D) Because Mark is still employed by Big Trucks, he is not required to take his first RMD until December 31 of the year he actually retires from Big Trucks.

A

A) Explanation
Non-owner participants in qualified plans, Section 403(b) plans, and governmental Section 457 plans may defer the required beginning date until April 1 following the year of retirement, if the participant continues to work after attaining age 72. However, if the employee-participant is a greater than 5% owner of the business sponsoring the retirement plan, the RMD may not be deferred, but must be taken by April 1 of the year after the employee attains age 72.

LO 6.2.1

PREV

260
Q

Sam recently died at age 63, leaving an IRA with a fair market value (FMV) of $200,000 to his wife, Susan, 55, who was the only designated beneficiary. Susan has no IRA of her own. Which of these statements regarding Sam’s IRA is CORRECT?

A) Susan must receive the entire account balance within five years of Sam’s death.

B) Susan must begin taking distributions over Sam’s remaining single-life expectancy.

C) Susan can receive a distribution from the IRA now but will be subject to a 10% early withdrawal penalty.

D) Susan can receive distributions over her remaining single-life expectancy, recalculated each year.

A

D) Explanation
Susan, as the surviving spouse, can receive distributions over her remaining life expectancy. A spouse beneficiary can recalculate life expectancy each year, but distributions must begin no later than the year in which Sam would have reached age 72. Susan would also have the option of treating the IRA as her own and deferring distributions until she attains age 72. This means moving the money to an IRA in her own name as opposed to an inherited IRA. An inherited IRA has the deceased’s name in the title. For example, “Sam B Jones (deceased July 28, 2009) FBO Sally G Jones.”

LO 6.4.1

261
Q

All of the following statements regarding interest charged to a plan participant for a loan from the participant’s qualified retirement plan account are correct except

A) Generally, loans from qualified plans must be repaid within 10 years, unless the loan is used to acquire a primary residence.

B) Generally, interest on a loan from a qualified plan is nondeductible consumer interest for the participant-borrower.

C) Interest on all qualified plan loans made to key employees for the purpose of securing a primary residence is always nondeductible by the key employee.

D) Interest on qualified plan loans secured by the participant’s principal residence is deductible by taxpayers who claim itemized deductions on their tax return if the entire loan is due to employer contributions only.

A

A) Explanation
Generally, loans from qualified plans must be repaid within 5 years, unless the loan is used to acquire a primary residence.

LO 6.3.2

PREV

262
Q

Which of the following reasons for an early distribution from an IRA is NOT an exception to the 10% penalty?

A) Early distributions made for qualifying medical expenses exceeding 10% of the account owner’s adjusted gross income (AGI)

B) A distribution made after age 55 and separation from service with an employer

C) Made on or after the account owner attains age 59½

D) The plan owner becomes totally and permanently disabled

A

B) Explanation
A distribution made after age 55 and separation from service with an employer is only a qualified retirement plan early distribution penalty exception.

LO 6.3.1

PREV

263
Q

Which of these are CORRECT regarding the rules governing IRA distributions if an IRA owner dies before his or her required beginning date?

I. A spouse who is named as the beneficiary may roll the IRA over into another IRA titled in his or her own name.

II. If a designated beneficiary isn’t named, the entire account must be distributed under the five year rule.

III. If the beneficiary is a spouse, that person must begin taking distributions no later than December 31st of the year following the year in which the deceased died.

IV. A nonspouse beneficiary may take distributions based upon the Uniform Lifetime Table (Table III) each year.

A) II and III

B) III and IV

C) I and II

D) I, II, III, and IV

A

C) Explanation
Options I and II are correct. A surviving spouse who is named as the beneficiary may roll over the IRA to another IRA and retitle this account in his or her own name (option I). If the IRA owner dies before the required beginning date and if no designated beneficiary is named, the entire account must be distributed by the end of the fifth year following the year of the owner’s date of death (option II, the five-year rule). If the beneficiary is a spouse, that person can either begin taking distributions immediately or defer them until the end of the year in which the deceased would have been age 72, so option III is incorrect. Option IV is incorrect because a nonspouse beneficiary must use the Single Life Table (Table 1) for determining the first required minimum distribution; life expectancy is then reduced by one for each subsequent year.

LO 6.4.1

PREV

264
Q

Which of the following will exempt a qualified plan distribution from the 10% premature distribution penalty?

A) Separation from service under a plan provision at age 50

B) Part of a series of substantially equal periodic payments to be paid over the life expectancy of the individual

C) As a result of the individual incurring financial hardship, as that term is separately defined in IRS regulations

D) Used to pay qualified higher-education expenses

A

B) Explanation
Of these choices, a qualified plan distribution is exempt from the 10% premature distribution penalty if it is made as part of substantially equal periodic payments. The exception for qualified higher-education expenses applies only to IRA distributions. The separation from service exception only applies if the qualified plan participant is at least age 55 at the time of distribution. Hardship withdrawals are subject to the 10% early distribution rules. That is part of the hardship. It is also why a withdrawal from an IRA is better than a hardship withdrawal from an employer-provided retirement account to pay eligible college costs. The IRA withdrawal for qualified college expenses is not subject to the 10% early withdrawal penalty (EWP).

LO 6.1.1

PREV

265
Q

Carrie retired last week and received a lump-sum distribution of her employer’s stock from the company’s stock bonus plan. The value of the shares at the time of contribution varied. The first contribution was 2,500 shares at $10 per share. The second consisted of 2,500 shares at $13 per share, and the last 2,500 shares were valued at $15 per share. All stock was contributed by the employer, and the current value of the stock on the date of her retirement is $22 per share. Because she is only 63 years old today, she wants to wait to sell the shares until she is 66. How much of the lump-sum distribution should Carrie report on her income tax return in the year of her retirement?

A) Carrie must report $95,000 of ordinary income and $70,000 of capital gain.

B) Carrie has held the stock long term and must report $165,000 in capital gains.

C) Carrie has net unrealized appreciation (NUA) of $70,000 in the stock and must report $95,000 as ordinary income.

D) Carrie has no basis in the shares and must report $165,000 in ordinary income.

A

C) Explanation
The stock has a basis to the trust of $95,000, and the basis is taxed as ordinary income in the year of the lump-sum distribution, which now becomes her basis in the stock. The NUA will be taxed at the long-term capital gains rate when the stock is subsequently sold. If Carrie keeps the stock until age 66 as planned, the subsequent growth after the lump-sum distribution will also be taxed as long-term capital gain. The holding period for any subsequent growth after the lump-sum distribution begins on the date of distribution.

LO 6.1.2

PREV

266
Q

Which of the following are exempt from the 10% penalty on qualified plan distributions made before age 59½?

I. Distributions made to an employee because of “immediate and heavy” financial need

II. In-service distributions made to an employee age 55 or older but younger than 59½

III. Distributions made to a beneficiary after the participant’s death

IV. Substantially equal periodic payments made to a participant following separation from service, based on the participant’s remaining life expectancy

A) I and II

B) III and IV

C) I and III

D) II and IV

A

B) Explanation
The 10% premature distribution penalty does not apply to distributions on account of death or annuitized payments based on an individual’s remaining life expectancy. Substantially equal periodic payments can be thought of as the person doing their own annuity arrangement. Options I and II are incorrect. The law does not recognize a heavy and immediate financial need as an exception to the penalty. The age 55 exception does not apply to in-service distributions; i.e., the employee must have separated from the service of the employer on or after attaining age 55. That means the person must be 55 on December 31 of the year of separation. For example, Joe was laid off on March 27th. He turned 55 on December 25 of that year. He would qualify as having separated from service in the year he turned 55 and thus he would avoid the 10% penalty for withdrawals from his qualified plan or 403(b), but not from his IRA.

LO 6.3.1

PREV

267
Q

On December 31 of last year (year 1), Samuel, age 73, had an IRA. He has named Trudy, his wife, as beneficiary. Samuel died in year 4, on April 15, after withdrawing the required minimum amount from his plan. Trudy is 37. Assume that Trudy elects to continue to receive distributions over her life expectancy. The balance of the IRA on December 31 of year 4 is $456,743. According to her insurance agent, her life expectancy is about 50 years. How much is to be distributed and when? Assume the RMD Single Life Table factors are age 37‒45.4, 38–44.4, 39–43.5.

A) She must begin distributions by December 31 of the year following Samuel’s death. The amount is $10,287 ($456,743 ÷ 44.4 = $10,287). The factors in the RMD Single Life Table must be used to determine her life expectancy at age 38.

B) She must continue distributions in the same amount. The recalculation is frozen upon death of the plan owner. Trudy’s distribution amount is $8,227 each year for life.

C) Trudy must begin distribution within 30 days of Samuel’s death. The amount is $9,422.

D) The distribution amount is $7,688 ($384,400 ÷ 50 = $7,688). According to her insurance agent, Trudy has a life expectancy of 50 years at her present age of 37. Payments must begin within 30 days of Samuel’s death.

A

A) Explanation
The amount is $10,287 ($456,743 ÷ 44.4 = $10,287). The factors in the RMD Single Life Table must be used to determine her life expectancy at age 38. According to the RMD Single Life Table, a person age 38 has a life expectancy factor of 44.4. Trudy is 38 in the year following the year of death when her distributions must begin. Some key elements in this question are that the owner died after his RBD and that he had already taken his RMD for the year of death.

LO 6.4.1

PREV

268
Q

Shockler Consolidated just established a simplified employee pension (SEP) plan for its employees. The company has over 500 employees, 60% of whom are highly compensated. This year, Shockler contributed 6% of each eligible employee’s salary to the SEP plan. Which of the following statements regarding SEP plans are CORRECT?

I. Employees can roll money that is distributed from a SEP plan into an IRA within 60 days without withholding or penalty.

II. Employees can make direct trustee-to-trustee transfers as often as desired.

III. Rollovers can occur only once every 12 months.

IV. A SEP plan can be integrated with Social Security.

A) II and III

B) I, II, III, and IV

C) I, III, and IV

D) II and IV

A

B) Explanation
All of these statements are correct. Note: SEP plans may only use the excess method (and not the offset method) of integration. Rollovers are limited to one per 12-month period.

LO 4.2.2

PREV

269
Q

The 20% mandatory withholding requirement applies to distributions from all of the following except

A) qualified plans.

B) Section 457 plans.

C) Section 403(b) plans.

D) IRAs.

A

D) Explanation
The 20% mandatory withholding requirement does not apply to distributions from traditional IRAs, SIMPLE IRAs, or SEP IRAs.

LO 6.1.1

PREV

270
Q

Henry works for an accounting firm that sponsors a Section 401(k) plan. Henry, who has a current salary of $35,000, was hesitant to contribute to the plan because in the past he felt as though he may need the money before retirement. At a recent employer-sponsored seminar, Henry learned that he could receive a loan from his Section 401(k) plan without paying any income tax. Henry is now considering making pretax elective deferrals to the Section 401(k) plan, but he wants to know more specific details regarding loan provisions. Which of the following statements regarding loans from qualified plans is(are) CORRECT?

The limit on loans is generally half of the participant’s vested account balance not to exceed $50,000.
The limit on the term of any loan is generally five years.
If an employee leaves the company, a retirement plan loan may be rolled over to an IRA and the participant continues making the loan payments as planned.
Participant loans to a 100% owner-employee are permissible.

A) I, II, III, and IV

B) I, II, and IV

C) II only

D) I only

A

B) Explanation
Statement I is correct. Generally, loans are limited to half the vested account balance and cannot exceed $50,000. Note: When account balances are less than $20,000, however, loans up to $10,000 are available. Statement II is correct. The limit on the term of any loan is generally five years, unless the loan is for a principal residence. Loans for the purpose of buying a residence must be repaid over a reasonable period of time. Statement III is incorrect. A qualified plan loan may not be rolled over to an IRA. Any outstanding loan balance is treated as a distribution and thus is subject to income taxes and the early withdrawal penalty rules. However, he would have until the due date of his tax return for the year the loan defaulted (including extensions) to get as much of the loan back into his IRA or a qualified plan as he could. This would lower his income tax bill and reduce his 10% EWP. It would also help his eventual retirement situation. Statement IV is correct. Participant loans from qualified plans to sole proprietors, partners, shareholders in S corporations and C corporations are permitted.

LO 6.3.2

PREV

271
Q

A preretirement distribution from a qualified retirement plan can escape the 10% penalty in each of the following situations except

A) the distributions must be made to cover medical expenses deductible for the year (medical expenses that exceed 10% of adjusted gross income).
Failed
B) the distributions must be made to a beneficiary or to an employee’s estate on or after the employee’s death.

C) the distributions must be made as part of a series of substantially equal periodic payments made at least annually over the life or life expectancy of the employee or the joint lives or life expectancies of the employee and beneficiary.

D) the distributions must be made after a separation from service for early retirement at any age.

A

D) Explanation
For a preretirement distribution to escape the 10% penalty for early distribution, the distribution must be made after a separation from service for early retirement after age 55. This exception is not applicable to IRAs.

LO 6.3.1

PREV

272
Q

Which of the following statements is(are) CORRECT regarding rollovers from qualified plans or IRAs?

I. Distributions from qualified plans and IRAs require 20% mandatory withholding for federal income taxes if a trustee-to-trustee direct transfer is not used to execute a rollover.

II. A taxpayer is limited to one rollover in a one-year period (on a 365-day basis) unless the rollover is a trustee-to-trustee direct transfer.

III. A distribution from a qualified plan may not be rolled over to a governmental Section 457 plan.

IV. If a qualified plan participant has an outstanding loan from a qualified plan upon separation from service, the participant may roll over the loan into a rollover IRA as long as loan repayments continue at least quarterly.

A) II only
B) III and IV
C) I and II
D) I, II, III, and IV

A

A) Explanation
Statement I is incorrect because IRA distributions do not require 20% mandatory federal income tax withholding. Statement III is incorrect because a rollover is permitted from a qualified plan to a governmental Section 457 plan. Statement IV is incorrect because loans are not permitted from an IRA. If a person separates from service with an unpaid retirement plan loan balance, the person is allowed to move some or all of the unpaid balance into an IRA or another employer plan as a plan loan offset up to the due date of the tax return for the year the unpaid retirement plan loan was defaulted, as described in Module 3.

LO 6.1.1

273
Q

Which of the following descriptions of a regular rollover from a qualified plan to a traditional IRA is CORRECT?

A) Amounts rolled over are taxable according to rules governing the source of contribution.

B) A 20% withholding tax applies in the event of the employee-participant’s physical possession of the amount rolled over.

C) The rollover amount to the IRA is counted as a new IRA contribution and thus limits the amount that can be contributed to the IRA that year.

D) It generally must be completed within 90 days of the date of distribution from the previous plan.

A

B) Explanation
The answer is a 20% withholding tax applies in the event of the employee-participant’s physical possession of the amount rolled over. In a regular rollover, the recipient physically receives a check, made payable to the recipient, for the eligible rollover distribution from the plan trustee. Under this method of distribution, the issuer must withhold 20% of the proceeds for federal income tax. A rollover does not count against the annual contribution limit.

LO 6.1.1

274
Q

Blake, age 73, is required to take substantial required minimum distributions (RMDs) from his qualified retirement plan. He has no current need for the cash and has established traditional IRAs with his children as beneficiaries and wishes to deposit the distributions in equal amounts to each IRA within 60 days of the distributions to eventually benefit his children. Which of the following statements regarding Blake’s rollover of the RMDs is CORRECT?

A) Because Blake is over age 72, he may not roll over the RMDs to a traditional IRA, but he may roll over the RMDs to a Roth IRA.

B) A good plan is for Blake to roll over the distribution within 60 days after receipt.

C) Required minimum distributions may not be rolled over, but Blake may make equivalent contributions within 60 days of his RMD to the traditional IRAs.

D) Required minimum distributions may not be rolled over.

A

D) Explanation
While Blake may not roll over the RMD distributions, if he has earned income, he may make contributions to a traditional or Roth IRA.

LO 6.1.1

275
Q

Ryan wants to take a distribution from his SIMPLE 401(k) account balance from his previous employer and deposit it in an IRA at his local banking institution. Which of the following statements regarding his transfer is(are) CORRECT?

I. The distribution from the SIMPLE 401(k) plan is not subject to mandatory 20% withholding.

II. A direct transfer from Ryan’s SIMPLE 401(k) to an IRA is not subject to the mandatory 20% income tax withholding requirement.

A) II only
B) Both I and II
C) I only
D) Neither I nor II

A

A)

Explanation
The answer is II only. Statement I is incorrect. SIMPLE 401(k) plans are qualified plans and are subject to mandatory 20% income tax withholding for a distribution that is not a direct trustee-to-trustee transfer. When there is a direct transfer of a distribution from a qualified plan to an IRA, the mandatory 20% withholding rule does not apply.

LO 6.1.1

276
Q

Which of the following statements regarding the net unrealized appreciation (NUA) portion of employer stock received in a lump-sum distribution by a plan participant is CORRECT?

A) The NUA on the employer stock is taxed as ordinary income in the year of the distribution.

B) The portion of the fair market value on the date of distribution that is NUA is tax free to the plan participant.

C) The NUA portion of the stock value is taxed at the capital gains rate when the stock is sold.

D) When the taxpayer receives the employer stock, the stock is taxed as ordinary income when sold.

A

C) Explanation
The adjusted basis of the stock to the qualified plan trust is taxed as ordinary income to the participant in the year of the distribution.

LO 6.1.2

277
Q

This year, Martin received a lump-sum distribution from his qualified retirement plan. The distribution consisted entirely of his employer’s stock, which has a fair market value of $100,000 on the date of distribution to Martin. The adjusted basis of the stock to the trustee of the plan was $70,000. Assuming Martin does not sell the stock this year, what amount is included in Martin’s gross income as a result of the distribution?

A) $30,000
B) $0
C) $70,000
D) $100,000

A

C) Because the distribution is a lump-sum distribution of employer stock, the net unrealized appreciation (NUA) rules apply. Under the NUA rules, the adjusted basis of the stock to the trust ($70,000) is included in Martin’s gross income in the year of the distribution and is treated as ordinary income.

LO 6.1.2

278
Q

Using the Uniform Lifetime Table to calculate the required minimum distributions (RMDs) from a qualified plan is mandatory unless

A) the designated beneficiary is a child under the age of 16.

B) there is more than 1 designated beneficiary.

C) the designated beneficiary is the participant’s spouse and the spouse is more than 10 years younger than the participant.

D) there is no designated beneficiary.

A

C) Explanation
The Uniform Lifetime Table must be used to calculate required minimum distributions (RMDs) under a qualified plan or IRA unless the designated beneficiary is the participant’s spouse and the spouse is more than 10 years younger than the participant.

LO 6.2.2

279
Q

Which of the following describe differences between a tax-advantaged retirement plan and a qualified plan?

I. IRA-funded employer-sponsored tax-advantaged plans may not incorporate loan provisions.

II. Employer stock distributions from a tax-advantaged plan do not benefit from NUA tax treatment.

A) II only
B) Both I and II
C) Neither I nor II
D) I only

A

B) Explanation
Both statements are correct. IRA-funded employer-sponsored tax-advantaged plans are SEPs, SAR-SEPs, and SIMPLE IRAs.

LO 6.1.2

280
Q

Which of the following beneficiaries is entitled to roll over a post-death distribution from a qualified plan into an IRA?

A) The surviving mother of the participant
B) All three choices are correct
C) The oldest surviving child of the participant
D) The surviving spouse of the participant

A

B) Explanation
A spouse beneficiary can roll the distribution over into an IRA and treat it as the spouse’s own; a nonspouse beneficiary can use a direct trustee-to-trustee transfer of the distribution into a specially titled inherited IRA.

LO 6.4.1

281
Q

Mark attained age 72 this year. He does not plan to retire from his position with Big Trucks, Inc., until his birthday on December 1, when he is 75. Mark is a 3% shareholder in Big Trucks. When must Mark begin to receive required minimum distributions (RMDs) from his qualified retirement plan at Big Trucks?

A) Because Mark is still employed by Big Trucks, he is not required to take his first RMD until December 31 of the year he actually retires from Big Trucks.

B) Mark is not required to receive his first RMD until December 31 of the year following his actual retirement date from Big Trucks.

C) Mark is not required to begin his RMDs until April 1 of the year following his actual retirement from Big Trucks.

D) Because Mark is a shareholder in Big Trucks, he must receive his first RMD by April 1 of next year.

A

C) Explanation
Participants in qualified plans, Section 403(b) plans, and governmental Section 457 plans may defer the required beginning date until April 1 following the year of retirement, if the participant continues to work after attaining age 72. If the employee-participant owns more than 5% of the business sponsoring the retirement plan, the RMD may not be deferred but must be taken by April 1 of the year after the employee attains age 72.

LO 6.2.1

282
Q

The trustee for Debra Bennett’s qualified retirement plan is calculating her required minimum distribution for 2021. The trustee should use the plan balance for

A) December 31, 2019.
B) December 31, 2021.
C) December 31, 2022.
D) December 31, 2020.

A

D) Explanation
The trustee should use the plan balance for December 31 of the year preceding the distribution year—in this case, December 31, 2020.

LO 6.2.2

283
Q

Jackie turns 72 years old on March 15, 2021. When must she begin taking distributions from her Section 401(k) plan, assuming she no longer works for the employer-sponsor of the plan?

A) April 1, 2021
B) December 31, 2022
C) April 1, 2022
D) December 31, 2021

A

C) Explanation
Generally, the first required minimum distribution (RMD) from a qualified plan must be taken by April 1 of the year following the year in which the participant reaches age 72, so she must complete her trigger year distribution by April 1, 2022. Even though all or part of the first RMD distribution could be taken in 2022, it is still the 2021 RMD (until April 2, 2022). Also, the 2021 RMD will be based on the account balance at the end of 2020.

LO 6.2.2

284
Q

Julie attained age 72 in 2021, and will correctly take her first required minimum distribution (RMD) from her qualified plan by April 1, 2022. Her qualified plan balance for which year is used to calculate the RMD she must receive by December 31, 2022?

A) The plan balance at the end of 2023
B) The plan balance at the end of 2022
C) The plan balance minus the April 1 distribution she receives in 2022
D) The plan balance at the end of 2021

A

D) Explanation
The postponed 1st-year RMD is based on the participant’s plan or IRA account balance as of the end of the year preceding the first distribution year. Her first distribution year is 2021 because she turned 72 in 2021. The RMD for each year is based on the balance at the end of the previous year. Because the first distribution year is 2021, the balance for the 2021 RMD calculation is the account balance at the end of 2020. The second RMD, which will be for the year 2022, is based on the account balance at the end of 2021. This is the year in question.

LO 6.2.2

285
Q

Elaine is currently age 76 and scheduled to take another distribution from her former company’s qualified retirement plan later this year. Her account balance in the plan as of December 31 last year was $320,000. Under the Uniform Lifetime Table, the divisor is 22.0. However, Elaine’s actual life expectancy is only 16 years. What is the amount, if any, of Elaine’s required minimum distribution (RMD) from this plan for this year?

A) $53,333
B) $20,000
C) $14,545
D) $0, because Elaine is over age 72

A

C)Explanation
Elaine’s required minimum distribution this year is $14,545, calculated as follows: $320,000 ÷ 22.0 = $14,545. She must calculate her required distribution using the Uniform Lifetime Table and not her actual life expectancy (nor by using the difference between projected and actual life expectancy as the applicable divisor). This is actually advantageous for the owner because the original owner’s RMD is usually based on the owner’s age and the age of someone 10 years younger than the owner. This is true even if the account owner is not married. The only time the assumed life expectancy in a RMD calculation for the original account owner is different than this is when the original owner is actually married to someone who is more than 10 years younger. In that case, the RMD is based on the actual ages of the married couple according to Table II.

LO 6.2.2

286
Q

Which of the following reasons for an early distribution from a qualified retirement plan is NOT an exception to the 10% penalty?

A) The plan owner becomes totally and permanently disabled.

B) It is made after separation from service from an employer-sponsor of the plan after age 55.

C) It is a distribution for higher-education costs.

D) The distribution is made to a beneficiary of the account due to the owner’s death.

A

C) Explanation
The exception from the 10% early distribution penalty for distributions for higher-education costs only applies to IRA distributions.

LO 6.3.1

287
Q

David, a 63-year-old investor, wants to know which of the following penalties he might be subject to at some point if he continues tax deductible contributions to his traditional IRA. The applicable penalties are

I. a 10% early distribution penalty.
II. a 50% minimum distribution penalty.
III. a 6% penalty on excess contributions.
IV. a 6% penalty on excess withdrawals.

A) I and III
B) I, II, and IV
C) II and III
D) II, III, and IV

A

C) Explanation
Statement I is incorrect. Because David is over age 59½, he will not be subject to the early withdrawal penalty. Statement II is correct. If David does not begin taking minimum distributions by the required beginning date (typically April 1 following the year in which the participant attains age 72), he will be subject to the 50% minimum distribution penalty. Statement III is correct. If David contributes more than the permitted amount to an IRA, he will be subject to a 6% penalty on the excess contribution. Statement IV is incorrect. There is no penalty for excess withdrawals.

LO 6.3.1

288
Q

Karen, age 51, wishes to take distributions from her traditional IRA and avoid imposition of the 10% early distribution penalty. Which of the following distributions will allow Karen to avoid the penalty?

I. Karen is totally and permanently disabled.

II. Karen wants the distribution to pay medical expenses exceeding 10% of her adjusted gross income (AGI).

III. Karen may take distributions under the substantially equal payments rule.

IV. Karen needs cash to pay for tuition for her child at State University.

A) II and III
B) I, II, III, and IV
C) I and IV
D) III only

A

B) Explanation
All of these statements are correct.

LO 6.3.1

289
Q

Jerry and Barbara recently filed for divorce after 25 years of marriage. The property settlement approved by the court included an award to Barbara of half of Jerry’s vested benefit in his defined benefit pension plan. This was done via the drafting and implementation of a qualified domestic relations order (QDRO). Which of the following is an implication of the QDRO for Jerry and Barbara?

A) Barbara’s benefit is subject to an additional 10% penalty if received before her age 59½.

B) Barbara’s benefit upon receipt is not subject to income tax.

C) Under QDRO rules, Barbara is not eligible to roll over the distribution to an IRA.

D) When Jerry retires, Barbara’s benefit is taxable to her.

A

D) Explanation
A distribution by a qualified retirement plan to an alternate payee who is a spouse or former spouse of the participant is taxable to the spouse, if it is made pursuant to a QDRO. The distribution to Barbara pursuant to a QDRO is not subject to the 10% penalty for early distribution. Barbara may roll over the distribution to an IRA.

LO 6.3.1

290
Q

Claudia’s simplified employee pension (SEP) plan balance is $60,000. She wants to know her options for taking a loan from her SEP plan to pay some college expenses for her daughter, Caroline. Which of the following statements is CORRECT?

A) Claudia may not make a loan from her SEP plan account.

B) Claudia may borrow up to 50% of her SEP account balance to pay for Caroline’s college expenses because she is 100% vested in the account contributions.

C) SEP plan loan repayments must be in level installments payable at least quarterly over a 5-year period.

D) Because Claudia is 100% vested in the SEP plan, she may borrow up to $50,000 from the plan.

A

A) Explanation
A SEP plan is a type of IRA. A participant is not permitted to borrow from a SEP plan.

LO 6.3.2

291
Q

The 20% mandatory withholding requirement applies to distributions from all of the following except

A) Section 403(b) plans.
B) qualified plans.
C) Section 457 plans.
D) IRAs.

A

D) Explanation
The 20% mandatory withholding requirement does not apply to distributions from traditional IRAs, SIMPLE IRAs, or SEP IRAs.

LO 6.1.1

292
Q

Which of the following statements regarding the net unrealized appreciation (NUA) portion of employer stock received in a lump-sum distribution is CORRECT? The NUA portion is

A) taxed as ordinary income when the stock is sold.
B) taxed as ordinary income in the year of the distribution.
C) taxed at the capital gains rate when the stock is sold.
D) received tax free.

A

C) Explanation
The NUA portion of the distribution is taxed at the capital gains rate when the stock is sold. The adjusted basis of the stock to the qualified plan trust is taxed as ordinary income to the participant in the year of the distribution.

LO 6.1.2

293
Q

Geraldine participates in a Section 403(b) plan at work. Three years ago, she borrowed $5,000 from the plan. She has had an outstanding loan balance of $1,000 the past year, and 6 months ago she made the final payment of $1,000 to pay off that loan. Her vested account balance is currently $300,000. What is the maximum allowable loan amount she can take from the plan this year?

A) $145,000
B) $50,000
C) $150,000
D) $49,000

A

D) Explanation
Generally, the limit on loans from qualified plans is 50% of the vested account balance, up to a maximum loan of $50,000. The maximum loan must be reduced by any loan balance the participant had in the 1-year period preceding the loan. Geraldine’s maximum allowable loan is $49,000.

LO 6.3.2

294
Q

A stretch IRA

I. is the informal name for intentionally leaving money in an IRA over an extended time after the death of the original account owner.

II. extends or stretches the period of tax-deferred earnings within an IRA beyond the lifetime of the original owner.

III. allows the IRA owner’s beneficiary to name his own beneficiary upon the owner’s death.

A) III only
B) I and II
C) I, II, and III
D) II and III

A

C) Explanation
All of these statements are correct.

LO 6.4.1

295
Q

Maryellen is considering naming her estate as the beneficiary of her traditional IRA. Which of the following is(are) a disadvantage of this approach?

I. Her estate cannot be treated as a designated beneficiary for purposes of determining the distribution period after she dies.

II. The estate will probably pay more income tax on the IRA distributions than would an individual beneficiary.

A) II only
B) Neither I nor II
C) I only
D) Both I and II

A

D) Explanation
These are both potential disadvantages of naming one’s estate as beneficiary. The estate cannot be treated as a designated beneficiary for purposes of determining required minimum distributions, and the estate will begin paying income tax at the maximum rate at a much lower level than an individual beneficiary would.

LO 6.4.1

296
Q

Jake has named a trust as the beneficiary of his qualified retirement plan. The trust beneficiaries include his four children, ages 4 through 18, and his nephew, age 30. For the trust beneficiaries to be treated as the designated beneficiaries of the plan, the trust must meet all of the following requirements except

A) the appropriate documentation has been provided to the plan administrator.

B) the trust must be irrevocable or become irrevocable when Jake dies.

C) the trust beneficiaries must not be identifiable from the trust instrument.

D) the trust must be valid under state law.

A

C) Explanation
For the trust beneficiaries to be treated as designated beneficiaries, the beneficiaries must be identifiable from the trust instrument.

LO 6.4.1

297
Q

Which of the following statements is NOT a requirement for the beneficiaries of a trust to be treated as a designated beneficiary of a qualified plan or an IRA?

A) The trust is valid under state law.
B) The trust is irrevocable at the participant’s death.
C) The appropriate documentation is provided to the plan administrator.
D) The beneficiary of the trust is named on the decedent’s retirement account as a named beneficiary.

A

D) Explanation
The beneficiaries of the trust must be named (identified) in the trust instrument. Only the trust needs to be named as a beneficiary on the account.

LO 6.4.1

298
Q

Which of the following is correct regarding qualified joint and survivor annuities (QJSA) provided for in qualified pension plans?

A) QJSA provisions may never be waived.

B) The maximum QJSA payable to a surviving spouse may not exceed 150% of the amount of the annuity payable during the life of the participant.

C) QJSA provisions are never applicable to a Section 401(k) plan.

D) The QJSA amount payable to the surviving spouse may not be less than 50% of the amount of the annuity payable during the life of the participant.

A

D) Explanation
The QJSA amount payable to the surviving spouse may not be less than 50% (nor greater than 100%) of the amount of the annuity payable during the life of the participant. The QJSA may be waived with written notarized consent of the participant’s spouse. A Section 401(k) plan may be subject to QJSA requirements unless certain requirements are met.

LO 6.4.2

299
Q

Which of the following is NOT correct regarding qualified preretirement survivor annuities (QPSAs)?

A) QPSA payments are to begin no later than the month in which the participant would have reached the earliest retirement age under the plan.

B) To waive the QPSA benefit for a married participant’s spouse, the participant and the spouse must consent on a notarized written form.

C) QPSA payments must be the actuarial equivalent of not less than half of the participant’s vested account balance as of the date of the participant’s death.

D) The QPSA payable to the surviving spouse must be equal to the benefit that would have been payable to the participant.

A

D) Explanation
QPSA payments are not required to equal the benefit that would have been payable to the participant at retirement but must be the actuarial equivalent of not less than half of the participant’s vested account balance as of the date of the participant’s death.

LO 6.4.2

300
Q

Claude’s ex-wife, Sara, has secured a qualified domestic relations order against his Section 401(k) plan. What are Sara’s rollover options if she takes a lump-sum distribution of her share of Claude’s retirement plan account?

A) Sara may rollover the distribution to her own qualified plan, Section 457 plan, or an IRA, but not to a Section 403 (b) plan.

B) Sara may rollover the distribution to her own qualified plan, Section 403(b) plan, or an IRA, but not to a Section 457 plan.
C) Sara may rollover the distribution to her own qualified plan, Section 403(b) plan, Section 457 plan, or IRA.

D) Sara may rollover the distribution to her own qualified plan, Section 403(b) plan, or Section 457 plan, but not to an IRA.

A

C) Explanation
Sara may rollover the distribution to her own qualified plan, Section 403(b) plan, Section 457 plan, or IRA.

LO 6.5.1

301
Q

Which of the following statements is(are) CORRECT regarding tax-free rollovers of qualified plan distributions pursuant a qualified domestic order (QDRO)?

I. The recipient of a distribution from a qualified plan pursuant a QDRO may execute a tax-free rollover into another qualified plan.

II. The recipient of a distribution from a qualified plan pursuant a QDRO may execute a tax-free rollover into a SEP.

III. The recipient of a distribution from a qualified plan pursuant a QDRO may execute a tax-free rollover into a Section 403(b) plan.

IV. The recipient of a distribution from a qualified plan pursuant a QDRO may execute a tax-free rollover into a Section 457 plan.

A) I, II, III, and IV
B) I only
C) I and II only
D) None of these

A

A) Explanation
All of these statements are correct.

LO 6.5.1

302
Q

Which one of the following statements correctly describes the Social Security primary insurance amount (PIA)?

A) PIA will be reduced if a worker begins receiving his Social Security payments before full retirement age.

B) PIA is used only to determine a worker’s Social Security retirement benefits.

C) The formula for determining one’s PIA is weighted so that higher-income earners have a higher “income replacement ratio.”

D) PIA is derived from a retiree’s average indexed monthly earnings (AIME).

A

D) Explanation
The PIA is calculated from the AIME. The PIA is used for multiple Social Security benefits, not just the worker’s Social Security retirement benefit. The PIA is not reduced when someone takes early retirement. The retirement benefit is reduced. This is important for spousal and other Social Security benefits. Social Security is weighted so that the lower the income the higher the replacement ratio.

LO 7.2.1

303
Q

Tyrese will turn 67, his full retirement age (FRA), on August 2nd of this year and begin drawing $1,475 per month in Social Security benefits. He earns $5,000 per month and plans to continue working as long as he is able. He has asked his advisor about the reduction in Social Security since his older brother told him his benefit will be reduced if he continues working. Does a benefit reduction apply to him? If so, how does it apply?

A) Yes. His benefit will be reduced $1 for every $2 he earns over $18,960 (in 2021).

B) No. His benefit will not be reduced because his earnings are below the threshold.

C) No. There is no benefit reduction for those who continue to work after attaining full retirement age (FRA).

D) Yes. His benefit will be reduced. His income for the year will be $60,000, or $9,480 more than the $50,520 (2021) reduction threshold.

A

C) Explanation
His income during the year before his attainment of FRA is irrelevant because he had no Social Security income. There is no reduction for earnings after attaining FRA.

LO 7.2.1

304
Q

Which one of these individuals will be eligible for Medicare coverage?

A) A 55-year-old corporate director, in the capacity of director

B) A 65-year-old trust fund recipient who has always received only dividend income from the trust

C) A 68-year-old farmer

D) A 62-year-old federal government employee who was hired in 1999

A

C) Explanation
The farmer is in a covered occupation and is over age 65. Thus, he or she would receive benefits if fully insured. The 55-year-old and 62-year-old individuals are in covered occupations, but they must be age 65 to be eligible for Medicare benefits. The trust fund recipient is age 65, but he or she is not in a covered occupation for Social Security purposes.

LO 7.2.1

305
Q

Betty has worked for the same retail store for 23 years. She is approaching age 62 and is considering retiring and starting her Social Security benefit, although her full retirement age is 66. Before she does so, however, she wants to know when she will qualify for Medicare. Which of the following statements would be correct?

A) She will qualify for Medicare as soon as she begins receiving Social Security benefits.

B) She will not qualify for Medicare until she reaches age 65.

C) After she has been on Social Security for two years, she will qualify for Medicare.

D) She will not qualify for Medicare until she reaches full retirement age.

A

B) Explanation
All persons, age 65 or older, who are eligible for Social Security cash benefits, qualify for Medicare.

LO 7.2.1

306
Q

Sam was born in 1954, so his full retirement age (FRA) is 66. If he begins receiving Social Security benefits early, the amount of his monthly Social Security checks will be

A) reduced for the rest of his life.

B) reduced until he reaches age 72.

C) reduced until he reaches age 66.

D) reduced by the amount of the benefits that is above a taxable base amount.

A

A) Explanation
Early retirees born in 1954 will have their retirement benefits reduced throughout the remainder of their lives.

LO 7.2.1

307
Q

Gary was born in 1960. His plan is to start Social Security retirement benefits when he reaches his full retirement age (FRA). His wife, Lisa, is exactly four years younger. She has earned 36 credits so far and has retired to take care of Gary’s mother, who requires an abundance of assistance. She has no plans to return to the labor force. Their plan is for her to start receiving Social Security benefits when Gary files for his benefits. If Gary’s full benefit is $2,000, what will Lisa’s spousal retirement benefit be?

A) $750

B) $666

C) $1,000

D) $700

A

D) Explanation
The first thing to do is to determine Lisa’s FRA. She is four years younger than Gary, so she was born in 1964. FRA for someone born in 1960 or later is 67. She is entitled to 50% of Gary’s full benefit amount as her full spousal benefit at her FRA. By filing early, Lisa will receive a reduced benefit. If she files at age 63, she will be 4 years early. This is 48 months early. The reduction for the first 36 months is 25/36 of 1%, which equates to a 25% reduction. The remaining 12 months are reduced by 5/12 of 1% per month. This is an additional 5% (12 × 5/12% = 5%). Thus, the total reduction will be 30% for starting at age 63 (4 years early). So, her reduction will be 30% of Lisa’s spousal benefit of $1,000. $1,000 × 0.30 = $300. Thus her final spousal benefit for starting at 63 will be $700 ($1,000 ‒ $300).

LO 7.2.2

308
Q

Sandy Rigby is 29 years old. After college, Sandy worked for four years as a dental assistant. She then returned to college for four years to earn a graduate degree. For the past year, she has worked as a management trainee at a large computer company.

Which one of the following correctly describes Sandy’s status with respect to Social Security benefits?

A) She is neither fully nor currently insured.

B) She is fully insured but not currently insured.

C) She is both fully and currently insured.

D) She is not fully insured but is currently insured.

A

B) Explanation
She worked for four years, so she has 16 credits. To be fully insured at age 29 she needs 7 credits. This is one credit per year after she turned 21 (29 ‒ 22 = 7). Also, the minimum is 6 and the maximum is 40. Thus, she has plenty of credits to be fully insured. To be currently insured, she would need 6 of the last 13 credits. She only has 4 of the last 13 credits, so she is not currently insured. Not being currently insured would not hurt her as long as she is fully insured. The currently insured status is similar to life insurance in that it will only pay off if the person dies. It was added to Social Security to help the survivors of workers who have not yet earned enough credits to be fully insured.

LO 7.2.1

309
Q

You are discussing Social Security retirement benefits with your clients. You correctly state that

A) Social Security retirement benefits are indexed for cost-of-living increases every three years.

B) it is always advantageous for individuals to delay claiming their Social Security benefits until age 70.

C) unearned income may have a bearing on the amount of their Social Security retirement benefits.

D) unearned income may have a bearing on the taxation of their Social Security benefits.

A

D) Explanation
Unearned income will never lower the amount of any Social Security benefit because Social Security is not means-tested. However, anything that increases adjusted gross income (AGI) can push provisional income to the point that a larger percentage of Social Security benefits will be taxable. On the other hand, once 85% of Social Security is taxable income, that process is completed. Thus, unearned income may have a bearing on the taxation of their Social Security benefits. COLAs are indexed annually.

LO 7.2.1

310
Q

Beulah Tibbs, age 57, has been receiving Social Security disability benefits for three years. Which one of the following is a correct statement regarding her eligibility for Medicare?

A) Beulah is currently eligible for Part A and Part B Medicare coverage.

B) Beulah will first be eligible for full Medicare coverage at age 62.

C) Beulah will first be eligible for full Medicare coverage at age 65.

D) Beulah is currently eligible for Medicare Part A coverage and can apply for Part B coverage at age 65.

A

A) Explanation
People who are receiving Social Security disability become eligible for Medicare two years after they are entitled to Social Security disability. Most other people become eligible for Medicare at age 65.

LO 7.2.1

311
Q

Which of the following groups are covered by the Social Security program?

I. Self-employed persons
II. Employees of private, for-profit businesses
III. All state and local government employees
IV. Members of the armed services

A) I and II

B) I, II, III, and IV

C) I, II, and IV

D) I, II, and III

A

C) Explanation
Only Statement III is incorrect. Approximately 75% of state and local government employees are covered under the Social Security system.

LO 7.1.1

312
Q

Social Security benefits are increased annually based on increases in the cost of living. This is called

A) a COLA.

B) an FRA.

C) a PIA.

D) an RMD.

A

A) Explanation
COLA stands for cost-of-living adjustment. RMD stands for required minimum distribution. PIA stands for primary insurance amount. FRA stands for full retirement age.

LO 7.1.1

313
Q

Bill Jones, age 35, comes to see you because he has just been diagnosed with a terminal illness. His doctor told him he will not be able to work more than another 6 months and that his life expectancy is only 12–18 months. Bill also tells you that he has always been self-employed and, with the exception of the last two years, has never paid into Social Security. What benefits will be available to Bill and his family from Social Security as a result of his disability?

I. Medicare Part A
II. Monthly disability benefit
III. Lump-sum disability benefit
IV. No benefits

A) I and III

B) I and II

C) I only

D) IV only

A

D) Explanation
Social Security disability-related benefits are only available if you have paid into the system (20 credits, at four per year maximum, over the last 10 years at Bill’s age). Medicare eligibility for people age 65 and older requires payment into the system for at least 10 years. Bill does not qualify for either set of benefits.

LO 7.2.2

314
Q

Michael, a 62-year-old single man, is considering beginning his Social Security benefits to supplement his income of $14,000 per year. How much will he lose in Social Security benefits due to the earned income restrictions?

A) $1 for every $5 earned

B) $1 for every $2 earned

C) $0

D) $1 for every $3 earned

A

C) Explanation
Because his earned income is below the $18,960 (2021) earnings cap for singles, he will not be impacted by the earned income benefit reduction.

LO 7.2.1

315
Q

Carl is going to reach full retirement age (FRA) later in the current year. He has begun Social Security benefits but is still working. His Social Security benefits will

A) be reduced $1 for every $3 earned above the earnings cap.

B) be reduced $1 for every $2 earned above the earnings cap.

C) be taxed solely due to his employment.

D) not be impacted, because he is going to reach FRA in the current year.

A

A) Explanation
Compensation for work received in the year you obtain FRA will be reduced $1 for every $3 earned above the earnings cap until the first of the month in which you will obtain your FRA.

LO 7.2.2

316
Q

Which one of the following is a correct statement about the amount of Social Security retirement benefits available when a fully insured worker’s retirement benefit begins at her full retirement age (FRA)?

A) At his FRA, the worker’s spouse will receive 50% of the worker’s primary insurance amount (PIA).

B) A 63-year-old spouse of the retired worker will receive 50% of her PIA.

C) The worker will receive 80% of her PIA.

D) If the spouse has attained his FRA and is entitled to benefits on his earning record, he will receive the lower of 100% of his own PIA or 50% of the PIA based on her earnings.

A

A) Explanation
The spouse, at his full retirement age (age 65 to age 67), will receive 50% of the worker’s PIA unless the spouse’s Social Security benefit is higher based on his own earnings.

LO 7.2.2

317
Q

Which of the following correctly describes the Government Pension Offset that applies to a spouse receiving a Social Security spousal benefit while their spouse is receiving a government pension?

A) Dollar-for-dollar reduction on up to two-thirds of the Social Security spousal benefit.

B) There is no offset.

C) Dollar-for-dollar reduction on the first $1,000 only of the Social Security spousal benefit.

D) Dollar-for-dollar reduction on up to the full Social Security spousal benefit.

A

A) Explanation
There is a dollar-for-dollar reduction on up to two-thirds of the Social Security spousal benefit pension amount if their spouse is receiving a government pension.

LO 7.2.2

318
Q

This year, your 63-year-old client had $14,000 of earned income and $30,000 of investment income. He was also drawing Social Security benefits. Which one of the following correctly describes the impact on his Social Security benefits?

A) He loses $1 of benefits for every $3 above the allowable limit.

B) There is no reduction to his benefits.

C) He loses $1 of benefits for every $1 above the allowable limit.

D) He loses $1 of benefits for every $2 above the allowable limit.

A

B) Explanation
The client’s earnings (earned income) are below the allowable limit for the current year ($18,960 for 2021). Remember that according to the work penalty rule, only earned income is counted toward the allowable limit.

LO 7.2.1

319
Q

Max had net self-employment income of $100,000 from his auto repair business. What percentage is the deductible employer share of his self-employment taxes?

A) 15.3%
B) 7.65%
C) 6.2%
D) 1.45%

A

B) Explanation
The deductible employer share of payroll taxes for the self-employed is 7.65% (6.2% OASDI + 1.45% Medicare). This percentage is taken after a certain deduction is taken to calculate the net self-employment income. The point is that the employer’s share of Social Security taxation is 7.65% up to the taxable wage base.

LO 7.1.1

320
Q

Stanley received $55,000 in taxable pension plan and IRA benefits in 2021. His adjusted gross income (AGI) is $60,000 before considering his Social Security benefits. His Social Security benefit is $25,000 annually. What percentage of his Social Security benefit may be taxable in 2021?

A) 0%
B) 30%
C) 85%
D) 50%

A

C) Explanation
Because Stanley’s modified adjusted gross income (MAGI) of $72,500 (AGI of $60,000 + 50% of his Social Security income [$12,500]) exceeds the second threshold amount for a single taxpayer by an amount greater than his Social Security benefit, up to 85% of his Social Security benefits will be taxable.

LO 7.2.2

321
Q

If provisional income exceeds the thresholds given, then a maximum of ___________ of Social Security benefits are subject to taxation.

A) 50%
B) 85%
C) 20%
D) 37%

A

B) Explanation
If provisional income exceeds the stated threshold, a maximum of 85% of the excess amount is taxable as ordinary income.

LO 7.2.1

322
Q

Fully insured status generally requires a measure of employment covered by Social Security (OASDI) that is 40

A) years.
B) calendar quarters.
C) credits of coverage.
D) months.

A

C) Explanation
To qualify for most benefits under Social Security, an individual must be in fully insured status. This is defined as one credit per year since age 21 with a minimum of 6 and a maximum of 40 credits of covered employment, where an individual has earned above a specified dollar amount.

LO 7.2.1

323
Q

A covered individual or his survivor is entitled to which of the following benefits under Social Security if the worker is only in a currently insured status?

A) Surviving spouse not caring for dependent child benefit
B) Surviving child’s (dependent) benefit
C) Spousal retirement benefits
D) The worker’s own retirement benefits

A

B) Explanation
An individual who is only currently insured under Social Security (6 credits of coverage in preceding 13-quarter period) is entitled to a surviving child’s benefit. This is the case provided the child is under age 18, over age 18 and disabled by a disability that began before age 22, or under age 19 and a full-time elementary or secondary school student who is not married and was dependent on the deceased parent.

LO 7.1.1

324
Q

Which of the following statements best describes the definition of disability in qualifying for disability benefits under Social Security?

A) An individual must not be able to engage in the work associated with his last employment position.

B) An individual must be able to engage in work as specified by a Social Security Administration examiner.

C) An individual must not be able to engage in the work of any occupation for which she is trained.

D) An individual must not be able to engage in any substantial, gainful activity, and the impairment must be expected to last at least 12 months or result in death.

A

D) Explanation
Disability, for the purpose of Social Security, means an individual is so severely physically or mentally disabled that the person cannot engage in any substantial, gainful activity, which must be expected to last at least 12 months or result in death.

LO 7.2.1

325
Q

In 2021, James earned $6,000 from employment subject to Social Security taxation between January 1 and September 30. He was then unemployed for the remainder of the year. How many credits of coverage did James earn for the year 2021?

A) 3
B) 4
C) 1
D) 2

A

B) Explanation
For 2021, a worker receives 1 credit for each $1,470 in annual earnings on which Social Security taxes are withheld up to a maximum of 4 credits annually. All 4 credits may be earned in the same calendar quarter.

LO 7.2.1

326
Q

What amount is used to determine a participant’s actual Social Security retirement benefit?

A) QJSA
B) Covered compensation limit
C) Permitted disparity limit
D) Primary insurance amount (PIA)

A

D) Explanation
The PIA is used to determine a participant’s actual Social Security retirement benefit. The PIA is derived from the worker’s average indexed monthly earnings (AIME), which is based on lifetime earnings history.

LO 7.2.1

327
Q

All of the following workers are covered by the Social Security system except

A) self-employed workers.
B) railroad workers covered under the federal Railroad Retirement Act.
C) employees of tax-exempt organizations.
D) members of the armed services.

A

B) Explanation
Railroad workers covered under the federal Railroad Retirement Act are not covered by Social Security but are covered by Medicare if they meet those rules separately.

LO 7.1.1

328
Q

The employer portion of the payroll tax rate, including the portion dedicated to Social Security (OASDI) and the portion dedicated to Medicare funding, on a covered worker’s earnings up to the Social Security taxable wage base is

A) 1.45%.
B) 7.65%.
C) 6.2%.
D) 15.3%.

A

B) Explanation
Employers must pay a combined Social Security (OASDI) and Medicare rate of 7.65%. Employers pay 6.2% for Social Security and 1.45% for Medicare. The Social Security portion is payable up to the Social Security wage base of $142,800 for 2021. The Medicare portion is paid without an income limit.

LO 7.1.1

329
Q

To qualify for disability income benefits under Social Security, a worker must have an impairment that

A) is related solely to alcoholism.
B) is expected to result in death within 6 months.
C) is expected to last at least 12 months or result in death.
D) is related solely to drug addiction.

LO 7.1.1

A

C) Explanation
To qualify for Social Security disability benefits, a person must suffer an impairment that is expected to Medicare tax is assessed on an unlimited amount of compensation.

330
Q

What is the taxable wage base for 2021 subject to full Social Security taxation?

A) $142,800
B) $58,000
C) Unlimited
D) $290,000

A

A) Explanation
The Social Security taxable wage base (TWB) in 2021 is $142,800.

LO 7.1.1

331
Q

Social Security payments are

A) taxed only if you are still employed.

B) taxable if your provisional income exceeds the applicable threshold.

C) taxed only if received prior to full retirement age (FRA).

D) never included in income for purposes of taxation.

A

B) Explanation
This is the case regardless of your age or if you are still employed.

LO 7.2.2

332
Q

Charles (age 38) has just died. He has been credited with the last 30 consecutive credits of Social Security coverage in the last 30 quarters since he left school and began full-time employment. He had never worked before leaving school. Which of the following persons are eligible to receive Social Security survivor benefits as a result of Charles’s death?

I. Charles’s child, Bill, age 16
II. Charles’s child, Dawn, age 19
III. Charles’s widow, Maggie, age 38
IV. Charles’s dependent mother, Betty, age 60

A) I, II, and IV
B) II, III, and IV
C) I only
D) I and II

A

C) Explanation
Dawn is too old under the rules, Maggie does not have a child under age 16 for whom she is caring, and Betty is not eligible because she is not age 62 or older. Only Bill is eligible to receive a dependent or surviving child’s benefit.

LO 7.2.1

333
Q

Alice died five years ago. Joe, age 46, Alice’s surviving spouse, has been raising their three children (John, Mary, and Susan). Joe earns over $95,000 a year and receives $900 per month from Social Security. Alice was insured under Social Security when she died. Which of the following relatives will be eligible for a survivor benefit this year?

I. John, age 20, qualifies for the monthly child benefit because he is a full-time college student.

II. Mary, who will be 19 in September, will continue to qualify until she graduates from high school in June.

III. As the surviving spouse, Joe qualifies for the father’s benefit while he is caring for a child under age 16, but the benefit is reduced to zero because of his earnings.

IV. Susan, who is age 15, qualifies for the child’s monthly benefit.

A) II, III, and IV
B) I, II, and III
C) I, II, III, and IV
D) I and II

A

A) Explanation
To qualify for the child’s benefit, the child must be under age 18, or a full-time student at an elementary or secondary school if under age 19. A child may also qualify if the child became disabled before reaching age 22. Joe qualifies for the father’s benefit, but that benefit is reduced by $1 for each $2 of earned income in excess of the limit for persons under full retirement age (FRA). Joe’s income would reduce the benefit to zero. At John’s age, he would only qualify for a benefit if he were disabled.

LO 7.2.1

334
Q

Delaying receipt of benefits (for example until age 70) will result in all of the following except

A) receipt of delayed retirement credits.
B) higher annual cost-of-living adjustments.
C) permanently reduced benefit amount.
D) larger survivor benefits for the surviving spouse.

A

C) Explanation
Delaying receipt of benefits will allow you to earn delayed retirement credits, and your future cost-of-living adjustments will be higher because they will be based on a higher base. Also, the increased base will ultimately be used to determine survivor benefits.

LO 7.2.1

335
Q

Brent and Carol have an adjusted gross income (AGI) of $40,000, and they receive a combined Social Security benefit of $15,000. They have no tax-exempt income. What percentage of their Social Security benefit will be subject to taxation?

A) 100%
B) 50%
C) 0%
D) 85%

A

D) Explanation
Provisional income = $47,500. This is over the $44,000 threshold, so 85% of their Social Security benefit will be subject to taxation.

LO 7.2.2

336
Q

Mary is 66 years old and receives full old-age benefits from Social Security—in her case, $1,200 per month. Her husband, Ralph, age 67, who has not worked enough quarters outside the home to be covered in his own right, receives 50% of what Mary receives each month ($600). Assume that Mary dies tomorrow. What will Ralph’s Social Security benefit be?

A) $600
B) $600 + $1,200, or $1,800
C) $1,200

A

C) Explanation
The $600 spousal benefit stops, and Ralph will begin receiving 100% of Mary’s old-age Social Security benefit. This survivor benefit is not reduced because Ralph has reached his full retirement age (FRA) when his wife died.

LO 7.2.1

337
Q

Jill and Mark are celebrating their 20th wedding anniversary, receiving 20-year watches from their employers, and reaching full retirement age (FRA), all on the same day. Which of the following statements correctly describe the Social Security benefit Jill is eligible to receive?

A) 100% of her own benefit
B) 85% of her own benefit
C) 100% of Mark’s benefit
D) 75% of Mark’s benefit

A

A) Explanation
Currently, the spouse of a Social Security recipient is entitled to 50% of the recipient’s primary insurance amount (PIA), subject to a family maximum, as long as the spouse is of full retirement age (FRA) or between age 62 and FRA for reduced benefits. The spouse will take the greater of either 100% of their own benefit or 50% of their spouse’s benefit.

LO 7.2.1

338
Q

Higher-income earners will have a Social Security retirement income replacement ratio that is

A) the same as low-income earners.
B) There is no relationship between the Social Security income replacement ratios of high and low earners.
C) lower than low-income earners.
D) higher than low-income earners.

A

C) Explanation
Replacement rates, or the amount of one’s paycheck that is replaced by Social Security retirement benefits, favor lower-income earners by replacing about 90% of their (very low) earnings. Higher earners will see only a 26% replacement.

LO 7.2.1

339
Q

Social Security began as a program to provide retirement income but has been expanded to provide all of the following income except

A) survivor benefits to spouses at age 60‒61.
B) disability.
C) survivor benefits to spouse caring for a child under 19.
D) survivor benefits to children under age 19.

A

C) Explanation
Survivor benefits are provided to a spouse caring for a child under 16 or disabled. Age 16 is not “sweet 16” for the surviving spouse because when the youngest child turns 16, the survivor benefits ceased for the widow(er) caring for the deceased’s child.

LO 7.2.1

340
Q

Assume that a worker’s Social Security full retirement age (FRA) is 67, and the worker retires and starts drawing Social Security early at age 64. What are the consequences?

A) The worker receives a reduced benefit starting at age 64 but will receive the full benefit starting at age 67.

B) The worker receives a reduced benefit for the rest of his life.

C) The worker receives the full benefit at age 64 but receives a reduced benefit starting at age 70.

D) The worker’s retirement benefit will not receive cost-of-living increases until the year the person reaches FRA.

A

B) Explanation
A worker who starts Social Security retirement benefits early (as early as age 62 is allowed) will receive a reduced benefit, which will continue for life. This reduced benefit will be adjusted for inflation each year.

LO 7.2.1

341
Q

If a retiree’s annual Social Security benefit is $9,000 at age 62 and $9,600 at age 63, how many years would it take to equal the benefit of forgoing a year of payments (waiting one more year to receive a larger payment), not including inflation?

A) 11 years
B) 8 years
C) 15 years
D) 13 years

A

C) Explanation
$9,000 divided by the extra amount ($600) equals 15 years to equal the same benefit taking the larger amount the next year. ($9,000 ÷ $600 = 15 years)

LO 7.2.1

342
Q

When does the maximum family benefit limitation NOT apply to reduce total Social Security benefits payable?

A) When both the husband and the wife receive a retirement benefit based on their own employment record

B) When the husband receives a spousal benefit based on the wife’s employment record

C) When a divorced spouse qualifies for benefits because she is caring for a dependent child of the worker-participant

D) When the wife receives a spousal benefit based on the husband’s employment record

A

A) Explanation
The maximum family benefit limitation does not apply when both the husband and wife receive a retirement benefit based on their own (or each) employment record. It does apply to reduce total Social Security benefits payable in the other situations.

LO 7.2.1

343
Q

What is the maximum amount of Social Security benefits that may be subject to income taxation?

A) 85%
B) 50%
C) 0%
D) 35%

A

A) Explanation
If someone has provisional income in excess of $32,000 for unmarried taxpayers and $44,000 for married taxpayers, up to 85% of Social Security benefits may be subject to income taxation. These numbers are not indexed. Also, the threshold is $0 for people who file their return as married filing separately (MFS) if they are not living apart.

LO 7.2.2

344
Q

Janice is the self-employed owner of an unincorporated business. She has hired the following family members to work in her business. Which family member(s) is(are) covered by Social Security?

I. Her husband
II. Her daughter, age 16
III. Her son, age 19

A) I only
B) I, II, and III
C) II and III
D) I and III

A

D) Explanation
A child under the age of 18 who is employed in the parent’s unincorporated business is not covered by Social Security.

LO 7.1.1

345
Q

Jeff and Julie, both age 50, are married and are both fully insured under Social Security. They have two children. Amy is 24 and was injured in an auto accident when she was 21 leaving her disabled and dependent on her parents for her care. Brad is 17 and a senior in high school. Jeff’s mother, Lisa, is age 66 and lives with the couple but is not a dependent for tax purposes. If Julie dies, who may receive a survivor benefit under Social Security based upon her fully insured status?

I. Amy
II. Brad
III. Jeff
IV. Lisa

A) I only
B) I, II, III, and IV
C) I, II, and III
D) III and IV

A

C) Explanation
Amy, Jeff, and Brad will receive benefits. Amy became disabled before age 22 and will receive a benefit, as will Jeff, who is now her caregiver. Brad will receive a benefit because he is an unmarried child under age 19 and still in high school. Because Lisa is not a dependent, she is not eligible for a benefit.

LO 7.2.1

346
Q

George was born in 1962. His full retirement age (FRA) for Social Security purposes is

A) 65.
B) 67.
C) 62.
D) 70.

A

B) Explanation
The FRA for individuals born in 1960 and later is 67.

LO 7.2.1

347
Q

Maryellen, age 63, is receiving Social Security retirement benefits. She also works part time, and her earnings are $10,000 more than the earnings limit. Her Social Security retirement benefits this year will be reduced by

A) $5,000.
B) $7,500.
C) $0.
D) $10,000.

A

A) Explanation
Maryellen has not reached full retirement age, so she is subject to the retirement earnings test. Her Social Security benefits will be reduced by $1 for every $2 in earnings over the applicable limit.

LO 7.2.1

348
Q

What is the earliest age at which a currently insured worker may claim Social Security retirement benefits based on her own work history?

A) Retirement benefits are not available for a currently insured worker
B) Age 67
C) Age 62
D) Any age over age 59½, but benefits will be reduced

A

A) Explanation
The worker must be fully insured to qualify for retirement benefits based on her own work history. Reduced spousal retirement benefits are available as early as age 62. Reduced survivor benefits are available as early as age 60 unless the surviving spouse is disabled, then reduced survivor benefits are available as early as age 50.

LO 7.2.1

349
Q

Whose Social Security benefit is included in the calculation of the maximum family benefit for those eligible for benefits based on a retired worker’s fully insured status?

I. The retired worker’s benefit
II. The retired worker’s former spouse
III. A dependent child’s benefit
IV. A caregiver spouse’s benefit for caring for a qualified disabled child

A) III and IV
B) I, III, and IV
C) I, II, III, and IV
D) I and II

A

B) Explanation
Only Statement II is incorrect. Benefits paid to a former spouse of a covered worker are not considered in the application of the family maximum benefit limit.

LO 7.2.1

350
Q

Which of the following statements is(are) CORRECT regarding the Social Security Windfall Elimination Provision (WEP)?

I. The WEP is designed to reduce Social Security retirement benefits for any worker who is eligible for retirement benefits from multiple employers.

II. An example of a worker who may be subject to the WEP is one who works for a government agency or a foreign employer.

III. The WEP applies to all Social Security benefits for which a family member or dependent of a covered worker may qualify.

IV. In the case of a worker subject to the WEP, the provision is applied by reducing the formula used to calculate a worker’s primary insurance amount (PIA), resulting in a lower benefit.

A) II and IV
B) I and III
C) II, III, and IV
D) I only

A

A) Explanation
The WEP does not apply simply if a worker is eligible for retirement benefits from multiple employers. It may apply to a worker who qualifies for a Social Security retirement or disability benefit but who also earns a pension from an employer who does not withhold Social Security taxes. An example of such an employer could be a state or local government agency that had opted out of the Social Security retirement system; a public school employee is some states; or an employer in a foreign country. Under this provision, the formula used to calculate a worker’s PIA is reduced, resulting in a lower benefit. Social Security benefits are designed to pay a higher percentage of career average earnings for lower-paid workers than are paid to higher earning workers. Statement III is incorrect because the WEP does not apply to spousal or survivors benefits under Social Security. The Government Pension Offset would apply to Social Security spousal or survivor benefits when the non-Social Security worker is the spouse or the survivor of a fully covered retired or deceased worker.

LO 7.2.2

351
Q

Under the Social Security system, immediate survivor income benefits based on a deceased worker’s primary insurance amount and coverage are available to which of the following persons?

I. A surviving spouse, age 55, caring for the worker’s 13-year-old child

II. Unmarried, dependent children under age 18
Unmarried children who become disabled before age 22

III. A surviving divorced spouse, age 62, who has not remarried and was married to the decedent for more than 10 years

A) III and IV
B) I, II, III, and IV
C) II and III
D) I only

A

B) Explanation
Each of these persons is eligible for survivor’s benefits.

LO 7.2.2

352
Q

Tom and Martha, longtime clients of yours, have reviewed their retirement planning and believe they do not have time to accumulate sufficient retirement assets to retire in six years when Tom is 62. Which of the following alternatives should they consider?

I. Retire at an older age
II. Save more
III. Reduce the amount of income they require at retirement

A) I, II, and III
B) II only
C) I and II
D) I only

A

A) Explanation
Because the desired retirement age is so soon, the amount of retirement assets available may be insufficient to allow for retirement at age 62. Therefore, all of the alternatives should be considered.

LO 8.3.1

353
Q

Bedford Enterprises Inc. is a closely held corporation that manufactures computer software products. David owns 100% of the stock in the corporation. Bedford Enterprises Inc. sponsors a money purchase plan. Additional information about the corporation is presented in the following.

David is 45 and hopes to retire at age 65.
The next-oldest employee expects to retire in 22 years.
The corporation has been in business for 10 years and has a history of increasing profits.
The corporation employs 35 people, with 60% of the employees under age 40.
The average turnover rate among employees is high.
The owner’s risk tolerance level is medium.
The fair market value of the money purchase plan’s assets is $4 million.
As the financial planner for David, you have been asked to evaluate the money purchase plan’s investment portfolio, which is distributed as follows:
10% invested in short-term certificates of deposit with staggered maturity dates
20% limited partnership interest in a private commercial real estate project that generates a high income yield
40% invested in company stock
30% in long-term government bonds with staggered maturity dates
Which of the following statements best describe the appropriateness of the money purchase plan’s investment portfolio?

I. The investments in the short-term certificates of deposit have fixed maturity dates and therefore are not appropriate for liquidity reasons.

II. The investments in company stock exceed the 10% maximum allowed for a money purchase plan.

III. The investment in the limited partnership may be subject to unrelated business taxable income (UBTI) treatment and therefore is inappropriate.

IV. Overall, the types of investments selected by the plan are sufficiently diversified and therefore minimize investment risk.

A) I, II, and III
B) II and III
C) I and II
D) I, II, and IV

A

B)Explanation
Options II and III are correct. The investment of 40% of the plan’s assets in company stock violates the requirements for money purchase plan investments. A qualified retirement plan that invests in a limited partnership is considered to be engaged in the partnership’s business as if it were a general partner; hence, the UBTI rules apply.

Option I is incorrect because the short-term certificates of deposit do not pose an interest rate risk and therefore can be used to satisfy the plan’s liquidity requirements (like former employees rolling their accounts out of the plan). Option IV is incorrect because disproportionately large investments have been made in particular asset categories, such as employer securities, that do not minimize risk.

LO 8.2.1

PREV

354
Q

Richard participates in a traditional defined benefit pension plan at work. His projected monthly benefit under the plan is $1,000. If the plan provides life insurance for Richard, the death benefit payable under the policy is limited to

A) $100,000.
B) $17,500.
C) $215,000.
D) $52,000.

A

A) Explanation
Defined benefit plans use the 100 times test for determining whether they comply with the incidental benefit rules. Under this test, the death benefit cannot exceed 100 times the participant’s projected monthly benefit (in this instance, $100,000).

LO 8.2.2

355
Q

Which one of the following describes a basic provision of a SIMPLE IRA?

A) An employer may add a SIMPLE IRA plan to an existing defined benefit plan to allow employees to make elective deferrals.

B) SIMPLE IRA plans can be arranged to allow for in-service loans for up to 50% of the account balance, but not to exceed $50,000.

C) One contribution formula an employer can use under a SIMPLE IRA is to make a 2% nonelective contribution on behalf of each eligible employee with at least $5,000 in current compensation.

D) Only employers that average fewer than 20 employees can establish a SIMPLE IRA.

A

C) Explanation
SIMPLE IRA plans are available to employers with 100 or fewer employees and with no other qualified retirement plan. The employer contribution requirement may be satisfied by either a 3% matching contribution formula or a 2% nonelective contribution for each employee with current-year compensation of $5,000 or more.

LO 8.1.1

356
Q

Paul estimates he will need a $75,000 annual income in today’s dollars when he retires 10 years from now. He assumes a 3% annual rate of inflation, a 5% after-tax rate of return on his investments, and a 20-year retirement period. Using the level payment approach, how much will Paul need to save in a single annual payment at the end of each year to fund his retirement need?

A) $120,880.49

B) $134,331.53

C) $75,000.00

D) $100,794.00

A

B)Explanation
Paul will need to make 10 level annual payments of $134,331.53 at the end of each year.

Step 1: What will it take to produce $75,000 of purchasing power in 10 years?

$75,000 PV 3, I

10, N

FV Answer: $100,793.7285

Step 2: How much do they need at retirement to pay a serial payment of $100,793.7285 at the beginning of each year for 20 years?

$100,793.7285, PMT 20, N

[(1.05 ÷ 1.03) – 1] × 100, I (This number is 1.9417)

0, FV

Shift Mar (To get into the Begin Mode) PV Answer is $1,689.607.5595

Step 3: How much needs to be saved as a level payment at the end of each year until retirement so the account will have

$1,689,607.5595?

$1,689,607.5595, FV 10, N

5, I O, PV

Shift, MAR (To change to the End Mode) PMT Answer $134,331.53

LO 8.3.2

PREV

357
Q

Which of these qualified retirement plans allow unrestricted investment in employer securities?

I. Money purchase pension plans
II. Stock bonus plans
III. Employee stock ownership plan (ESOPs)
IV. Traditional profit-sharing plans

A) II, III, and IV

B) I, III, and IV

C) II and III

D) I, II, III, and IV

A

A) Explanation
Money purchase pension plans (and other pension plans) allow only 10% contributions in employer securities. All of the other plans are types of profit-sharing plans and may invest 100% in securities.

LO 8.1.1

358
Q

Caitlin has determined she has a future value of retirement savings need of $1,157,140. If she retires in 16 years and achieves an 8% after-tax annual return on her investments, what amount of level end of year annual deposit is required to fund this need?

A) $38,159
B) $20,285
C) $35,332
D) $23,878

A

A) Explanation
The annual level end of year deposit required is $38,159, calculated as follows: In END mode

FV = $1,157,140

n = 16

i = 8

PMT = −38,158.8576

LO 8.3.2

PREV

359
Q

Which of these types of qualified retirement plans do NOT allow integration with Social Security?

I. Traditional defined benefit pension plan
II. Money purchase pension plan
III. Profit-sharing plan
IV. Employee stock ownership plan (ESOP)

A) IV only
B) II, III, and IV
C) I, II, III, and IV
D) II and IV

A

A)
Explanation
An ESOP may not be integrated with Social Security.

LO 8.1.2

360
Q

A fully insured Section 412(e)(3) pension plan is funded exclusively by

A) blue-chip stocks.
B) cash value life insurance or annuity contracts.
C) Treasury bonds.
D) municipal bonds.

A

B) Explanation
A fully insured 412(e)(3) pension plan is funded exclusively by cash value life insurance or annuity contracts. Using insurance as a funding vehicle ensures the payment of a death benefit to plan beneficiaries.

LO 8.2.1

PREV

361
Q

Barry is the sole shareholder of Zippy Internet Services, Inc., a C corporation that employs 10 people. The company has been in business for four years and has had fluctuating cash flows during that time. Barry would like to install a qualified plan with the following criteria:

I. Reward, motivate, and retain employees

II. Reduce corporate income taxes

III. Provide for his own retirement

IV. Barry would like to contribute to each employee’s account an amount equal to 8% of compensation. What would be the most appropriate course of action for Barry?

A) Establish a profit-sharing plan
B) Establish a defined benefit pension plan combined with a profit-sharing plan
C) Establish a money purchase pension plan
D) Establish a traditional defined benefit pension plan

A

A).Explanation
A profit-sharing plan would accomplish all of Barry’s objectives and is appropriate for a business with unstable cash flow. Defined benefit pension plans and money purchase pension plans require annual mandatory funding, making them inappropriate for businesses with unstable cash flows.

LO 8.1.2

362
Q

All of the following are primary assumptions in any retirement needs calculation except

A) the number of dependents a client will have at retirement.

B) the client’s age at retirement and anticipated life expectancy.

C) the anticipated annual rate of inflation.

D) the projected rate of annual investment return.

A

A) Explanation
The answer is the number of dependents a client will have at retirement. There are three primary assumptions that are made in any retirement needs analysis calculation. They are

the anticipated annual rate of inflation;
the projected rate of annual investment return; and
the client’s age at retirement and anticipated life expectancy.
LO 8.3.1

363
Q

Use the following information about Lisa and Tim, a married couple, to answer the question that follows.

Lisa and Tim are both 48 and are planning to retire at 62.
They estimate that their annual income need at retirement will be $62,000 in today’s dollars.
They expect to receive $32,000 (in today’s dollars) annually from Social Security and they wish to include this amount in their retirement needs analysis.
Assume that Social Security benefits will be adjusted for inflation.
After discussions with their financial planner, they feel confident that they can earn a 7% after-tax return on their investments and would like to assume that inflation will average 4% over the long term.
Life expectancy tables are provided in IRS Regulations Section 1.401(a)(9)-9. RMD Single Life Table—Life Expectancy indicates a factor of 23.5 years at age 62. RMD Joint Life and Last Survivor Table—Life Expectancy indicates a factor of 29.0 years at age 62.
Due to a history of longevity in both their families, Lisa and Tim would like to assume a retirement period of 35 years.
What amount of assets will Lisa and Tim need at the beginning of their retirement period to fund an annual income need that increases annually with inflation (i.e., a growing annuity)?

A) $1,168,060

B) $1,017,211

C) $1,635,275

D) $1,135,310

A

A) Explanation
First, the retirement income deficit is $30,000 (PV) in today’s dollars. This is determined by reducing their gross income need of $42,000 by the anticipated Social Security benefit, $12,000. This net figure is inflated at 4% (I) over the 14 years (N) until retirement (age 48 to age 62). Thus, the income deficit in the first year of retirement (FV) is $51,950.

To determine the $1,168,060 lump sum needed: 51,950 PMT, 35 N, 2.8846 I (using a 7% after-tax rate of return and a 4% inflation rate). Note that a retirement period of 35 years is used, which differs from the mortality tables referenced in the question text. Very small differences in these numbers come from how many decimal places are used. For example, calculating the inflation-adjusted interest rate and then pressing I/YR, actually enters the calculated interest rate to nine decimals even though the calculator only shows 4. Thus, if you write down 2.8846 and then enter that number for I/YR a very small difference will be calculated for the PV. This difference will not be enough to change an answer on the test.

The other choices are incorrect for the following reasons:

$1,017,211 is wrong because a retirement period of 28 years (27.8 years rounded up) was used. Note how the lump sum needed is reduced by assuming a shorter period of retirement.
$1,135,310 is wrong because the result was arrived at by calculating the present value of an ordinary annuity (End mode), not an annuity due (Begin mode).
$1,635,275 is wrong because the calculation was performed using a gross number of $42,000, not $30,000.
LO 8.3.2

364
Q

Barry is the sole shareholder of Zippy Internet Services, Inc., a C corporation that employs 10 people. The company has been in business for four years and has had fluctuating cash flows during that time. Barry would like to install a qualified plan with the following criteria:

Reward, motivate, and retain employees
Reduce corporate income taxes
Provide for his own retirement
Barry would like to contribute to each employee’s account an amount equal to 8% of compensation. What would be the most appropriate course of action for Barry?

A) Establish a profit-sharing plan

B) Establish a defined benefit pension plan combined with a profit-sharing plan

C) Establish a money purchase pension plan

D) Establish a traditional defined benefit pension plan

A

A) Explanation
A profit-sharing plan would accomplish all of Barry’s objectives and is appropriate for a business with unstable cash flow. Defined benefit pension plans and money purchase pension plans require annual mandatory funding, making them inappropriate for businesses with unstable cash flows.

LO 8.1.2

365
Q

All of the following are primary assumptions in any retirement needs calculation except

A) the number of dependents a client will have at retirement.

B) the client’s age at retirement and anticipated life expectancy.

C) the anticipated annual rate of inflation.

D) the projected rate of annual investment return.

A

A) Explanation
The answer is the number of dependents a client will have at retirement. There are three primary assumptions that are made in any retirement needs analysis calculation. They are

the anticipated annual rate of inflation;
the projected rate of annual investment return; and
the client’s age at retirement and anticipated life expectancy.
LO 8.3.1

366
Q

Use the following information about Lisa and Tim, a married couple, to answer the question that follows.

Lisa and Tim are both 48 and are planning to retire at 62.
They estimate that their annual income need at retirement will be $62,000 in today’s dollars.
They expect to receive $32,000 (in today’s dollars) annually from Social Security and they wish to include this amount in their retirement needs analysis.
Assume that Social Security benefits will be adjusted for inflation.
After discussions with their financial planner, they feel confident that they can earn a 7% after-tax return on their investments and would like to assume that inflation will average 4% over the long term.
Life expectancy tables are provided in IRS Regulations Section 1.401(a)(9)-9. RMD Single Life Table—Life Expectancy indicates a factor of 23.5 years at age 62. RMD Joint Life and Last Survivor Table—Life Expectancy indicates a factor of 29.0 years at age 62.
Due to a history of longevity in both their families, Lisa and Tim would like to assume a retirement period of 35 years.
What amount of assets will Lisa and Tim need at the beginning of their retirement period to fund an annual income need that increases annually with inflation (i.e., a growing annuity)?

A) $1,168,060

B) $1,017,211

C) $1,635,275

D) $1,135,310

A

A) Explanation
First, the retirement income deficit is $30,000 (PV) in today’s dollars. This is determined by reducing their gross income need of $42,000 by the anticipated Social Security benefit, $12,000. This net figure is inflated at 4% (I) over the 14 years (N) until retirement (age 48 to age 62). Thus, the income deficit in the first year of retirement (FV) is $51,950.

To determine the $1,168,060 lump sum needed: 51,950 PMT, 35 N, 2.8846 I (using a 7% after-tax rate of return and a 4% inflation rate). Note that a retirement period of 35 years is used, which differs from the mortality tables referenced in the question text. Very small differences in these numbers come from how many decimal places are used. For example, calculating the inflation-adjusted interest rate and then pressing I/YR, actually enters the calculated interest rate to nine decimals even though the calculator only shows 4. Thus, if you write down 2.8846 and then enter that number for I/YR a very small difference will be calculated for the PV. This difference will not be enough to change an answer on the test.

The other choices are incorrect for the following reasons:

$1,017,211 is wrong because a retirement period of 28 years (27.8 years rounded up) was used. Note how the lump sum needed is reduced by assuming a shorter period of retirement.
$1,135,310 is wrong because the result was arrived at by calculating the present value of an ordinary annuity (End mode), not an annuity due (Begin mode).
$1,635,275 is wrong because the calculation was performed using a gross number of $42,000, not $30,000.
LO 8.3.2

367
Q

If a businessowner-client is of an older age, near his retirement date, and just establishing a qualified plan, which of the following plans would generally NOT be advantageous to the owner?

A) Traditional profit-sharing plan

B) New comparability plan

C) An age-weighted profit-sharing plan

D) Traditional defined benefit pension plan

A

A) Explanation
A traditional profit-sharing plan would not allow the plan contributions to be skewed for the benefit of the older owner. All of the other plan choices listed would allow greater contributions for the older owner.

LO 8.1.2

368
Q

Which of the following types of retirement plans would be suitable for a businessowner who is uncomfortable with the idea of mandatory annual contributions?

I. Defined benefit pension plan
II. Profit-sharing plan

A) I only

B) II only

C) Neither I nor II

D) Both I and II

A

B) Explanation
All defined benefit plans require mandatory annual funding. A defined benefit plan would not be appropriate for a businessowner who is uncomfortable with mandatory annual funding.

LO 8.1.2

369
Q

Which of the following statements describes a component of calculating the lump-sum capital amount necessary to fund the projected income need over the retirement need period?

I. Adjust or inflate the projected first-year retirement income need (expressed in present value dollars) to future dollars at the time of retirement.

II. Calculate the total retirement fund needed (lump-sum capital amount) to meet the projected income demands. To do this, the planner must calculate the present value of an annuity due using an inflation-adjusted rate of return.

A) II only

B) Both I and II

C) Neither I nor II

D) I only

A

B) Explanation
The answer is both I and II.

LO 8.3.1

370
Q

Carol has an additional retirement need of $30,000 annually in today’s dollars. She will retire in 15 years and projects a retirement period of 20 years. Carol believes she can achieve a 6% after-tax rate of return and is assuming a 4% annual rate of inflation. She has accumulated $175,000 toward her retirement plan. What lump-sum amount should Carol have accumulated over the next 15 years to support her retirement income need?

A) $873,553

B) $503,707

C) $732,144

D) $907,144

A

D) Explanation
Carol’s total retirement fund needed to support her desired standard of living is $907,144, calculated as follows:

Carol’s first year retirement income need is $54,028.

PV = –$30,000

i = 4

n = 15

FV = $54,028

The total capital required to support this need for 20 years is $907,144.

In BEGIN mode (the client will make annual withdrawals at the beginning of each year)

PMT = $54,028

n = 20

i = 1.9231 [(1.06 ÷ 1.04) – 1] × 100

PVAD = –$907,144

LO 8.3.2

371
Q

The principal disadvantage of assuming a flat annual rate of return when performing a retirement needs analysis is that this method

A) does not take into consideration the effect of taxes.

B) generally underestimates the actual rate of return.

C) does not take into consideration the effect of inflation.

D) does not take into consideration the annual volatility of returns.

A

D) Explanation
A flat annual rate of return has the disadvantage of not taking into consideration the annual volatility of returns.

LO 8.3.1

372
Q

Freddy owns a small business as a sole proprietorship and is in the 12% tax bracket. He and his employees are all under 30. He would really like to buy a boat and cruise the coast. It looks like the company will have a second profitable year if the new equipment needed to update the business doesn’t cost too much. What plan would be appropriate for this situation?

A) Cash balance plan

B) None; the business isn’t mature enough yet

C) Stock bonus plan

D) Money purchase plan

A

B) Explanation
The business needs to be more profitable and established. Also the other choices either require mandatory annual funding (money purchase plan and cash balance plan) or stock (stock bonus plan). The business is not established enough to commit to mandatory annual funding and there is no stock with a sole proprietorship.

LO 8.1.1

373
Q

Dave is covered by a qualified retirement plan. If the plan provides life insurance on Dave’s life, which of the following statements regarding the income tax ramifications to Dave is CORRECT?

I. Dave must include the pure protection cost of the life insurance in his income.

II. The pure protection cost of the life insurance will be treated as nontaxable basis once Dave begins receiving distributions from the plan.

A) Both I and II
B) I only
C) II only
D) Neither I nor II

A

A) Explanation
A qualified plan participant must include the pure protection cost of the life insurance as income, but this amount is treated as nontaxable basis once the participant begins receiving distributions from the plan.

LO 8.2.2

374
Q

All of the following are individual savings alternatives an employed person may typically use to meet his retirement savings goals except

A) Roth IRAs.

B) nonqualified deferred compensation (NQDC) plans.

C) fixed and variable annuities.

D) traditional IRAs.

A

B) Explanation
IRAs and annuities are individual savings plan alternatives a client can implement to meet his retirement income goals. NQDC plans are employer-sponsored alternatives.

LO 8.3.1

375
Q

Which of the following should a businessowner accomplish before considering the adoption of a retirement plan?

I. Purchase personal and business liability insurance.
II. Establish cash reserves sufficient to cover potential emergencies.
III. Ensure the business has sufficient cash flow to support ongoing funding of the plan.

A) II only
B) I only
C) I, II, and III
D) III only

A
C) Explanation
A businessowner (or a person planning for his own retirement) should accomplish all of these objectives before considering the adoption of a retirement plan.

LO 8.1.1

376
Q

Which of the following retirement plans can be adopted only by private, tax-exempt organizations and state or local governments?

A) Stock bonus plans
B) ESOP
C) Section 457 plan
D) Section 403(b) plan

A

C) Explanation
A Section 457 plan can be adopted only by private, tax-exempt organizations and state and local government entities. Section 403(b) plans may be adopted by Section 501(c)(3) nonprofit organizations, and ESOPs and stock bonus plans may be adopted by corporations.

LO 8.1.1

377
Q

A businessowner-client approaches a financial planner for advice on selecting a retirement plan for the business. What factors should guide the financial planner’s recommendations?

I. The owner’s retirement savings need
II. The owner’s current age
III. The amount of risk the client is comfortable assuming
IV. The financial stability of the business

A) I, II, III and IV
B) III and IV
C) I, II, and III
D) I and III

A

A) Explanation
All of the factors listed should be considered in selecting a retirement plan for the business.

LO 8.1.1

378
Q

All of the following retirement plans permit employees to make elective deferrals except

A) SEP plans.
B) profit-sharing plans with Section 401(k) provisions.
C) SIMPLE 401(k)s.
D) Section 401(k) plans.

A

A) Explanation
Employee elective deferrals are permitted in Section 401(k) plans, profit-sharing plans with a Section 401(k) provisions, and SIMPLEs, but not in SEP plans.

LO 8.1.1

379
Q

Which of the following plans would NOT be subject to annual additions limit?

A) Age-weighted profit-sharing plan
B) Traditional profit-sharing plan
C) Defined benefit pension plan
D) Target benefit pension plan

A

C) Explanation
A defined benefit pension plan is not subject to the annual additions limit. Each of the other plans listed is a defined contribution plan which is subject to the annual additions limit.

LO 8.1.1

380
Q

Bernie and Tim, both age 53, are partners in a computer software consulting firm. They have 20 employees whose average age is 25 and average length of employment is three years. The firm is highly profitable and enjoys stable cash flows. Of the following retirement plan options, which is best suited to the partners’ business?

A) An eligible Section 457 plan
B) Traditional defined benefit pension plan
C) A stock bonus plan
D) Section 403(b) plan

A

B) Explanation
Of the listed plans, only a traditional defined benefit pension plan is available to Bernie and Tim’s business. A Section 403(b) plan may only be adopted by a Section 501(c)(3) organization. An eligible Section 457 plan may only be adopted by a private, tax-exempt organization or a state or local governmental organization. A partnership cannot use a stock bonus plan because this form of business does not issue stock. Notice how subtlety the form of business was introduced in the question. The form of business is very important in selecting a retirement plan.

LO 8.1.2

381
Q

What is a client’s personal retirement plan objective if she selects a qualified defined benefit pension plan as her company’s retirement plan?

I. An approaching retirement need
II. A desire to maximize tax benefits to the business
III. The ability to selectively choose participants in the plan
IV. Maximize retirement contributions for older workers

A) I and IV
B) I, II, III, and IV
C) II and III
D) I, II, and IV

A

D) Explanation
Only Statement III is incorrect. A qualified pension plan is subject to nondiscrimination rules and the client would not be able to cover only select participants.

LO 8.1.2

382
Q

Brad Elberly has been the sole owner and operator of Woodmasters Inc. for the past 15 years. Brad is age 45, and his salary from the business is $130,000. Brad and his wife, Laura, want to retire when Brad is age 65. Relevant information regarding the business is summarized below:

Financial performance fluctuated over the first 10 years.
Cash flow and profits have stabilized during the past five years and are expected to show modest but consistent growth in the future. Excess cash flow of approximately $150,000 is expected to be available this year. Future years should be about the same. Brad has expressed some concern about the company’s outdated equipment and is considering renovating the plant and replacing the outdated equipment over the next five years. The total cost should be about $300,000.
Total compensation for all employees (including Brad) is $245,000.
The four full-time rank-and-file employees range from age 19 to age 38 and have been with Woodmasters for periods ranging from four months to six years. Age and service information is shown below:
Employee Age Completed Years of Service Compensation
Brad 45 15 years $130,000
Beth 38 6 years $40,000
Todd 27 6 months $25,000
Carol 30 2 years $28,000
Jim 19 4 months $22,000
Assuming that Brad and Laura need to start saving approximately 17% of Brad’s salary to build their retirement fund, which one of the following statements describes the most appropriate qualified plan(s) for Woodmasters, assuming Brad wants to minimize the plan cost?

A) A money purchase plan that maximizes the allowable contribution limits would be appropriate.

B) Any type of qualified plan, either defined contribution or defined benefit, would be appropriate.

C) An integrated, age-weighted profit sharing plan would be appropriate.

D) A safe harbor 401(k) plan would be appropriate.

A

D) Explanation
Selecting a plan means selecting the best plan for the owner (i.e., the owner gets the desired or needed amount at the minimum cost for the other employees). Any retirement plan installed by the company would have to be affordable, considering the need for large cash outlays to update company equipment. The safe harbor 401(k) plan is the best choice since it not only fits within the contribution requirements with flexibility, but also meets the objective of minimizing cost. By increasing his compensation and using the increase as his deferral, Brad will get a disproportionate share of the contribution, and because of his much higher compensation, the nonelective contribution will be skewed in his favor. Thus, he can meet his personal contribution requirement while paying significantly less for the other participants.

A 401(k) safe harbor plan will allow Brad to defer $19,500 in 2021. The required matching contribution is dollar-for-dollar of the first 3% of deferral and $.50 on the dollar for the next 2%. Brad needs only 17% of his current compensation, so we would choose the safe harbor 401(k), and not a defined benefit plan, since we can achieve the desired savings with a contribution that is less than 25%. (This calculation is not required, but knowledge of the safe harbor plan is.)

LO 8.1.2

383
Q

Joe, age 52, has just started a consulting company. He currently employs six people, who range in age from 22 to 31 years old. Joe estimates the average employment period for his employees will be approximately three years and would like to implement a retirement plan that will favor older participants while including an appropriate vesting schedule. In addition, Joe would like the employees to bear the risk of investment performance within the plan. Which of the following plans is most appropriate for Joe’s company?

A) SIMPLE 401(k)
B) Cash balance pension plan
C) SEP plan
D) Target benefit pension plan

A

D) Explanation
A target benefit pension plan is likely most appropriate. It would permit Joe to favor older participants and allow for a vesting schedule. A cash balance pension plan does not favor older participants and provides employees with a guaranteed rate of return on investments (thus, not transferring the risk of investment performance to the employees). SEP plans and SIMPLE 401(k) plans both provide for 100% immediate vesting of employer contributions.

LO 8.1.2

384
Q

XYZ Inc. has 80 employees in the current year and is expected to employ the same number next year. They are considering the adoption of a retirement plan next year. The objectives are to use elective deferrals by employees with an appropriate match and immediate vesting. Which of the following plans would be appropriate?

I. Traditional defined benefit pension plan
II. Section 403(b) plan
III. SIMPLE 401(k)
IV. SIMPLE IRA

A) I and III
B) I only
C) I, II, III and IV
D) III and IV

A

D) Explanation
Traditional defined benefit pension plans do not permit employee contributions through elective deferrals. Section 403(b) plans are for 501(c)(3) tax-exempt organizations, and there is no indication that this is a tax-exempt organization. SIMPLE plans permit employee contributions through elective deferrals, matching, and immediate vesting.

LO 8.1.2

385
Q

The employee bears the investment risk in all of the following types of retirement plans except

A) money purchase pension plans.
B) target benefit pension plans.
C) cash balance pension plans.
D) traditional profit-sharing plans.

A

C) Explanation
Employees bear the investment risk in defined contribution plans, and employers bear the investment risk in defined benefit plans. Cash balance pension plans are defined benefit plans; all the other answer choices are defined contribution plans.

LO 8.2.1

386
Q

Sharon plans to retire next year and begin taking distributions from her traditional IRA. Her investment objectives are low risk, safety of principal, and liquidity. She is content with minimal rates of return.

Which of the following investments is most suitable for her IRA?

A) Dividend-paying common stock
B) Money market instruments
C) Corporate bonds
D) Nondividend-paying common stock

A

B) Explanation
Money market instruments provide the low risk, safety, and liquidity Sharon is seeking for her IRA. Corporate bonds and stocks do not provide safety of principal.

LO 8.2.1

387
Q

Which of the following constitutes unrelated business taxable income (UBTI) to a qualified plan trust?

A) Rents from real property investments
B) Reinvested dividends from mutual funds
C) Dividends from common stock purchased on margin
D) Interest payments from corporate bonds

A

C) Explanation
Unrelated business taxable income (UBTI) is gross income generated by a qualified plan trust that is not related to the function that is the basis for the trust’s income tax exemption. In addition, the trust is generally prohibited from incurring debt. Therefore, dividends declared on common stock purchased on margin are treated as UBTI.

LO 8.2.1

388
Q

Which of the following statements accurately describes a reason for the suitability of an asset class in a qualified retirement plan portfolio?

A) Common stocks offer capital appreciation and are relatively low risk.

B) Bonds are useful for funding future fixed obligations and offer protection against inflationary pressures.

C) Money market investments provide liquidity and a high real rate of return.

D) Mutual funds provide diversification and offer a competitive rate of return.

A
D) Explanation
In determining the suitability of an asset class in a retirement plan portfolio, mutual funds provide diversification and provide a comparable historical rate of return to that of equities (of course, depending on the investment objective and composition of the particular fund).

LO 8.2.1

389
Q

In the allocation of assets to determine the best portfolio composition for a qualified plan, what is a major factor to be considered?

A) The type of plan and who bears the investment risk
B) The maximum deductible contribution amount permitted under the plan
C) The reputation and experience of the plan sponsor
D) The administration requirements associated with the plan

A

A) Explanation
A major factor to be considered in the asset allocation process of qualified plans is the type of plan (defined contribution or defined benefit) and who bears the investment risk (the employee or employer). Very broadly, a more conservative allocation is appropriate for a defined benefit plan than in the defined contribution approach.

LO 8.2.1

390
Q

Which of the following can be used as a funding vehicle for tax-sheltered annuities (TSAs)?

I. Stocks
II. Bonds
III. Annuities
IV. Regulated investment company shares held within a custodial account

A) II, III, and IV
B) III and IV
C) I and II
D) I, II, III, and IV

A

B) Explanation
The primary investment vehicles for Section 403(b) plans are annuities or mutual funds held in a custodial account, although funds can also be used to purchase life insurance. Section 403(b) accounts can be invested in annuities or mutual funds held in a custodial account. Funds also can be used to purchase life insurance. Neither stocks nor bonds can be used as an investment for a Section 403(b) plan.

LO 8.2.1

391
Q

Alice participates in a qualified retirement plan at work. The plan provides Alice with life insurance. If Alice dies, which of the following statements correctly describes the income tax treatment of the life insurance death benefit paid to Alice’s beneficiary?

A) The entire death benefit is income tax free to the beneficiary.

B) The pure insurance element of the death benefit is income tax free to the beneficiary.

C) The beneficiary must pay income tax on the entire death benefit.

D) The beneficiary must pay income tax plus a 10% penalty on the entire death benefit.

A

B) Explanation
The beneficiary receives the pure insurance element of the death benefit income tax free.

LO 8.2.2

392
Q

Life insurance may be a suitable investment for all of the following retirement plans except

A) money purchase pension plans.
B) defined benefit pension plans.
C) SEP plans.
D) profit-sharing plans.

A

C) Explanation
Life insurance cannot be held in any type of IRA, including SEPs. Qualified retirement plans and Section 403(b) plans can purchase life insurance if they comply with the incidental benefit rules.

LO 8.2.1

393
Q

Which of the following investments would be the least suitable for a qualified retirement plan?

A) Guaranteed investment contract (GIC)
B) Real estate investment trust (REIT)
C) Municipal bond fund
D) Equity mutual fund

A

C) Explanation
The tax-exempt nature of qualified retirement plans must be considered in determining if a specific type of investment is appropriate. Municipal bonds are tax-free investments. Because a qualified plan is already tax exempt, there would be no reason to utilize them in plan assets.

LO 8.2.2

394
Q

Richard participates in a defined benefit pension plan at his place of employment. His projected monthly benefit under the plan is $1,000. If the plan provides life insurance for Richard, the death benefit payable under the policy is limited to

A) $1,000.
B) $25,000.
C) $10,000.
D) $100,000.

A

D) Explanation
Defined benefit pension plans use the 100 times test for determining whether they comply with the incidental benefit rules. Under this test, the death benefit cannot exceed 100 times the participant’s projected monthly benefit.

LO 8.2.2

395
Q

Which of the following statements regarding the incidental death benefit test for life insurance inclusion in a qualified plan is CORRECT?

I. For defined contribution plans, no more than 25% of the pension plan’s assets may be in the form of universal life insurance cash value.

II. For defined benefit pension plans, the life insurance death benefit cannot exceed 100 times the expected monthly benefit for an employee.

A) I only
B) II only
C) Both I and II
D) Neither I nor II

A

B) Explanation
For defined contribution plans, no more than 25% of the employer contributions (not plan assets) can be used to purchase universal life or term insurance. No more than 50% of the employer contributions in a defined contribution plan can be used to purchase whole life insurance.

LO 8.2.2

396
Q

Which of the following statements regarding the anticipated effective income tax rate a planner should use for required retirement plan distributions is CORRECT?

I. The projected rate should be based only on a blend of current federal and state marginal income tax rates.

II. The projected rate should be based only on current federal marginal income tax rates.

III. Precisely predicting future income taxes is not feasible.

IV. A planner should only use before-tax rate of return assumptions on retirement plan distributions.

A) IV only
B) I and III
C) II only
D) II and III

A

B) Explanation
Before retirement, the planner may use a before-tax rate of return in the assumptions, particularly if tax-advantaged savings vehicles (such as a traditional or Roth IRA) are used in the planning process. However, at the time of either optional or required retirement plan distributions, the client’s anticipated effective income tax rate is very important. This rate should be a blend of the client’s federal and state marginal income tax rates but should be projected based only on current rates because precisely predicting future income tax rates is not feasible.

LO 8.3.1

397
Q

Which type of assumed annual rate of return is frequently used in retirement planning calculations?

A) Black-Scholes model
B) A flat annual return based on past performance
C) Quantum simulation
D) Consumer Price Index (CPI)

A

B) Explanation
Most planners use a flat annual rate of return based on past performance. The Black-Scholes model is a stock option valuation model. The Consumer Price Index (CPI) measures inflation.

LO 8.3.1

398
Q

Which of the following events should trigger a recalculation of the retirement needs analysis?

I. Marriage
II. Employment change
III. Change in FICA rate
IV. Significant income change

A) III only
B) I, II, and IV
C) I and II
D) III and IV

A

B) Explanation
The retirement needs analysis should be recalculated any time there is a change in the client’s life such as marriage, divorce, births, deaths, employment and income, or health. Temporary changes in tax laws will not impact the retirement needs over the long term.

LO 8.3.1

399
Q

A planner may use a before-tax rate of return in making projections regarding the preretirement investment returns for all of the following assets except

A) Roth IRA.
B) Section 401(k) plan.
C) mutual fund held outside a qualified retirement plan.
D) traditional IRA.

A

C) Explanation
Using a before-tax rate of return to project investment performance is appropriate for tax-advantaged assets, such as qualified plans and IRAs, because the earnings on these assets are not taxed each year as they accumulate.

LO 8.3.1

400
Q

Lisa has accumulated assets of $350,000 that she wishes to dedicate to her retirement. Inflation is anticipated to average 2% over the next 20 years. She plans on retiring in 15 years. What would be the value of her fund at retirement if Lisa can average a 5% after-tax rate of return on her accumulated assets?

A) $520,082
B) $467,035
C) $928,654
D) $727,625

A

D) Explanation
This is a future value calculation.

PV = –$350,000

i = 5 (after-tax rate of return)

n = 15 (years to retirement)

FV = $727,625

LO 8.3.2

401
Q

Paul estimates he will need a $75,000 annual income in today’s dollars when he retires 10 years from now. He assumes a 3% annual rate of inflation, a 5% after-tax rate of return on his investments, and a 20-year retirement period. What lump-sum amount should Paul have accumulated over the next 10 years to support his retirement income need?

A) $1,689,612
B) $1,657,429
C) $2,015,880
D) $1,257,227

A

A) Explanation
His total retirement fund needed to support his standard of living is $1,689,612, calculated as follows:

The client’s first-year retirement income need is $100,794.

PV = –$75,000

i = 3

n = 10

FV = $100,794

The total capital required to support this need for 20 years is $1,689,612. In BEGIN mode (the client will make annual withdrawals at the beginning of each year)

PMT = $100,794

n = 20

i = 1.9417 [(1.05 ÷ 1.03) – 1] × 100

PVAD = –$1,689,612

LO 8.3.2

402
Q

Jeanette uses the serial payment approach to calculate the amount she must save each year to accumulate her desired retirement income fund. She assumes an annual inflation rate of 4%, and an annual investment rate of return of 7%. If she determines that she must save $10,000 in the first year, how much must she save in the second year to meet her goal?

A) $10,400
B) $10,700
C) $10,288
D) $10,000

A

A) Explanation
Under the serial payment approach, the first-year savings amount is computed and then increased each year by the inflation rate. In this example, the amount in the second year is $10,000 × 1.04, or $10,400.

LO 8.3.2

403
Q

Carol has an additional retirement need of $30,000 annually in today’s dollars. She will retire in 15 years and projects a retirement period of 20 years. Carol believes she can achieve a 6% after-tax rate of return and is assuming a 4% annual rate of inflation. Using the level payment approach, how much will Carol need to save in a single annual payment at the end of each year to fund her retirement need?

A) $34,044.67
B) $30,000.00
C) $38,973.65
D) $36,767.56

A

C) Explanation
Carol will need to make 15 level annual payments of $38,973.65 at the end of each year.

Here is how to work this problem using the regular TVM keystrokes.

What will it take to have $30,000 of today’s purchasing power in 15 years?

$30,000, +/-, PV

4, I

15, N

0, PMT

FV Answer: $54,028.3052

How much needs to be in the account at retirement to fund 20 years of serial payments at the first of each year?

+/-, +/-, PMT (Hitting the +/- button twice before PMT allows the entire FV of 54,028.3052 to be entered as the PMT quickly and to nine decimal places, The second +/- makes the value positive again)

Shift, MAR (To get into the Begin Mode)

0, FV

[(1.06 ÷ 1.04) -1] × 100 = 1.9231, I

20, N

PV Answer: -$907,149.5440

How much needs to be saved at the end of each year to have $907,149.5440?

+/-, $907,149.5440, FV (Hitting the +/- here allows the calculator to take the answer and turns the number positive)

15, N

6, I

0, PV

Shift MAR (To be in the End Mode.)

PMT Answer: $38,973.65

LO 8.3.2

404
Q

Paul estimates he will need $75,000 of annual income in today’s dollars when he retires 10 years from now. He assumes a 3% annual rate of inflation, a 5% after-tax rate of return on his investments, and a 20-year retirement period. Using the serial payment approach, how much will Paul need to save in a single annual payment at the end of the first year to fund his retirement need?

A) $115,124.27
B) $118,578.00
C) $100,794.00
D) $120,880.49

A

B) Explanation
Paul will need to make a serial payment of $118,578.00.

Here is how to calculate this type of problem using the regular TVM keystrokes:

What will it take in ten years to have $75,000 of today’s purchasing power?

75,000, +/-, PV

3, I

10, N

FV Answer: $100,793.7285

What does Paul need to have in his account when he retires to produce a serial payment of $100,793.7285 at the beginning of the year for 20 years?

+/-, +/-, PMT (This enters $100,793.7285 into the PMT as a positive value. The first +/- allows the calculator to accept the number. The second +/- makes it a positive number.)

Shift, MAR (To get into the Begin mode.)

20, N

0, FV

[(1.05 ÷ 1.03) – 1] × 100 = 1.9417, I

PV Answer: $1,689,607.5595

What is $1,689,607.5595 in 10 years with inflation taken out of it? This step is critical because it prepares for the serial payment from today to retirement by removing the inflation from the dollar goal in 10 years. Serial payments account for inflation by adjusting the interest rate. Without expressing the FV in today’s dollars, inflation will be double counted for the investment period.

+/- $1,689,607.5595, FV (Only one +/- is needed here because the first +/- allows the calculator to accept the value and the PV was negative so after one +/- the new value will for FV will be positive.)

10, N

0, PMT

3, I

PV Answer: -$1,257,226.7036

What is the first serial payment required at the end of the first year to have $1,257,226.7036 in 10 years?

+/-, FV (This enters the result from aboe and changes the sing to positive $1,257,226.7036

Shift, MAR (To get in the End mode.)

[(1.05 ÷ 1.03) – 1] × 100 = 1.9417, I

0, PV

10, N

PMT Answer: $115,124.2738

This answer is in today’s dollars, but the first payment will be made in a year, so…

$115,124.27 × 1.03 = $118,578

LO 8.3.2

405
Q

Case Study question
Kaitlyn is interested in starting a SEP to help them save for their retirement. If she continues making $50,000 of self-employment income, what is the maximum she could contribute to a SEP each year? Assume the self-employment tax would be $7,065 each year.

A) $42,935
B) $50,000
C) $ 9,294
D) $12,500

A

C) Explanation
SEP contributions for self-employed owners require two adjustments. First, her net self-employment income after deducting half of her self-employment income must be calculated. $50,000 − ($7,065 ÷ 2) = $46,467.50. Second, the contributions rate must be adjusted by dividing the rate by 1 + the contribution rate. The maximum rate for a SEP is 25%. Thus the adjusted contribution rate is 20% (.25 ÷ 1.25). Now the adjusted contribution rate is multiplied times the adjusted self-employment income. In this case, .20 × $46,467.50 = $9,293.50. This is rounded up to $9,294.

406
Q

The Dawsons are considering how to pay for the last year of the twins’ college tuition. How much of a loan could they take from Dillon’s 401(k)? They have never borrowed from his retirement account before.

A) $50,000
B) $43,500
C) $20,250
D) $87,000

A

B) Explanation
Retirement account loans are generally 50% of the vested balance minus the highest loan amount in the previous 12 months. In this case there is no previous loan amount in the last 12 months, so they have access to $43,500 ($87,000 ÷ 2).

LO 9.1.1

407
Q

The Dawsons are considering how to pay for the last year of the twins’ college tuition. Despite your objections, they have decided to take the remaining $23,000 from one of their retirement accounts. Which retirement account should the money be taken from if their goal is to maximize cash flow this year?

A) Kaitlyn’s 403(b)
B) Kaitlyn’s Roth IRA
C) Dillon’s traditional IRA
D) Dillon’s 401(k) has a hardship withdrawal

A

B) Explanation
Kaitlyn’s Roth IRA will cause the least disruption to their current cash flow. The two employer retirement accounts will be subject to both income taxes and the 10% early withdrawal penalty. Neither IRA will be subject to the 10% penalty because qualified higher education expenses are an exception to the penalty. Thus, the answer would have to be one of the IRAs. There is no indication that the traditional IRA has ever received nondeductible contributions. Thus, all withdrawals from the traditional IRA will be taxable income. Withdrawals from the Roth IRA will first be accounted for as coming from contributions and then conversions. These withdrawals will not be subject to income tax. Thus, withdrawing the money from the Roth IRA will present a lower income tax bill. In fact, since she has contributed $35,000 to her Roth IRA over the years, there would be no income tax bill if she withdrew the $23,000 needed for the final college year. Pulling money from the Roth IRA might not be the best decision long-term, but it is the best option of the given choices to meet the client’s current goal.

LO 9.1.1

408
Q

With their current MAGI of $120,000, how much can the Dawsons contribute to deductible IRAs for 2021?

A) $12,000
B) $6,000
C) $7,500
D) $1,200

A

C) Explanation
Their MAGI of $120,000 is inside the phaseout range for active participants who are married filing jointly. Dillon is an active participant at work. Thus, he may only deduct $1,500 in 2021. [($125,000 − $120,000) ÷ $20,000] × $6,000 = $1,500. Kaitlyn is self-employed with no retirement plan and their MAGI is well below the $198,000 start of the phaseout range for those married filing jointly who are not active participants. Thus, she can deduct up to $6,000 of IRA contributions. That brings their total deductible IRA contributions to $7,500 for 2021.

LO 9.1.1

409
Q

If the Dawsons were to both die in a car accident on April 1, 2021, how long would their beneficiaries be able to stretch their IRAs?

A) Their full IRA accounts would need to be distributed by December 31, 2021.

B) Their accounts would be immediately disgorged to their estate.

C) Their full IRA accounts would need to be distributed by December 31, 2026.

D) Their accounts would be split between their two children and each would have his own RMD.

A

C) Explanation
An estate cannot be a designated beneficiary. Thus, each of their IRAs would have to take the entire balance by the end of the fifth year following the year of death. 2021 is the year of death. 2022 is the first year following the year of death. That makes 2026 the fifth year following the year of death.

LO 9.1.1

410
Q

The Dawsons have a goal of $100,000/year of income in today’s dollars for their retirement while they are both alive. They chose to estimate their retirement at 30 years and use the capital utilization method. Together their Social Security estimates total $46,000/year (in today’s dollars). Approximately how much would they need in their retirement and investment accounts when they retire when he is 67 and she is 65? They assume they will earn 8% prior to retirement and 6% after they have retired. They assume inflation will average 3% through their lives.

A) $1,238,770
B) $1,877,092
C) $1,931,764
D) $2,182,440

A

C) Explanation
The first step is to estimate their actual withdrawal for their first year of retirement. Their goal is $100,000 in today’s dollars, but Social Security is assumed to provide $46,000 in today’s dollars for their 30-year retirement. Thus, they need to fund $54,000/year in today’s dollars. With inflation averaging 3% for 19 years, this will take $94,689.33 to produce $54,000 of today’s purchasing power. The second step is determining how much principal would be required at retirement to produce a serial payment starting at $94,689.33 and lasting for 30 years assuming a 6% investment return and 3% inflation. The answer is $1,931,764. This is calculated using the following factors: 30 for N, 0 for the FV, 2.9126 for the inflation adjusted interest rate and $94,689.33 has the initial payment in the Begin Mode. $1,877,092 is the result in the End Mode. Small differences in these final numbers are the result of using differences in rounding. These numbers were computed using the full values produced at each step. The HP-10bII used nine decimal places when it calculates a number (even if fewer decimal places are shown). For example, the inflation-adjusted interest rate calculated using the formula shows 2.9126%. However, by pressing Shift, =, 9, you will see the actual interest rate used is 2.9126211359. On a test, these small differences will not change the answers.

LO 9.1.1

411
Q

Assume the Dawsons’ retirement goal is $1,100,000 in today’s dollars when Dillon turns 67. They assume they will earn 8% prior to retirement and 6% after they have retired. They assume inflation will average 3% through their lives. What would their first year’s serial payment be if they saved at the end of each year? Take into account that they have $255,000 in their current retirement accounts.

A) $15,417
B) $16,164
C) $15,694
D) $14,968

A

B) Explanation
$16,164. The FV is $1,100,000. N is 19 years. The inflation adjusted interest rate shows 4.8544% but if you use the formula and immediately press I/YR, it will enter 4.854368932. The PV is -$255,000. Pressing PMT in the end mode will give you $15,694. However, the payment will not be made for a year. Thus, you must multiply times 1.03 to account for the first year of inflation.

LO 9.1.1

412
Q

What is the maximum salary reduction contribution Dillon could make to his 401(k) for 2021?

A) $ 6,000
B) $16,500
C) $19,500
D) $13,500

A

C) Explanation
The maximum elective deferral into a 401(k) for someone under age 50 is $19,500 in 2021.

LO 9.1.1

413
Q

Dillon’s 401(k) has just been changed to allow Roth elective deferrals. Which of the following does a Roth 401(k) plan have in common with a Roth IRA?

I. They both are phased out for higher income earners.

II. Withdrawals after age 59½ are tax-free if the account is at least five years old.

III. Contributions are not deductible.

IV. They are both subject to RMDs for retirees starting after attaining age 70½.

A) I and IV
B) I and II
C) II and III
D) II and IV

A

C) Explanation
Although Roth IRAs have an additional reason for qualified distributions (up to $10,000 of first-time homebuyer expenses), a distribution from either type of Roth account after five years and over age 59½ would be a qualified distribution. No type of Roth contribution is deductible. Roth IRAs are not subject to RMDs while the original owner is alive, but employer Roth accounts for retirees are subject to the normal RMD rules. Roth IRAs are phased out for higher earners, but not for employer Roth accounts.

LO 9.1.1

414
Q

According to Social Security, Dillon’s Social Security retirement benefit at age 67 (his FRA) would be $2,200. What would his Social Security benefit be if he starting drawing his Social Security on his 65th birthday (24 months early)?

A) $2,552
B) $2,200
C) $1,907
D) $1,650

A

C) Explanation
$1,907. Social Security retirement benefits for workers are reduced five-ninths of 1% for the first 36 months and five-twelfths of 1% for any months in excess of 36 months. Dillon would be starting 24 months early, so 5/9 x .01 x 24 = 13.333% reduction. Thus, he would be receiving 86.667% of $2,200.

LO 9.1.1

415
Q

The Department of Labor (DOL) issues

A) guaranty insurance.
B) private-letter rulings.
C) rulings, including prohibited transaction exemptions (PTEs).
D) approval of plan documents in the summary plan description.

A

C) Explanation
The answer is rulings including prohibited transaction exemptions (PTEs). The Department of Labor issues advisory opinions and rulings (including prohibited transaction exemptions) similar to private-letter rulings, which are issued by the IRS. The DOL does not issue guaranty insurance. The summary plan description is required by the DOL, but the DOL does not approve plan documents. The IRS approves plan documents.

LO 1.1.1

416
Q

Pension Benefit Guaranty Corporation (PBGC) insurance coverage is required for which of the following plans?

I. Traditional defined benefit pension plan
II. Target benefit pension plan
III. Money purchase pension plan
IV. Profit-sharing plan

A) III and IV only
B)I only
C) I and IV only
D) I and II

A

B) Explanation
The answer is I only. Money purchase pension plans, profit-sharing plans, and target benefit pension plans do not require PBGC insurance because they are forms of defined contribution plans. Only defined benefit pension plans (traditional defined benefit plans and cash balance plans) require the payment of PBGC insurance premiums.

LO 1.1.1

417
Q

A qualified plan is

I. a company-sponsored retirement plan with benefits guaranteed by the Employee Retirement Income Security Act (ERISA).

II. a tax-efficient way to save for retirement.
only applicable for firms with 50 or more employees.
considered a plan that benefits highly compensated employees only.

A) II only
B) II, III and IV
C) II and IV
D) I and III

A

A) Explanation
The answer is II only. ERISA does not guarantee plan benefits; the Pension Benefit Guaranty Corporation (PBGC) guarantees benefits in defined benefit pension plans. A qualified plan is a tax-efficient way to save for retirement. Qualified plans may be established for firms with as few as one employee. Qualified plans also benefit non-highly compensated employees.

LO 1.2.1

418
Q

Which of the following are examples of qualified retirement plans?

I. Cash balance plan
II. Section 401(k) plan
III. Section 403(b) plan
IV. Employee stock ownership plan (ESOP)

A) I and III
B) I, II, and IV
C) III and IV
D) I, II, III, and IV

A

B) Explanation
The answer is I, II, and IV. A Section 403(b) plan is a tax-advantaged plan but not an ERISA-qualified retirement plan. While tax-advantaged plans are very similar to qualified plans, there are some minor differences. For example, a tax-advantaged plan is not allowed to have net unrealized appreciation (NUA) treatment. They are also not allowed to offer 10-year forward averaging or special pre-1974 capital gains treatment. Tax-advantaged plans also have less restrictive nondiscrimination rules. Otherwise they are very similar to qualified plans.

LO 1.2.1

419
Q

Which of the following are minimum coverage tests for qualified retirement plans?

I. Nondiscrimination test
II. Average benefits percentage test
III. Ratio test
IV. Maximum compensation test

A) II and III
B) II, III and IV
C) I, II and III
D) I and II

A

A) Explanation
The answer is II and III. The two minimum coverage tests for qualified retirement plans are the average benefits percentage test and the ratio test. In order to be qualified, a retirement plan must meet at least one of these tests if the plan does not meet the percentage (safe harbor) test.

LO 1.3.1

420
Q

Roderick Manufacturing maintains a qualified defined benefit pension plan. There are 100 eligible employees working for the company. What is the minimum number of employees the retirement plan must cover to satisfy the 50/40 test?

A) 50
B) 90
C) 40
D) 70

A

C) Explanation
The answer is 40. Under the 50/40 test, a defined benefit plan must cover the lesser of 50 employees or 40% of all eligible employees. In this case, the lesser of 50 employees or 40% of all eligible employees (100) is 40 employees. One way to remember the 50/40 test is the phrase “people before percentages” (50 people or 40%). Also, note that there are no qualifiers to the types of people. It is not 50 non-highly compensated people. It is just 50 employees who work for Roderick Manufacturing.

LO 1.3.1

421
Q

Which of the following statements regarding the coverage rules for qualified plans is (are) CORRECT?

I. The coverage tests for qualified plans include the percentage test, the ratio test, and the average contribution percentage test.

II. A retirement plan can cover any portion of the workforce, provided it satisfies one of the three coverage tests under Section 410(b).

A) Neither I nor II
B) II only
C) Both I and II
D) I only

A

B) Explanation
The answer is II only. Statement I should list the third coverage test as the average benefit percentage test, not the average contribution percentage test.

LO 1.3.1

422
Q

Several years ago, Greener Grass Company implemented a traditional defined benefit plan. According to the plan document, the employer must contribute an annual amount that will provide the employees with a specified benefit at retirement. Which of the following events would be expected to decrease the employer’s annual contribution to a traditional defined benefit pension plan using a percentage for each year of service benefit formula?

I. Inflation is higher than expected.
II. Benefits are cost of living adjusted.
III. Forfeitures are higher than anticipated.
IV. The investment returns of the plan are greater than expected.

A) II, III, and IV
B) III and IV
C) I and III
D) II and IV

A

B) Explanation
The answer is III and IV. Defined benefit pension plan contributions decrease because of the events described in statements III and IV. Statements I and II are incorrect. Inflation would likely cause salaries and plan expenses to increase, thereby causing contributions to increase. Likewise, benefits that are adjusted for the cost of living would result in greater employer contributions, not less.

LO 1.3.2

423
Q

Which of the following statements regarding integrating a plan with Social Security are FALSE?

I. Only the excess method can be used by a defined benefit pension plan.

II. The maximum increase in benefits for earnings above the covered compensation level is 26.25% for a defined benefit pension plan.

III. Because there is a disparity in the Social Security system, all retirement plans are allowed integration with Social Security.

IV. Under the offset method, a fixed or formula amount approximates the existence of Social Security benefits and reduces the plan formula.

A) I and III
B) III and IV
C) I, II and III
D) II and IV

A

A) Explanation
The answer is I and III. Statement I is incorrect because the excess method may only be used by a defined contribution plan. A defined benefit pension plan may use either the excess method or the offset method for Social Security integration. Statement III is incorrect because not all retirement plans may be integrated with Social Security.

LO 1.3.3

424
Q

Which of the following retirement plans can be integrated with Social Security?

I. Profit-sharing plan

II. Simplified employee pension (SEP) plan

III. Money purchase pension plan

IV. Defined benefit pension plan

A) I, II, III, and IV
B) IV only
C) II and III
D) I, III, and IV

A

A) Explanation
The answer is I, II, III, and IV. All these plans may be integrated with Social Security. Employee stock ownership plans (ESOPs), savings incentive match plan for employees (SIMPLEs), and salary reduction SEPs (SARSEPs) are not permitted to use integration. Also, employee elective deferrals and employer matching contributions cannot be integrated.

LO 1.3.3

425
Q

Which of the following statements regarding prohibited transactions is CORRECT?

I. The lending of money or other extension of credit between the plan and a party in interest (outside of nondiscriminatory retirement plan loans based on the participant’s account balance) is not a prohibited transaction.

II. One category of prohibited transactions involves self-dealing.

III. One category of prohibited transactions bars a fiduciary from causing the plan to engage in a transaction if the fiduciary knows or should know that such a transaction constitutes a direct or indirect involvement between the plan and the parties in interest.

IV. One category of prohibited transactions involves the investment in the sponsoring employer’s stock or real property above certain limits.

A) I and IV
B) II and III
C) III and IV
D) II, III, and IV

A

D) Explanation
The answer is II, III, and IV. Statement I is incorrect. The sale, exchange, lending, or leasing of any property between the plan’s assets and a party in interest is a prohibited transaction. A plan participant can take a retirement plan loan based on the participant’s balance, but the plan cannot make a loan of the general plan assets to a party in interest.

LO 1.4.1

426
Q

Which of the following is one of the differences between defined benefit pension plans and defined contribution plans?

A) A guaranteed retirement benefit is the goal of a defined contribution plan, while a guaranteed contribution is the focus of a defined benefit pension plan.

B) Accounts are commingled in a defined contribution plan, while a defined benefit pension plan maintains separate accounts for each participant.

C) Investment risk is borne by the employer in a defined benefit pension plan, whereas the employee bears the risk in a defined contribution plan.

D) A defined contribution plan has a benefit limit, whereas a defined benefit pension plan has a contribution limit.

A

C) Explanation
The answer is investment risk is borne by the employer in a defined benefit pension plan, whereas the employee bears the risk in a defined contribution plan. A defined benefit pension plan has a benefit limit, whereas a defined contribution plan has a contribution limit. Investment risk is borne by the employer in a defined benefit pension plan, whereas the employee bears the risk in a defined contribution plan.

Accounts are commingled in a defined benefit pension plan, and a defined contribution plan maintains separate accounts for each participant. A guaranteed retirement benefit is the goal of a defined benefit pension plan, while a guaranteed contribution is the focus of a defined contribution plan.

LO 2.1.1

427
Q

A fully insured Section 412(e)(3) pension plan is funded exclusively by

A) municipal bonds.
B) Treasury bonds.
C) blue-chip stocks.
D) cash value life insurance or annuity contracts.

A

D) Explanation
The answer is cash value life insurance or annuity contracts. A fully insured 412(e)(3) pension plan is funded exclusively by cash value life insurance or annuity contracts. Using insurance as a funding vehicle ensures the payment of a death benefit to plan beneficiaries.

LO 2.2.2

428
Q

Under a profit-sharing plan,

A) the employer bears investment risk.
B) the company has flexibility as to annual funding.
C) the company must make annual contributions.
D) up to 25% of the plan’s assets can be invested in the employer’s stock.

A

B) Explanation
The answer is the company has flexibility as to annual funding. Pension plans can invest up to 10% only of plan assets in employer stock. Profit-sharing plans have no restrictions regarding investment in employer stock. The employer may deduct a contribution limited to only 25% of participating employees’ covered compensation. The employer must make substantial and recurring employer contributions, or the IRS will remove the plan’s qualified status. The employee bears investment risk.

LO 3.3.1

429
Q

Which of the following statements regarding fully insured Section 412(e)(3) plans is(are) CORRECT?

I. A fully insured plan is inappropriate for an employer who cannot commit to regular premium payments.

II. This type of plan is not required to be certified by an enrolled or licensed actuary.

III. All Section 412(e)(3) plans must meet minimum funding standards each plan year.

IV. A Section 412(e)(3) plan is a type of defined benefit pension plan.

A) II and IV
B) I, II, III, and IV
C) I and II
D) I, II, and IV

A

D) Explanation
The answer is I, II, and IV. Statement III is incorrect. Section 412(e)(3) plans must only meet minimum funding standards if there is a loan outstanding against the insurance policy funding the plan.

LO 2.2.2

430
Q

In a money purchase pension plan, forfeitures

I. revert to the plan.
II. may be used to reduce future employer contributions.
III. can be reallocated among the remaining plan participants.
IV. do not count against remaining participants’ annual additions limits.

A) I, II, III, and IV
B) I, II, and III
C) I and II
D) III and IV

A

B) Explanation
The answer is I, II, and III. Forfeitures count against the remaining participants’ annual additions limits.

LO 2.3.1

431
Q

Which of the following statements regarding plan forfeitures in a money purchase pension plan is(are) CORRECT?

I. Plan forfeitures may be used to reduce future employer contributions.

II. An employer may reallocate the plan forfeitures among the remaining plan participants, increasing their potential individual account balances but only up to the annual additions limit for each participant.

A) II only
B) Both I and II
C) I only
D) Neither I nor II

A

B) Explanation
The answer is both I and II. Both of these statements are correct for money purchase pension plans.

LO 2.3.1

432
Q

Bill’s employer maintains a target benefit pension plan. Bill is age 59. The plan was originally designed to benefit a 38-year-old key employee. There is also substantial turnover at Bill’s company. Which of the following statements is (are) CORRECT?

I. Bill knows exactly what retirement benefit to expect.

II. Bill’s retirement benefit is funded through elective deferrals.

III. Forfeitures are likely to be allocated equally to Bill and the 38-year-old key employee.

IV. Contributions to the plan are mandatory.

A) II, III, and IV
B) IV only
C) I only
D) II and IV

A

B) Explanation
The answer is IV only. Benefits depend on such plan’s account balances, and the final benefit amount is not guaranteed. Target benefit pension plans are funded by the employer, not through employee elective deferrals. Forfeitures in such plans are likely to be unequal as a result of unequal compensation.

LO 2.3.2

433
Q

All the following statements describe situations in which a target benefit pension plan would best suit the company except

A) the workforce has a few older, well-paid owner-employees and key employees compared to the younger and lower paid rank-and-file workers.

B) an employee census showing young owners and young rank-and-file employees.

C) recent economic success and the likelihood of continued success.

D) there is a desire to have a defined benefit plan without the resources to match that desire.

A

B) Explanation
The answer is an employee census showing young owners and young rank-and-file employees. A good candidate for a target benefit pension plan is a business that has an employee census showing older owners who are around 50 or older and younger rank-and-file employees. The other choices are additional characteristics that describe good candidates for target benefit pension plans.

LO 2.3.2

434
Q

Which of the following statements regarding profit-sharing plans is (are) CORRECT?

I. Profit-sharing plans are only suited for companies with predictable cash flows.

II. Company profits are required to make contributions to a profit-sharing plan.

III. Companies adopting a profit-sharing plan are required to make annual contributions to the plan.

IV. The maximum tax-deductible employer contribution to a profit-sharing plan is 25% of total covered employee compensation.

A) II and IV
B) IV only
C) II, III, and IV
D) I, II, and III

A

B) Explanation
The answer is IV only. Statements I, II, and III are incorrect. Profits are not required to be able to make a contribution to a profit-sharing plan. A current contribution may be made from retained earnings or current cash flow. Annual contributions are not required in a profit-sharing plan. Profit-sharing plans are suitable for companies with unstable earnings given that they have discretion over contributions.

LO 3.1.1

435
Q

Which of the following are considered profit-sharing plans?

I. Stock bonus plan
II. Traditional Section 401(k) plan
III. New comparability plan
IV. Employee stock option plan (ESOP)

A) I, II, and III
B) I and II
C) II, III, and IV
D) I, II, III, and IV

A

D) Explanation
The answer is I, II, III, and IV. All of these are considered profit-sharing plans. They don’t require fixed annual employer contributions and are not affected by the minimum funding standard requirements.

LO 3.1.1

436
Q

Armor Company has implemented an age-based profit-sharing plan. Under this plan,

A) the employer contribution is allocated in such a way that the benefit at retirement is guaranteed.

B) younger employees generally receive the greatest allocation.

C) compliance with the nondiscrimination (coverage) rules is tested in accordance with contributions rather than benefits.

D) allocations to participants are made in proportion to the participant’s age-adjusted compensation.

A

D) Explanation
The answer is allocations to participants are made in proportion to the participant’s age-adjusted compensation. A participant’s compensation is age-adjusted by multiplying the participant’s actual compensation by a discount factor based on the participant’s age and the interest rate elected by the plan sponsor. As a result, older employees generally receive the greatest allocation under an age-based profit-sharing plan. Nondiscrimination rules are tested in accordance with benefits rather than contributions. The final retirement benefit is not guaranteed in any type of profit-sharing plan.

LO 3.2.1

437
Q

John Irving, the 55-year old owner of ABC Corporation, wants to implement a new comparability plan. John’s salary is $150,000. The remaining eligible participant census is as follows:

Age	Salary
Employee A	35	$50,000
Employee B	33	$45,000
Employee C	54	$60,000
Employee D	41	$35,000
Employee E	43	$36,000
If John wants to contribute an aggregate total of $41,300 to the plan this year, what is the maximum amount John can contribute to the comparability plan for himself?

A) $15,000
B) $37,500
C) $49,000
D) $30,000

A

D) Explanation
The answer is $30,000. A new comparability plan will only satisfy the nondiscrimination rules if the plan design satisfies one of either of these:

Each eligible non-highly compensated employee (HCE) must receive an allocation of at least 5% of compensation.
If the plan provides for an allocation rate of less than 5%, the minimum allocation rate for the non-HCEs is one-third of the highest allocation rate under the plan.
John can contribute a maximum $30,000 (20%) of his salary to the plan while limiting his other employees to as little as 5% of salary. In this example, the total compensation of the eligible employees is $226,000. A 5% contribution for this group totals $11,300, leaving $30,000 of the total $41,300 for John’s benefit.

LO 3.2.1

438
Q

When is an employee stock ownership plan (ESOP) an appropriate choice for an employer to implement?

I. The employer is either a C or an S corporation.

II. Creating a market for the employer stock helps diversify the employer-owner’s stock portfolio.

III. The employer wishes to increase the company’s liquidity by pledging the stock for a loan in the name of the ESOP.

IV. The employer wishes to transfer ownership of the business to the employees.

A) I, II, III, and IV
B) II and III
C) I and IV
D) I and III

A

A) Explanation
The answer is I, II, III, and IV. All these statements regarding ESOPs are correct.

LO 3.2.2

439
Q

Total annual contributions to an individual participant in a traditional Section 401(k) plan are limited in 2021 to

A) $19,500.
B) $290,000.
C) the lesser of 100% of compensation, or $58,000.
D) $15,000.

A

C) Explanation
The answer is the lesser of 100% of compensation, or $58,000. Total annual contributions to a participant’s account are limited to the lesser of 100% of employee compensation, or $58,000 (2021) with only the first $290,000 (2021) of employee compensation considered in the contribution formula. The total contribution is made up of the worker contribution, the employer contribution and reallocated forfeitures. The worker contribution alone is limited to $19,500 in 2021 for those 49 and younger.

LO 3.3.1

440
Q

Ross Company has a traditional Section 401(k) plan. The actual deferral percentage (ADP) for all eligible non-highly compensated employees (non-HCEs) is 4%. What is the maximum ADP for the highly compensated employees (HCEs) group at Ross Company?

A) 4%
B) 5.7%
C) 5.25%
D) 6%

A

D) Explanation
The answer is 6%. The maximum ADP for HCEs at the Ross Company is 6%. To satisfy the ADP test, a traditional 401(k) plan must meet one of the following two tests.

The ADP for eligible HCEs must not be more than the ADP of all other eligible employees multiplied by 1.25. In this case, the non-HCEs averaged 4%. (4% × 1.25 = 5%)

The ADP for eligible HCEs must not exceed the ADP for other eligible employees by more than 2% (4% + 2% = 6%), and the ADP for eligible HCEs must not be more than the ADP of all other eligible employees multiplied by 2 (4% × 2 = 8%). This second test is a lesser-than test. The lesser number is 6%. Thus, 6% is the second test answer.

Thus, the two test answers are 5% and 6%. The plan only must pass one of these tests, and the HCEs want to know how much they can contribute, so the higher number always wins. That is 6% in this case.

LO 3.3.2

441
Q

Which of the following are true of the actual contribution percentage test (ACP) test for 401(k) plans?

I. The ACP test is not used unless a 401(k) plan has a match or allows employee after-tax contributions.

II. The ACP test uses the same two test structure and percentage rules as the ADP test.

III. The ADP test accounts only for employee deferrals.
The ACP test accounts for employer matching and after-tax contributions, but not pretax contributions and elective deferrals.

IV. If the ADP of the non-highly compensated employees is greater than 2% but less than or equal to 8%, then the maximum ADP of the highly compensated employees is 2% more than the ADP of the non-highly compensated employees.

A) II, III, and IV
B) I, II, and IV
C) I, II, III, and IV
D) I, III, and IV

A

C) Explanation
The answer is I, II, III, and IV. All statements are true.

LO 3.3.2

442
Q

To be eligible to adopt a SIMPLE 401(k), an employer may have no more than

A) 25 employees who earned more than $5,000 last year.

B) 150 employees who earned more than $5,000 last year.

C) 100 employees who earned at least $5,000 last year.

D) 50 employees who earned more than $5,000 last year.

A

C) Explanation
The answer is 100 employees who earned at least $5,000 last year. An employer with 100 or fewer employees who earned $5,000 or more during the preceding year may adopt a SIMPLE 401(k).

LO 3.3.3

443
Q

Napoleon Enterprises sponsors a SIMPLE 401(k) for its employees. Under the plan, the company matches employee contributions up to 3% of compensation. Which of the following statements about Napoleon Enterprises’ SIMPLE 401(k) is CORRECT?

A) Napoleon Enterprises can match as little as 1% of compensation for 2 out of 5 years.

B) Employees cannot make after-tax contributions to the plan.

C) Napoleon Enterprises’ contributions must be vested using either a 3-year cliff or 2-to-6-year vesting schedule.

D) Withdrawals made within 2 years of initial participation are subject to a 25% premature distribution penalty tax.

A

B) Explanation
The answer is employees cannot make after-tax contributions to the plan. Employee after-tax contributions are not allowed. All the other statements are incorrect:

Unlike SIMPLE IRAs, employers that sponsor SIMPLE 401(k)s cannot reduce the matching percentage to below 3%. Employer contributions to a SIMPLE 401(k) are 100% vested.

The 25% penalty applies only to SIMPLE IRAs.

LO 3.3.3

444
Q

Jill has decided to offer a retirement plan to her employees. She wants to implement a savings incentive match plan for employees (SIMPLE) and is trying to decide between a SIMPLE IRA and a SIMPLE 401(k). All of the following statements apply to both types of SIMPLE plans except

A) there is a 25% penalty for early distributions from a participant’s account within two years of entry into the plan.

B) SIMPLE assets may only be rolled over into another SIMPLE within the first two years of initial participation in the plan by a participant.

C) employer contributions are not subject to payroll taxes (FICA and FUTA).

D) SIMPLEs are not subject to the top-heavy rules that apply to qualified plans.

A

A) Explanation
The answer is there is a 25% penalty for early distributions from a participant’s account within two years of entry into the plan. Only early distributions from a SIMPLE IRA within the first two years of initial participation in the plan are subject to the 25% early withdrawal penalty tax.

LO 3.3.3

445
Q

Which of the following may be eligible for rollover treatment?

I. A total distribution from a Section 401(k) plan
II. A distribution from an IRA
III. The nontaxable portion of qualified plan distribution
IV. A required minimum distribution payment

A) II, III, and IV
B) II and III
C) I and II
D) I, II, and III

A

D) Explanation
The answer is I, II, and III. All IRA and qualified plan distributions are eligible for rollover treatment except distributions made to satisfy the minimum distribution rules and distributions made as part of a series of substantially equal periodic payments. Required minimum distributions are not eligible for rollover treatment. The law states a nontaxable portion of a qualified plan distribution must be made using a trustee-to-trustee transfer because that is the only reliable way to provide documentation that this money was actually nontaxable.

LO 6.1.1

446
Q

Which of the following statements describing how qualified pension plans differ from SEP and SIMPLE plans is(are) CORRECT?

I. Qualified plan rules provide greater flexibility in the number and makeup of the employees covered by the plan than do the rules pertaining to SEP and SIMPLE plans.

II. Participants must be fully and immediately vested in the contributions to qualified plans, but SEP and SIMPLE plans are permitted to have vesting schedules.

A) Neither I nor II
B) II only
C) Both I and II
D) I only

A

D) Explanation
The answer is I only. Statement II is incorrect because participants must be fully and immediately vested in the contributions to SEPs and SIMPLEs. Qualified plans can include vesting schedules.

LO 4.2.2

447
Q

Basic provisions of SIMPLE IRAs include which of the following?

I. They are subject to actual deferral percentage test (ADP) nondiscrimination rules.

II. Employees are 100% vested in their elective deferrals.

III. Employees are not fully vested in employer contributions until completing five years of service.

IV. Employers with fewer than 100 employees who earned $5,000 during any two preceding years and are reasonably expected to earn at least $5,000 during the current year must be allowed to participate.

A) II, III, and IV
B) I and IV
C) II and IV
D) II and III

A

C) Explanation
The answer is II and IV. Employees are 100% vested in their elective deferrals. Employers with fewer than 100 employees who earned $5,000 during any two preceding years and are reasonably expected to earn at least $5,000 during the current year must be allowed to participate. SIMPLE IRAs are not subject to the nondiscrimination rules generally applicable to qualified plans (including top-heavy rules). The employee is 100% vested in both his elective deferrals as well as any employer contributions.

LO 4.2.1

448
Q

Because a simplified employee pension (SEP) plan is not a qualified plan, it is not subject to all the same rules as qualified plans; however, it is subject to many of the same rules. Which of the following statements when comparing or contrasting a SEP plan to a qualified plan is CORRECT?

A) The maximum contribution possible on behalf of an individual participant is $58,000 (2021).

B) SEP plans and qualified plans do not have the same protection from creditors.

C) SEP plans and qualified plans have different nondiscriminatory and top-heavy rules.

D) SEP plans and qualified plans have different funding deadlines.

A

A) Explanation
The answer is the maximum contribution possible on behalf of an individual participant is $58,000 (2021). Both SEP plans and qualified plans can be funded as late as the due date of the return plus extensions. The maximum contribution possible on behalf of an individual participant is $58,000 (2021). Qualified plans are protected under ERISA and federal bankruptcy law. SEPs share this protection. Both types of plans have the same nondiscriminatory and top-heavy rules.

LO 4.2.2

449
Q

A simplified employee pension (SEP) plan

I. requires employer contributions on a nondiscriminatory basis.

II. can be integrated with Social Security.

III. cannot deny participation to any employee 21 years of age or older based on age.

IV. imposes mandatory employer contributions.

A) I and II
B) III and IV
C) I, II, III, and IV
D) I, II, and III

A

D) Explanation
The answer is I, II, and III. Only statement IV is incorrect. A SEP plan is a retirement plan that uses an IRA as the receptacle for employer/employee contributions. The SEP plan is often a good choice for very small companies because of its low cost and ease of administration. All employer contributions to a SEP plan are discretionary.

LO 4.2.2

450
Q

Section 403(b) plan (tax-sheltered annuity plan or TSA) employer contributions

I. must abide by the annual additions limit.

II. must not discriminate in favor of highly compensated employees.

III. are based on a maximum annual covered compensation of $230,000 in 2021.

IV. are subject to FICA (Social Security and Medicare) and FUTA (federal unemployment) payroll taxes.

A) III and IV
B) II, III, and IV
C) I, II, and III
D) I and II

A

D) Explanation
The answer I and II. A Section 403(b) or TSA plan is subject to the annual additions limit of the lesser of 100% of compensation or $58,000 for 2021. Contributions must not discriminate in favor of highly compensated employees. Employer contributions are based on a maximum annual covered compensation of $230,000 and are not subject to FICA and FUTA payroll taxes (employee deferrals are subject to FICA and FUTA).

LO 4.3.1

451
Q

While Section 403(b) (tax-sheltered annuity plan or TSA) plans are an excellent source of retirement savings, they do have some disadvantages, such as

I. investments are limited to mutual funds and annuities.

II. Section 403(b) plans must comply with the actual contribution percentage (ACP) test for employer matching contributions.

III. actual deferral percentage (ADP) testing causes

IV. Section 403(b)/TSA plans to be relatively costly and complex to administer.
account balances at retirement age are guaranteed to be sufficient to provide adequate retirement amounts for employees who entered the plan at later ages.

A) I, II, III, and IV
B) III and IV
C) I, II, and III
D) I and II

A

D) Explanation
The answer is I and II. Section 403(b)/TSA plan investments are limited to mutual funds and annuities. Although the ADP test does not apply, Section 403(b)/TSA plans must comply with the ACP test for matching contributions. One way to remember that Section 403(b) plans must pass ACP testing and not ADP testing is that Section 403(b) plans are for 501(c)(3) organizations. The (c) in 501(c)(3) is like the “C” in ACP testing. Nondiscrimination testing causes Section 403(b)/TSA plans to be relatively costly and complex to administer. Account balances at retirement age may not be sufficient to provide adequate retirement amounts for employees who entered the plan at later ages.

LO 4.3.1

452
Q

Which of the following statements regarding Section 457 plans is (are) CORRECT?

I. Deductibility of plan contributions is an important factor for employers choosing a Section 457 plan to consider.

II. Earnings on assets in a Section 457 plan grow tax-deferred until withdrawn.

III. Required minimum distribution rules do not apply.

IV. A Section 457 plan is a nonqualified deferred compensation plan.

A) II and III
B) II and IV
C) I, II, III, and IV
D) IV only

A

B) Explanation
The answer is II and IV. Statement I is incorrect. Deductibility of plan contributions is not a factor for employers choosing a Section 457 plan because these employers are tax-exempt. Statement II is correct. Earnings on assets in a Section 457 plan grow tax-deferred until withdrawn. Statement III is incorrect. Required minimum distribution rules apply. Statement IV is correct. A Section 457 plan is a nonqualified deferred compensation plan.

LO 4.3.2

453
Q

A state or local government would choose to establish a Section 457 plan for all the following reasons except

A) tax deductibility of employer contributions.
B) the ability of a participant to make elective deferrals.
C) no early withdrawal penalty on distributions.
D) tax-deferred growth of assets.

A

A) Explanation
The answer is tax deductibility of employer contributions. Because Section 457 plans are sponsored by tax-exempt entities, deductibility of plan contributions is not an issue and would not be a reason to establish such a plan. A Section 457 plan is not a qualified plan and has no early withdrawal penalty on distributions.

LO 4.3.2

454
Q

Sally, age 37, works for two employers, ABC Corporation and XYZ Corporation, both of which maintain Section 401(k) plans. If Sally defers $6,000 to ABC’s Section 401(k) plan in 2021, how much can she then defer to XYZ’s plan this year?

A) $13,500
B) $58,000
C) $31,500
D) $19,500

A

A) Explanation
The answer is $13,500. The maximum allowable elective deferral for 2021 is $19,500. If Sally contributes $6,000 to ABC’s plan, then she can only contribute up to $13,500 to XYZ’s plan ($19,500 − $6,000 = $13,500).

LO 4.4.1

455
Q

Jack is a single taxpayer who retired at age 62 and receives a qualified plan pension of $1,500 each month. He has begun working as a consultant to various firms and is projecting he will earn $70,000 in 2021.

What is the maximum deductible contribution Jack may make to a traditional IRA for 2021?

A) $19,500
B) $6,000
C) $7,000
D) $0

A

C) Explanation
The answer is $7,000. Jack is not currently an active participant in a qualified plan, is age 50 or older, and has earned income in 2021. Jack may make a deductible IRA contribution of $7,000 (6,000 + $1,000 catch-up) for 2021.

LO 5.1.1

456
Q

A traditional IRA is appropriate when

I. it is considered an important supplement or alternative to a qualified pension or profit-sharing plan.

II. sheltering current compensation or earned income from taxation is a taxpayer’s goal.

A) Neither I nor II
B) I only
C) Both I and II
D) II only

A

C) Explanation
The answer is both I and II. A traditional IRA is also suitable when a taxpayer wishes to accumulate assets for retirement or defer taxes on investment income.

LO 5.1.1

457
Q

Thad and Debra, both age 48, are married and will file a joint return. Their 2021 modified adjusted gross income is $120,000 (including Thad’s $95,000 salary). Debra had no earned income of her own. Neither spouse was covered by an employer-sponsored retirement plan. What is the total maximum deductible contribution Thad and Debra may make to a traditional IRA this year?

A) $12,000
B) $0
C) $6,000
D) $4,000

A

A) Explanation
The answer is $12,000. Because neither Thad nor Debra participates in an employer-sponsored retirement plan, they can contribute and deduct $6,000 each for 2021. While Debra has no earned income, a spousal IRA may be established and funded based on Thad’s compensation.

LO 5.1.2

458
Q

Which of the following persons could make tax-deductible contributions to a traditional IRA regardless of their modified adjusted gross income (MAGI)?

I. A person who participates in a SEP IRA
II. A person who participates in a Section 457 plan
III. A person who participates in a Section 401(k) plan
IV. A person who participates in a Section 403(b) plan

A) II and IV
B) I and III
C) II only
D) II and III

A

C)

Explanation
The answer is II only. A person who participates in a qualified plan, SEP IRA, or Section 403(b) plan may not be able to make tax-deductible IRA contributions if the participant’s MAGI exceeds certain limits. Participation in a Section 457 plan does not subject a person to these limitations.

LO 5.1.2

459
Q

Which of the following statements regarding a stretch IRA is CORRECT?

I. It allows the IRA owner’s beneficiary to name his own beneficiary upon the owner’s death.

II. It extends or stretches the period of tax-deferred earnings within an IRA possibly over a decade.

A) Both I and II
B) Neither I nor II
C) II only
D) I only

A

A) Explanation
The answer is both I and II. A stretch IRA extends or stretches the period of tax-deferred earnings within an IRA beyond the lifetime of the original owner, possibly over a decade.

LO 5.2.1

460
Q

Jack inherited his father’s Section 401(k) plan account. Which of the following statements regarding Jack’s options is (are) CORRECT?

I. Jack is permitted to use a direct trustee-to-trustee transfer of the plan balance into an inherited IRA account.

II. If Jack creates an inherited IRA with the benefit, he can designate his own beneficiary to the account.

III. The payout of the benefit in an inherited IRA is over the lifetime of the original account holder, Jack’s father.

A) I and II
B) II and III
C) I only
D) I, II, and III

A

Explanation
The answer is I and II. Statement III is incorrect. As a non-spouse beneficiary, Jack is required to receive distributions over a ten-year period, beginning with the year of the death of the previous owner.

LO 5.2.1

461
Q

Todd, age 60, has made contributions of $75,000 to his traditional IRA, of which $15,000 were nondeductible contributions. He is considering taking a $20,000 distribution from his IRA, which currently has a fair market value of $175,000. When calculating the nontaxable portion of his IRA, which of the following formulas is CORRECT?

I. Nontaxable portion = nondeductible contributions ÷ [(IRA balance at the beginning of the year + the IRA balance at the end of the year) ÷ 2] × IRA distributions

II. Nontaxable portion = nondeductible contributions ÷ (IRA balance at the beginning of the year + any distributions taken during the year) × IRA distributions

III. Nontaxable portion = [(nondeductible contributions prior to current year + all contributions for current year) ÷ (balances at end of current year + distributions received in current year)] × total distributions during current year

A) II only
B) None of these
C) I only
D) III only

A

D) Explanation
The answer is III is only. The correct formula is:

nontaxable portion = [(nondeductible contributions prior to current year + all contributions for current year) ÷ (balances at end of current year + distributions received in current year)] × total distributions during current year

LO 5.3.1

462
Q

The premature distribution penalty does not apply to which of the following IRA distributions?

I. A distribution paid to a beneficiary after the death of the IRA owner who had not begun receiving minimum distributions

II. A distribution made after the owner is age 55 and after separation from service

III. A distribution made for purpose of paying qualified higher education costs

A) I and II
B) I and III
C) I only
D) III only

A

B)Explanation
The answer is I and III. A distribution made after the owner is age 55 and after separation from service is only an exception for distributions from qualified plans, not from IRAs.

LO 5.3.1

463
Q

Question #49 of 85
Question ID: 1340227
Robert established a Roth IRA. He turns age 72 this year. Which of the following statements is (are) CORRECT?

I. Robert must begin taking required minimum distributions (RMDs) by April 1 of next year.

II. Robert can no longer make contributions to the Roth IRA.

A) I only
B) Neither I nor II
C) II only
D) Both I and II

LO 5.4.1

A

B) Explanation
The answer is neither I nor II. The original owner of a Roth IRA is never subject to an RMD requirement during his lifetime and can continue making contributions after he reaches age 72.

464
Q

What is the taxable character of distributions that are made from a Roth IRA?

A)
Tax-free income if the distribution meets the holding period and qualified distribution requirements
B)
Tax-deferred income when converted to a traditional IRA
C)
Capital gain income if the distribution meets the required holding period
D)
Ordinary income if the taxpayer fails to make required minimum distributions
Explanation
The answer is tax-free income if the distribution meets the holding period and qualified distribution requirements. Distributions made from a Roth IRA are income tax free if the Roth IRA meets certain specified conditions. These conditions include meeting the 5-year holding period and 1 of the following: (1) the owner is age 59½ or older, (2) distribution upon disability of the owner, (3) distribution to a beneficiary upon the death of the owner, or (4) for a first-time home purchase (up to a $10,000 lifetime cap).

LO 5.4.1

A

A)

465
Q

Question #51 of 85
Question ID: 1250084
Which of the following statements is NOT correct regarding the conversion of a traditional IRA to a Roth IRA?

A) The IRA owner’s modified adjusted gross income (MAGI) cannot exceed $100,000 in the year of the conversion.

B) An amount distributed from a traditional IRA can be rolled over to a Roth IRA within 60 days of the distribution.

C) An amount in a traditional IRA may be transferred in a trustee-to-trustee transfer from the trustee of the traditional IRA to the trustee of the Roth IRA.

D) An amount in a traditional IRA may be transferred to a Roth IRA maintained by the same trustee.

A

A)Explanation
The answer is the IRA owner’s modified adjusted gross income (MAGI) cannot exceed $100,000 in the year of the conversion. There is no MAGI limit for a taxpayer in the year in which there is a conversion.

LO 5.4.2

466
Q

Question #52 of 85
Question ID: 1250085
Which of the following statements regarding the tax effects of converting a traditional IRA to a Roth IRA is (are) CORRECT?

I. The converted amount is treated as a taxable distribution from the traditional IRA.

II. The 10% premature penalty applies if the owner is not at least 59½ years old.

A) Both I and II
B) Neither I nor II
C) I only
D) II only

A

C)Explanation
The answer is I only. When a traditional IRA is converted to a Roth IRA, the converted amount is treated as a taxable distribution and is included in the owner’s gross income. The 10% penalty does not apply to the conversion amount when converted, regardless of the owner’s age. However, if the taxable portion of the converted amount is withdrawn within five years of the conversion, then the taxable portion of the conversion is treated as coming out first when the converted amount is withdrawn. This taxable amount would be subject to the early withdrawal rules and penalized 10% unless an exception applies. The point of this rule is to protect Roth conversions from being sham transactions intended to get around the 10% early withdrawal penalty.

The law treats withdrawals of converted amounts that are more than five years past the conversion date the same as contributions.

LO 5.4.2

467
Q

Question #53 of 85
Question ID: 1340228
Which of the following statements regarding Roth IRAs and pre-tax 401(k) plans is (are) CORRECT?

Roth IRAs require distributions no later than age 72 while the participant is living.
There is not an income limitation to participate in a pre-tax 401(k) plan or Roth IRA.
A)
II only
B)
Both I and II
C)
Neither I nor II
D)
I only
A

C)Explanation
The answer is neither I nor II. Although there is not an income limitation to participate in a pre-tax 401(k) plan, there are income limits for Roth IRAs (2021 – modified AGI: $208,000 married/$140,000 single). For Roth IRAs, there is no requirement to start taking distributions while the participant is living. For pre-tax 401(k) plans, distributions must begin no later than age 72, unless the participant is still working at the company and is not a 5% owner.

LO 5.4.3

468
Q

Question #54 of 85
Question ID: 1250092
Which of the following retirement plans generally permit in-service withdrawals at any age?

I. Money purchase pension plan
II. Profit-sharing plan
III. Section 401(k) plan
IV. SEP plan

A) I, II, III, and IV
B) II, III and IV
C) I, II, and IV
D) I, II, and III

A

B)Explanation
The answer is II, III, and IV. Pension plans (traditional defined benefit, cash balance, target benefit, or money purchase) generally do not allow in-service withdrawals to participants under the age of 62. All the others listed allow for in-service withdrawals if the plan document permits.

LO 6.1.1

469
Q

Question #55 of 85
Question ID: 1250093
A preretirement distribution from a qualified retirement plan can escape the 10% early withdrawal penalty in each of the following situations except

A) distributions made after a separation from service at any age.

B) distributions made to a beneficiary or to an employee’s estate on or after the employee’s death.

C) distributions made to cover medical expenses in excess of 10% of adjusted gross income for a taxpayer who is age 55.

D) distributions made as part of a series of substantially equal periodic payments made at least annually over the life or life expectancy of the employee or the joint lives or life expectancies of the employee and beneficiary.

LO 6.1.1

A

A)Explanation
The answer is distributions made after a separation from service for early retirement after any age. For a preretirement distribution to escape the 10% penalty for early distribution, the distribution must be made after a separation from service in the year the worker turns age 55 or later. This exception is not applicable to IRAs.

470
Q

Question #56 of 85
Question ID: 1250094
Steve retired from ABC Corporation this year and received a lump-sum distribution from ABC’s qualified retirement plan. The distribution consisted entirely of ABC stock valued at $200,000 on the date of distribution. The fair market value of the stock at the time of contribution to the plan was $80,000. Assuming Steve does not sell the stock this year, what amount is included in Steve’s gross income this year as a result of the distribution?

A) $120,000
B) $200,000
C) $80,000
D) $0

A

C)Explanation
The answer is $80,000. Because the distribution is a lump-sum distribution of employer stock, the net unrealized appreciation (NUA) concept applies. Under the NUA rules, the adjusted basis of the stock to the plan trust ($80,000) is included in Steve’s gross income in the year of the distribution and is treated as ordinary income. When Steve later sells the stock, he will have an $80,000 basis in it.

LO 6.1.2

471
Q

Question #57 of 85
Question ID: 1250095
When she retired at age 64, Lauren received a lump-sum distribution from her employer’s stock bonus plan. The fair market value of the employer stock contributed to her account was $200,000 at the time of contribution. At the time of the distribution, the employer stock in Lauren’s account had a fair market value of $300,000. Six months later, Lauren sold the stock for $310,000. Which of the following statements regarding the sale of Lauren’s stock is(are) CORRECT?

I. The $300,000 distribution is taxed at the long-term capital gain rate.

II. Lauren has a $10,000 short-term capital gain when the stock is sold.

III. There was no income tax liability incurred when the stock was contributed to the plan.

IV. The net unrealized appreciation (NUA) on the stock is $100,000.

A) I and II
B) I, II, III, and IV
C) II, III, and IV
D) IV only

A

C) Explanation
The answer is II, III, and IV. Of the $300,000 Lauren received as a lump-sum distribution from the stock bonus plan, $100,000 is net unrealized appreciation (NUA) and will be taxed at the long-term capital gain rate. The remaining $200,000 is taxed at Lauren’s ordinary income tax rate in the year of the lump-sum distribution. Because Lauren sold the stock within six months of distribution, the $10,000 post-distribution appreciation is taxed as short-term capital gain.

LO 6.1.1

472
Q

Question #58 of 85
Question ID: 1340235
Blake, age 72, is required to take substantial required minimum distributions (RMDs) from his qualified retirement plan. He has no current need for the income and wants to decrease the amount of the distributions without incurring a penalty. Blake is not interested in a lump-sum distribution from the plan at this time. Which of the following statements regarding Blake’s options is CORRECT?

I. Blake may take a distribution in addition to his RMD from his qualified plan and convert the additional distribution to a Roth IRA within 60 days.

II. Blake cannot roll over retirement plan proceeds to a traditional IRA after age 72.

A) II only
B) Neither I nor II
C) Both I and II
D) I only

LO 6.2.1

A

D)Explanation
The answer is I only. If Blake takes a distribution that is in addition to his RMD, he can pay the required income tax on this distribution and convert it to a Roth IRA. Because there are no required minimum distributions for Roth IRAs, Blake will have effectively reduced the amount of his pretax retirement plan account against which he must calculate subsequent RMD. Statement II is incorrect. A direct transfer or rollover may occur at any age.

473
Q

Question #59 of 85
Question ID: 1340236
Required minimum distributions from a traditional IRA must begin no later than

A)
the year in which the IRA owner attains age 72.
B)
age 59½.
C)
April 1 of the year following the year in which the IRA owner attains age 72.
D)
December 31 of the year following the IRA owner’s attainment of age 72.

A

C)Explanation
The answer is April 1 of the year following the year in which the IRA owner attains age 72. The Internal Revenue Code provides that minimum distributions from a traditional IRA must begin no later than April 1 of the year following the year in which the IRA owner attains age 72.

LO 6.2.2

474
Q

Question #60 of 85
Question ID: 1250103
Which of the following qualified plan distributions is subject to the 10% penalty for early withdrawal?

A) An in-service hardship distribution from a Section 401(k) plan to an employee-participant, age 55

B) A lump-sum benefit payable to a disabled employee-participant, age 57

C) A lump-sum distribution made to an employee-participant, age 63, from a profit-sharing plan after the funds have been in the plan for two years

D) A death benefit payable to a beneficiary upon the death of an employee, age 52

A

A)Explanation
The answer is an in-service hardship distribution from a Section 401(k) plan to an employee-participant, age 55. Even if the distribution is a hardship withdrawal, the penalty applies unless the employee-participant has attained the age of 59½ (or one of the other 10% penalty exceptions applies).

LO 6.3.1

475
Q

Question #64 of 85
Question ID: 1250111
Sally, age 60, has received a $50,000 distribution from $100,000 she is to receive from her ex-husband’s (who is age 55) qualified plan account under a qualified domestic relation order (QDRO). Which of the following statements is CORRECT regarding the QDRO and the distributed funds?

I. Irrespective of the plan document, Sally may demand an immediate cash distribution of the remaining funds from the plan trustees.

II. She may roll over the $50,000 distribution into an IRA.

III. Sally’s ex-husband is not subject to an early distribution penalty in the execution of the QDRO.

IV. Sally may be required to leave the remaining funds with the plan trustee until the earliest time for distributions under the plan.
A) II and III
B) I, II, and III
C) I, II, III, and IV
D) II, III, and IV
A

D)Explanation
The answer is II, III, and IV. A trustee may not be forced to distribute assets from a plan unless the plan document allows for it.

LO 6.5.1

476
Q

Question #67 of 85
Question ID: 1250116
If a covered worker were to become disabled for Social Security benefit purposes, which of the following individuals would be eligible to also receive a benefit from Social Security based on the disabled worker’s record?

A dependent parent of the disabled worker, age 62 or over
The disabled worker, age 62 or over
The spouse of the disabled worker
An unmarried dependent child of the disabled worker, under age 19 and still in high school

A) II, III, and IV
B) I, II, III, and IV
C) III and IV
D) I and II

A

A) Explanation
The answer is II, III, and IV. The dependent parent of a disabled worker is not eligible for benefits under the disabled worker’s employment record when the worker receives Social Security disability benefits.

LO 7.2.1

477
Q

Question #72 of 85
Question ID: 1250124
Build Corporation communicated several goals to its financial adviser when it decided to implement a qualified retirement plan for the company. After reviewing these goals, the adviser recommended that Build Corporation implement a defined benefit pension plan. Which of the following statements regarding employers who are candidates for a defined benefit pension plan are CORRECT?

I. Typically, they have the objective of instituting a plan with highly predictable costs.

II. They indicate that the desire to provide a tax shelter for key employees outweighs the need for an administratively convenient plan.

III. They want benefit levels guaranteed.

IV. They want a simple and inexpensive plan.

A) I and IV
B) I, II, and III
C) II and III
D) I, II, III, and IV

A

C) Explanation
The answer is II and III. Employers who sponsor defined benefit pension plans typically do not have highly predictable costs, because funding is subject to an annual actuarial determination. Highly paid employees receive a tax shelter with a defined benefit plan in the sense that their compensation includes large contributions to the defined benefit and they are not subject to current income tax on this compensation. The Pension Benefit Guaranty Corporation (PBGC) and the employer guarantee benefit levels. Defined benefit pension plans are complex to design and expensive to install and maintain.

LO 8.1.2

478
Q

Larry is a sole proprietor of a business with 15 employees. He would like to implement a formal retirement plan for his business. Larry is 55 years old and is planning to retire in 10 years at age 65. His company currently has a strong cash flow, which is expected to continue. Larry’s own personal savings retirement need is $85,000 per year, and he pays himself $95,000 annually. The company can afford to contribute $100,000 this year for Larry’s account to any retirement plan that is implemented.

Larry will also commit to an annual contribution necessary to fund the retirement plan if needed. Based on limited information, which of the following types of qualified retirement plans would you recommend for Larry and his business?

A) Traditional defined benefit pension plan
B) Money purchase pension plan
C) Profit-sharing plan
D) Stock bonus plan

A

A) Explanation
The answer is traditional defined benefit pension plan. This type of plan is most appropriate for Larry and his business. The business has favorable cash flow and can commit to the annual contribution required by the defined benefit approach. Additionally, Larry’s savings need as a percentage of his compensation exceeds anything possible in a defined contribution plan. Finally, Larry is currently age 55 with only 10 years until retirement.

LO 8.1.2

479
Q

Martha has been impressed with the appreciation of the coin collection she received as a gift from her mother and would like to take advantage of this by using coins as an investment in the IRAs. Which of the following statements regarding coins as investments in IRAs is CORRECT?

A) Any government-issued gold coins, such as Krugerrands and American Eagles, are appropriate for IRA investment.

B) Martha should approach her coin dealer and ask that a collection similar to the collection her mother gave her be created as an investment for the IRAs.

C) American Eagle gold coins are permitted IRA assets.

D) No more than 25% of Martha’s IRA assets may be invested in coins.

A

A) Explanation
The answer is American Eagle gold coins are permitted IRA assets. Only permissible collectible that an IRA may invest in is certain U.S. coins, such as the American Eagle gold coin.

LO 8.2.1

480
Q

Why would a qualified retirement plan include real estate among its portfolio of investment assets?

A) Because real estate is always considered to be a very low risk holding
B) To fund future fixed obligations
C) To provide increased liquidity
D) As an inflation hedge

A

D) Explanation
The answer is as an inflation hedge. The inclusion of real estate among a retirement plan’s investment portfolio is most suitable as a hedge against future inflation. Otherwise, it is generally nonliquid, is not stable enough to fund fixed obligations, and is not always considered a very low risk holding.

LO 8.2.1

481
Q

Question #76 of 85
Question ID: 1250131
Richard participates in a traditional defined benefit pension plan at work. His projected monthly benefit under the plan is $1,000. If the plan provides life insurance for Richard, the death benefit payable under the policy is limited to

A) $215,000.
B) $52,000.
C) $17,500.
D) $100,000.

A

D) Explanation
The answer is $100,000. Defined benefit plans use the 100 times test for determining whether they comply with the incidental benefit rules. Under this test, the death benefit cannot exceed 100 times the participant’s projected monthly benefit (in this instance, $100,000).

LO 8.2.2

482
Q

Question #77 of 85
Question ID: 1250134
Regarding assumptions used in retirement needs analysis calculations, which of the following is (are) CORRECT?

All other things being equal, increasing the life expectancy of the retiree will lower the amount of capital needed on the first day of retirement to support the assumed retirement income.
All other things being equal, changing the assumed rate of return from 6% to 8% and the assumed inflation rate from 2% to 4%, will lower the amount of capital needed on the first day of retirement to support the assumed retirement income.
A)
Both I and II
B)
I only
C)
II only
D)
Neither I nor II
A

D) Explanation
The answer is neither I nor II. Increasing the life expectancy of the retiree increases the amount of capital needed on the first day of retirement to support the assumed retirement income because the retiree will draw on the capital fund for a longer period. Statement II is incorrect because an 8% investment return and 4% inflation rate produces a lower inflation-adjusted rate of return, and thus increases the amount of capital needed on the first day of retirement to support the assumed retirement income.

[(1.06 ÷ 1.02) − 1] × 100 = 3.9216%; [(1.08 ÷ 1.04) − 1] × 100 = 3.8462%

LO 8.3.1

483
Q

Question #78 of 85
Question ID: 1250135
Reed, age 45, has come to you for help in planning his retirement. He works for a manufacturing company, where he earns a salary of $75,000. Reed would like to retire at age 65. He feels this is a realistic goal because he has consistently earned 9% on his investments and inflation has only averaged 3%. Assuming he is expected to live until age 90 and he has a wage replacement ratio of 80%, how much will Reed need to have accumulated on the day that he retires to adequately provide for his retirement lifestyle?

A) $1,863,311
B) $1,490,649
C) $1,160,240
D) $1,408,595

A

B) Explanation
The answer is $1,490,649.

Step 1: Determine the present value of capital needs:

Current income $75,000
Wage replacement ratio × 80%
Present value of capital needs $60,000
Step 2: Determine the future value of the capital needs in the first year of retirement:

PV of capital needs ($60,000)
n (number of years until retirement) 20
i (use inflation rate) 3
FV (required income in the first year of retirement) $108,366.6741
Step 3: Determine the amount of savings (capital) needed at retirement to fund expenses throughout remainder of life expectancy:

PMTAD(annuity due) ($108,366.6741)
n (retirement life expectancy) 25 (90 − 65)
i (use real rate of return) 5.8252 [(1.09 ÷ 1.03) − 1] × 100
PV (capital needed at retirement) $1,490,649
LO 8.3.2

484
Q

Question #79 of 85
Question ID: 1250136
Reed, age 45, has come to you for help in planning his retirement. He works for a manufacturing company, where he earns a salary of $75,000. Reed would like to retire at age 65. He feels this is a realistic goal because he has consistently earned 9% on his investments and inflation has averaged 3%. If Reed expects to live until age 90 and he has a wage replacement ratio of 80%, assuming a capital preservation approach, how much will Reed need to have accumulated on the day that he retires to adequately provide for his retirement lifestyle?

A) $1,971,034
B) $1,663,516
C) $1,490,649.
D) $1,010,264

A

B)Explanation
The answer is $1,663,516.

Step 1: Determine the present value of capital needs:

Current income $75,000
Wage replacement ratio × 80%
Present value of capital needs $60,000
Step 2: Determine the future value of the capital needs in the first year of retirement:

PV of capital needs ($60,000)
n (number of years until retirement) 20
i (use inflation rate) 3%
FV (required income in the first year of retirement) $108,366.6741
Step 3: Determine the amount of savings (capital) needed at retirement to fund expenses throughout remainder of life expectancy using a capital utilization approach:

PMTAD(annuity due)
($108,366.6741)

n (retirement life expectancy) 25 (90 − 65)
i (use real rate of return) 5.8252 [(1.09 ÷ 1.03) − 1] × 100
PV (capital needed at retirement) $1,490,649
Step 4: For capital preservation, calculate the additional capital needed at retirement to generate the desired retirement income and leave a balance at life expectancy equal to the original capital utilization value:

FV = $1,490,649

n = 25

i = 9

Solve PV = (172,867)

Add to previous step: $1,490,649 + $172,867 = $1,663,516

LO 8.3.2

485
Q

Question #80 of 85
Question ID: 1340244
(Case Study Question)

Chris and Allison Bernard are dedicated to reducing living expenses to be able to save more for retirement. The couple is not counting any possible income from any potential part-time earnings for Allison because that income is not yet realized. Assuming they create the necessary discretionary income, what is the total maximum deductible contribution they can make to a traditional IRA together for 2021?

A) $12,000
B) $0
C) $9,000
D) $6,000

A

A) Explanation
The answer is $12,000. The maximum deductible contribution for the Bernards to a traditional IRA for 2021 is $12,000 ($6,000 each). Chris is an active participant in a qualified plan, but his AGI is below the married filing jointly (MFJ) phaseout of $105,000. If Allison has no earned income in 2021, she can make an IRA contribution based on Chris’s earned income, and her AGI for these purposes is below the nonparticipant spouse AGI limit of $198,000.

LO 9.1.1

486
Q

Question #81 of 85
Question ID: 1250141
(Case Study Question)

If Chris died today, what is the total lump-sum death benefit that would be paid by Social Security to his family?

A) $510
B) $255
C) $1,020
D) $1,275

A

B) Explanation
The answer is $255. It is the total amount paid, regardless of the number of Social Security beneficiaries.

LO 9.1.1

487
Q

Question #82 of 85
Question ID: 1340245
(Case Study Question)

Chris’s employer, Gilmore Glass Company, is considering making an additional profit-sharing contribution to the company’s Section 401(k) plan for 2021. What is the maximum amount of additional profit-sharing contribution the company could make on behalf of Chris for 2021?

A)
$19,500
B)
$53,800
C)
$15,300
D)
$58,000
A

B) Explanation
The answer is $53,800. Chris contributes 3% of his salary ($70,000 × 3% = $2,100) and the employer matches employee deferrals up to that amount. The maximum that can be contributed to Chris’s profit-sharing account is the lesser of 100% of his compensation, or $58,000. Because $2,100 is going in as an elective deferral from Chris’s salary and another $2,100 is the match, that leaves $53,800 the employer could contribute.

LO 9.1.2

488
Q

Question #83 of 85
Question ID: 1250145
(Case Study Question)

Assuming Chris Bernard’s income increases by 3% per year between now and age 62, what lump sum will be necessary at the date of retirement to provide Chris’s goal of 80% of preretirement income for 25 years in retirement without considering Social Security? Assume the Bernards’ required rate of return in the calculation.

A) $1,400,000
B) $839,790
C) $1,388,038
D) $2,313,975

A

C) Explanation
The answer is $1,388,038. The lump sum needed at the date of retirement to support Chris’s goal (without considering Social Security) is $1,388,038.

Step 1:

$70,000 × 80% = $56,000

$56,000 +/− PV

17 N

3 I/YR

Solve FV

$92,559

Step 2:

BEG mode

$92,559 PMT

25 N

[(1.08 ÷ 1.03) − 1 × 100] = 4.8544 I/YR

0 FV

Solve PV

−$1,388,038

LO 9.2.1

489
Q

Question #84 of 85
Question ID: 1340246
(Case Study Question)

A local competitor company has been aggressively recruiting Chris. If Chris leaves Gilmore Glass, which of the following statements regarding the tax status of any outstanding loan from his Section 401(k) plan is (are) CORRECT if Chris cannot arrange to pay off the loan?

I. Chris may roll over the outstanding loan to an IRA and continue paying the loan under the same rules as before..

II. The outstanding loan is subject to income tax if not repaid.

III. The outstanding loan may be subject to an additional 10% excise penalty.

IV. The loan balance continues to be tax free because Chris is essentially borrowing from himself.

A) II and III
B) I, III, and III
C) II only
D) III and IV

A

A) Explanation
The answer is II and III. No one can roll a retirement plan loan to an IRA and keep the payments and term the same. By using a plan loan offset, Chris could lower the amount of the defaulted loan by making payments into his new employer’s plan or an IRA up to the due date (including extensions) of his tax return for the year of the retirement plan loan default. The loan becomes fully taxable if not repaid and may be subject to an additional 10% excise penalty.

LO 9.2.2

490
Q

Question #85 of 85
Question ID: 1250149
(Case Study Question)

If you are unable to obtain sufficient and relevant quantitative information and documents from the Bernards to form a basis for recommendations, which of the following could you, a CFP® professional, do to follow the Code of Ethics and Standards of Conduct?

I. Terminate the engagement.
II. Restrict the scope of the engagement.

A) II only
B) I only
C) Both I and II
D) Neither I nor II

A

C) Explanation
The answer is both I and II. According to the Code and Standards, “if the practitioner is unable to obtain sufficient and relevant quantitative information and documents to form a basis for recommendations, the practitioner shall either restrict the scope of the engagement to those matters for which sufficient information is available; or terminate the engagement.”

LO 9.3.2